Y arctg x график функции: Функция y = arctgx — урок. Алгебра, 11 класс.

404 — Страница не найдена

Страницы

Партнеры сайта

_________________________________


404: Запрошенная страница с адресом [http://primer. by/algebra/funkcii/funkcija-yarctgx] не найдена.

Если Вы уверены, что набрали ссылку корректно, напишите, пожалуйста, об этом на:

меню пользователя

Новости


30.11.16 


17.03.15 


25.03.14 


29.08.13 


05. 05.13 



primer. by 2013-2016

Элементарная математика

  

Сканави М.И. Элементарная математика. 2-е изд., перераб. и доп., М.: 1974г. — 592с.

Книга представляет собой повторительный курс элементарной математики и рассчитана на тех, кто хочет пополнить, укрепить и систематизировать свои знания. Как и в первом издании, содержание ориентировано на программы вступительных экзаменов в технические вузы и, в особенности, на программы подготовительных отделений при высших учебных заведениях, для учащихся которых, как мы надеемся, книга окажется полезной.

(Книга включает в себя Ч1 — Арифметика, алгебра и элементарные функции и Ч2 — Геометрия. Каждый раздел включает в себя теоретическую часть и большое количество задач с решениями.)



Оглавление

ВВЕДЕНИЕ
Часть первая. АРИФМЕТИКА, АЛГЕБРА И ЭЛЕМЕНТАРНЫЕ ФУНКЦИИ
Глава I. ДЕЙСТВИТЕЛЬНЫЕ И КОМПЛЕКСНЫЕ ЧИСЛА
2. Простые и составные числа. Признаки делимости.
3. Наибольший общий делитель и наименьшее общее кратное.
4. Целые числа. Рациональные числа.
5. Десятичные дроби. Представление рациональных чисел десятичными дробями.
6. Иррациональные числа. Действительные числа.
7. Действия с приближенными числами.
8. Числовая ось. Координаты точки на плоскости.
§ 2. Степени и корни
9. Степени с натуральными показателями.
10. Степени с целыми показателями.
11. Корни.
12. Степени с рациональными показателями. Степени с действительными показателями.
13. Алгоритм извлечения квадратного корня.
§ 3. Комплексные числа
14. Основные понятия и определения.
15. Рациональные действия с комплексными числами.
16. Геометрическое изображение комплексных чисел. Тригонометрическая форма комплексного числа.
17. Действия с комплексными числами, заданными в тригонометрической форме. Формула Муавра.
18. Извлечение корня из комплексного числа.
Глава II. ТОЖДЕСТВЕННЫЕ ПРЕОБРАЗОВАНИЯ
19. Алгебраические выражения. Одночлены и многочлены.
20. Формулы сокращенного умножения.
21. Бином Ньютона.
22. Разложение многочлена на множители.
23. Дробные алгебраические выражения.
§ 2. Иррациональные алгебраические выражения
24. Радикалы из алгебраических выражений.
25. Освобождение от иррациональности в знаменателе дроби.
Глава III. ЛОГАРИФМЫ
26. Определение и свойства логарифмов.
27. Логарифмы по различным основаниям. Модуль перехода.
§ 2. Десятичные логарифмы
28. Характеристика и мантисса десятичного логарифма.
29. Применение десятичных логарифмов к вычислениям. n.
41. Обратная пропорциональная зависимость. Степенная функция с рациональным показателем степени.
42. Показательная функция.
43. Логарифмическая функция.
§ 3. Преобразование графиков
44. Параллельный сдвиг графика.
45. График квадратного трех члена.
46. График дробно-линейной функции.
47. Преобразование симметрии. Сжатие и растяжение графика.
48. Построение графиков функций.
49. Сложение графиков.
§ 4. Некоторые сведения о рациональных функциях
50. Целые и дробные рациональные функции. Деление многочленов.
51. Схема Горнера. Теорема Безу.
52. Нули многочлена. Разложение многочлена на множители.
Глава V. УРАВНЕНИЯ
53. Уравнение. Корни уравнения.
54. Равносильные уравнения.
55. Системы уравнений.
56. Графическое решение уравнений.
§. 2. Алгебраические уравнения с одной неизвестной
57. Число и кратность корней.
58. Уравнения первой степени (линейные уравнения).
59. Уравнения второй степени (квадратные уравнения).
60. Формулы Виета. Разложение квадратного трехчлена на множители.
61. Исследование квадратного уравнения.
62. Уравнения высших степеней. Целые корни.
63. Двучленные уравнения.
64. Уравнения, сводящиеся к квадратным.
65. Возвратные уравнения.
§ 3. Системы алгебраических уравнений
66. Линейные системы.
67. Определители второго порядка. Исследование линейных систем двух уравнений с двумя неизвестными.
68. Системы, состоящие из уравнения второй степени и линейного уравнения.
69. Примеры систем двух уравнений второй степени. Системы уравнений высших степеней.
§ 4. Иррациональные, показательные и логарифмические уравнения
70. Иррациональные уравнения.
71. Показательные уравнения.
72. Логарифмические уравнения.
73. Разные уравнения. Системы уравнений.
Глава VI. НЕРАВЕНСТВА
74. Свойства неравенств. Действия над неравенствами.
75. Алгебраические неравенства.
§ 2. Решение неравенств
76. Множество решений неравенства. Равносильные неравенства.
77. Графическое решение неравенств.
79. Квадратные неравенства.
80. Неравенства высших степеней. Неравенства, содержащие дробные рациональные функции от х.
81. Иррациональные, показательные и логарифмические неравенства.
82. Неравенства с двумя неизвестными.
Глава VII. ПОСЛЕДОВАТЕЛЬНОСТИ
83. Числовая последовательность.
84. Предел числовой последовательности.
85. Бесконечно малые. Правила предельного перехода.
§ 2. Арифметическая прогрессия
86. Арифметическая прогрессия. Формула общего члена.
87. Свойства арифметической прогрессии.
88. Формула для суммы n членов арифметической прогрессии.
§ 3. Геометрическая прогрессия
89. Геометрическая прогрессия. Формула общего члена.
90. Свойства геометрической прогрессии.
91. Формулы для суммы n членов геометрической прогрессии.
92. Бесконечно убывающая геометрическая прогрессия.
Глава VIII. ТРИГОНОМЕТРИЧЕСКИЕ ФУНКЦИИ УГЛА (ДУГИ)
93. Вектор, проекция вектора.
94. Положительные углы и дуги, меньшие 360°.
95. Углы и дуги, большие 360°.
96. Отрицательные углы. Сложение и вычитание углов.
§ 2. Тригонометрические функции произвольного угла
97. Определение основных тригонометрических функций.
98. Изменение основных тригонометрических функций при изменении угла от 0 до 2pi.
§ 3. Соотношения между тригонометрическими функциями одного и того же угла
99. Основные тригонометрические тождества.
100. Вычисление значений тригонометрических функций по значению одной из них.
101. Значения тригонометрических функций некоторых углов.
§ 4. Четность, нечетность и периодичность тригонометрических функций
102. Четность и нечетность.
103. Понятие периодической функции.
104. Периодичность тригонометрических функций.
§ 5. Формулы приведения
105. Зависимость между тригонометрическими функциями дополнительных углов.
106. Формулы приведения.
Глава IX. ТРИГОНОМЕТРИЧЕСКИЕ ФУНКЦИИ ЧИСЛОВОГО АРГУМЕНТА И ИХ ГРАФИКИ
§ 1. Тригонометрические функции числового аргумента
108. Области определения и области изменения значений тригонометрических функций.
109. Некоторые неравенства и их следствия.
§ 2. Графики тригонометрических функций
110. Первоначальные сведения о таблицах тригонометрических функций.
111. Основные графики.
112. Примеры построения графиков некоторых других тригонометрических функций.
113. Дальнейшие примеры построения графиков функций.
Глава X. ПРЕОБРАЗОВАНИЕ ТРИГОНОМЕТРИЧЕСКИХ ВЫРАЖЕНИЙ
114. Расстояние между двумя точками на плоскости.
115. Косинус суммы и разности двух аргументов.
116. Синус суммы и разности двух аргументов.
117. Тангенс суммы и разности двух аргументов.
118. О формулах сложения для нескольких аргументов.
§ 2. Формулы для двойного и половинного аргумента. Выражение sin na и cos na через степени sin a и cos a
119. Тригонометрические функции двойного аргумента.
120. Выражение sin na и cos na через степени sin a и cos a при натуральном числе n.
121. Тригонометрические функции половинного аргумента.
122. Выражение основных тригонометрических функций аргумента а через tg(a/2).
§ 3. Преобразование в сумму выражений вида sina•cosb, cosa•cosb и sinа•sinb
§ 4. Преобразование в произведение сумм вида
§ 5. Преобразование некоторых выражений в произведения с помощью введения вспомогательного аргумента
127. Преобразование в произведение выражения a•sina + b•cosa.
128. Преобразование в произведение выражений a•sina+b и a•cosa+b
129. Преобразование в произведение выражения a•tga+b.
Глава XI. ОБРАТНЫЕ ТРИГОНОМЕТРИЧЕСКИЕ ФУНКЦИИ И ИХ ГРАФИКИ
130. Функция у = arcsin x (арксинус).
131. Функция y = arccos x (арккосинус).
132. Функция y = arctg x (арктангенс).
133. Функция y = arcctg x (арккотангенс).
134. Пример.
§ 2. Операции над обратными тригонометрическими функциями
135. Тригонометрические операции.
136. Операции сложения (вычитания).
§ 3. Обратные тригонометрические операции над тригонометрическими функциями
137. Функция у = arcsin (sin x).
138. Функция y = arctg (tg x).
Глава XII. ТРИГОНОМЕТРИЧЕСКИЕ УРАВНЕНИЯ И НЕРАВЕНСТВА
139. Уравнение sin х = а.
140. Уравнение cos х = a.
141. Уравнение tg x = a.
142. Уравнение ctg x = a.
143. Некоторые дополнения.
§ 2. Способ приведения к одной функции одного и того же аргумента
145. Некоторые типы уравнений, приводящихся к уравнениям относительно функции одного аргумента.
146. Способ разложения на множители.
147. Решение рациональных тригонометрических уравнений с помощью универсальной тригонометрической подстановки tg(x/2) = t.
§ 3. Некоторые частные приемы решения тригонометрических уравнений и систем
148. Введение вспомогательного аргумента.
149. Преобразование произведения в сумму или разность.
150. Переход к функциям удвоенного аргумента.
151. Решение уравнения типа…
152. Применение подстановок sinx ± соsx = y.
§ 4. Решение тригонометрических неравенств
154. Простейшие тригонометрические неравенства.
155. Примеры тригонометрических неравенств, сводящихся к простейшим.
Часть вторая. ГЕОМЕТРИЯ
156. Точка. Прямая. Луч. Отрезок.
157. Плоскость. Фигуры и тела.
160. Равенство фигур. Движение.
161. Равенство тел.
§ 2. Измерение геометрических величин
162. Сложение отрезков. Длина отрезка.
163. Общая мера двух отрезков.
164. Сравнительная длина отрезков и ломаных.
165. Измерение углов.
166. Радианная мера угла.
167. Измерение площадей.
168. Площадь прямоугольника. Объем прямоугольного параллелепипеда.
Глава XIV. ПЕРПЕНДИКУЛЯРНЫЕ И ПАРАЛЛЕЛЬНЫЕ ПРЯМЫЕ. ЗАДАЧИ НА ПОСТРОЕНИЕ
169. Перпендикуляр и наклонные.
170. Свойство перпендикуляра, проведенного к отрезку в его середине.
171. Параллельные прямые.
172. Углы, образованные двумя параллельными прямыми и секущей.
173. Углы с параллельными или перпендикулярными сторонами.
§ 2. Геометрические места точек. Окружность
174. Геометрическое место точек.
175. Свойство биссектрисы угла.
176. Окружность.
177. Взаимное расположение прямой и окружности. Касательная и секущая.
178. Хорда и диаметр. Сектор и сегмент.
179. Взаимное расположение двух окружностей.
§ 3. Основные задачи на построение
181. Деление отрезка пополам. Построение перпендикуляров.
182. Построение углов.
183. Другие задачи на построение.
Глава XV. ТРЕУГОЛЬНИКИ, ЧЕТЫРЕХУГОЛЬНИКИ
184. Стороны и углы треугольника.
185. Биссектрисы треугольника. Вписанная окружность.
186. Оси симметрии сторон треугольника. Описанная окружность.
187. Медианы и выcоты треугольника.
188. Равенство треугольников.
189. Построение треугольников.
190. Равнобедренные треугольники.
191. Прямоугольные треугольники.
§ 2. Параллелограммы
192. Четырехугольники.
193. Параллелограмм и его свойства.
194. Прямоугольник.
§ 3. Трапеция
196. Трапеция.
197. Средняя линия треугольника.
198. Средняя линия трапеции.
199. Деление отрезка на равные части.
§ 4. Площади треугольников и четырехугольников
200. Площадь параллелограмма.
201. Площадь треугольника.
202. Площадь трапеции.
Глава XVI. ПОДОБИЕ ГЕОМЕТРИЧЕСКИХ ФИГУР
203. Пропорциональные отрезки.
204. Свойства биссектрис внутреннего и внешнего углов треугольника.
§ 2. Подобное преобразование фигур (гомотетия)
205. Определение гомотетичных фигур.
206. Свойства преобразования подобия.
§ 3. Общее подобное соответствие фигур
207. Подобные фигуры.
208. Периметры и площади подобных треугольников.
209. Применение подобия к решению задач на построение.
Глава XVII. МЕТРИЧЕСКИЕ СООТНОШЕНИЯ В ТРЕУГОЛЬНИКЕ И КРУГЕ
210. Углы с вершиной на окружности.
211. Углы с вершиной внутри и вне круга.
212. Угол, под которым виден данный отрезок.
213. Четырехугольники, вписанные в окружность.
214. Пропорциональные отрезки в круге.
215. Задачи на построение.
§ 2. Метрические соотношения в треугольнике
216. Пропорциональные отрезки в прямоугольном треугольнике. Теорема Пифагора.
218. Теорема синусов. Формула Герона.
217. Квадрат стороны, лежащей против острого или тупого утла и треугольнике. Теорема косинусов.
218. Теорема синусов. Формула Герона.
219. Радиусы вписанной и описанной окружностей.
§ 3. Решение треугольников
220. Таблицы функций.
221. Решение треугольников. Сводка основных формул.
222. Решение прямоугольных треугольников.
223. Решение косоугольных треугольников.
Глава XVIII. ПРАВИЛЬНЫЕ МНОГОУГОЛЬНИКИ. ДЛИНА окружности И ПЛОЩАДЬ КРУГА
224. Выпуклые многоугольники.
225. Правильные многоугольники.
226. Соотношения между стороной, радиусом и апофемой.
227. Периметр и площадь правильного n-угольника.
228. Удвоение числа сторон правильного многоугольника.
§ 2. Длина окружности. Площадь круга и его частей
229. Длина окружности.
230. Площадь круга и его частей.
Глава XIX. ПРЯМЫЕ И ПЛОСКОСТИ В ПРОСТРАНСТВЕ
231. Взаимное расположение двух прямых в пространстве.
232. Взаимное расположение прямой линии и плоскости.
233. Взаимное расположение двух плоскостей.
234. Свойства параллельных прямых и плоскостей.
235. Построения в стереометрии.
§ 2. Перпендикулярность прямых и плоскостей
236. Перпендикуляр к плоскости.
237. Перпендикуляр и наклонные.
238. Угол между прямой и плоскостью.
239. Связь между перпендикулярностью и параллельностью прямых и плоскостей.
240. Общий перпендикуляр двух скрещивающихся прямых.
§ 3. Двугранные и многогранные углы
241. Двугранный угол.
242. Взаимно перпендикулярные плоскости.
243. Трехгранные углы.
244. Многогранные углы.
§ 4. Многогранники
245. Многогранники.
246. Правильные многогранники.
Глава XX. МНОГОГРАННИКИ И КРУГЛЫЕ ТЕЛА
247. Цилиндры и призмы.
248. Параллелепипеды.
249. Объемы призм и цилиндров.
250. Площадь боковой поверхности призмы.
251. Площадь поверхности цилиндра.
§ 2. Пирамида. Конус
252. Свойства пирамиды и конуса.
253. Объем пирамиды и конуса.
254. Площадь боковой поверхности правильной пирамиды и конуса.
255. Усеченный конус и усеченная пирамида.
§ 3. Шаровая поверхность. Шар
256. Шар и шаровая поверхность.
257. Объем шара и его частей.
258. Площадь поверхности шара и ее частей.
259. Понятие телесного угла.
Ответы к упражнениям
Приложения

Мэтуэй | Популярные задачи

92) 9(3x) по отношению к x 92+1
1 Найти производную — d/dx бревно натуральное х
2 Оценить интеграл интеграл натурального логарифма x относительно x
3 Найти производную — d/dx
21 Оценить интеграл интеграл от 0 до 1 кубического корня из 1+7x относительно x
22 Найти производную — d/dx грех(2x)
23 Найти производную — d/dx
41 Оценить интеграл интеграл от cos(2x) относительно x
42 Найти производную — d/dx 1/(корень квадратный из х)
43 Оценка интеграла 9бесконечность
45 Найти производную — d/dx х/2
46 Найти производную — d/dx -cos(x)
47 Найти производную — d/dx грех(3x)
68 Оценить интеграл интеграл от sin(x) по x
69 Найти производную — d/dx угловой синус(х)
70 Оценить предел ограничение, когда x приближается к 0 из (sin(x))/x 92 по отношению к х
85 Найти производную — d/dx лог х
86 Найти производную — d/dx арктан(х)
87 Найти производную — d/dx бревно натуральное 5х92

Мэтуэй | Популярные задачи

92
1 Найти точное значение грех(30)
2 Найти точное значение грех(45)
3 Найти точное значение грех(30 градусов)
4 Найти точное значение грех(60 градусов)
5 Найти точное значение загар (30 градусов)
6 Найти точное значение угловой синус (-1)
7 Найти точное значение грех(пи/6)
8 Найти точное значение cos(pi/4)
9 Найти точное значение грех(45 градусов)
10 Найти точное значение грех(пи/3)
11 Найти точное значение арктан(-1)
12 Найти точное значение cos(45 градусов)
13 Найти точное значение cos(30 градусов)
14 Найти точное значение желтовато-коричневый(60)
15 Найти точное значение csc(45 градусов)
16 Найти точное значение загар (60 градусов)
17 Найти точное значение сек(30 градусов)
18 Найти точное значение cos(60 градусов)
19 Найти точное значение соз(150)
20 Найти точное значение грех(60)
21 Найти точное значение cos(pi/2)
22 Найти точное значение загар (45 градусов)
23 Найти точное значение arctan(- квадратный корень из 3)
24 Найти точное значение csc(60 градусов)
25 Найти точное значение сек(45 градусов)
26 Найти точное значение csc(30 градусов)
27 Найти точное значение грех(0)
28 Найти точное значение грех(120)
29 Найти точное значение соз(90)
30 Преобразовать из радианов в градусы пи/3
31 Найти точное значение желтовато-коричневый(30)
35 Преобразовать из радианов в градусы пи/6
36 Найти точное значение детская кроватка(30 градусов)
37 Найти точное значение арккос(-1)
38 Найти точное значение арктический(0)
39 Найти точное значение детская кроватка(60 градусов)
40 Преобразование градусов в радианы 30
41 Преобразовать из радианов в градусы (2 шт.

Сумма геометрической прогрессии конечной: Сумма членов конечной геометрической прогрессии, первый член которой равен 1, а знаменатель положителен, равна 21/16,…

111. Геометрическая прогрессия

Геометрическая прогрессия – это числовая последовательность, первый член которой отличен от нуля, а каждый член, начиная со второго, равен предыдущему, умноженному на одно и то же отличное от нуля число , где – знаменатель прогрессии: ().

Общий вид геометрической прогрессии:

:: ; ; ; …; ; …

Геометрическая прогрессия является возрастающей при и убывающей при .

Например, :: 2; 6; 18; 54; …; – возрастающая прогрессия; :: 250; 50; 10; …; – убывающая прогрессия.

Если заданы первый член и знаменатель , то -й член геометрической прогрессии определяют по формуле:

.

Сумму первых членов Геометрической прогрессии находят по формуле: .

Свойства геометрической прогрессии.

1. Квадрат каждого среднего члена прогрессии равен произведению равноотстоящих от него членов:

; ().

2. В конечной геометрической прогрессии произведения двух членов, равноотстоящих от ее концов, равны между собой и равны произведению крайних членов:

:: ; ; ; . ..; ; ; …; ; ;

.

Пример 7. Найдите первый и последний члены геометрической прогрессии, которая состоит из четырех членов, если и .

Решение. Подставим исходные данные в формулу:

.

Найдем по формуле и получим: .

Ответ. и .

Пример 8. В геометрической прогрессии (): . Найдите сумму восьми первых членов прогрессии .

Решение. ; ; , тогда запишем исходную систему так: .

Разделим почленно второе уравнение на первое. Получим:

.

Найдем из первого уравнения системы: .

По формуле для суммы найдем: .

Ответ. .

Пример 9. Шесть чисел составляют геометрическую прогрессию. Сумма первых трех чисел равна 168, а сумма последних трех чисел равна 21. Найдите эти числа.

Решение. Из условия задачи составим систему уравнений:

Найдем , для этого разделим первое уравнение на второе: .

Найдем из первого уравнения: .

Ответ. 96; 48; 24; 12; 6; 3.

Пример 10. Найдите сумму , .

Решение. По условию задания можно сделать вывод о том, что: – это геометрическая прогрессия. Найдем первый член прогрессии, знаменатель и общее количество ее членов: ; ; . Тогда .

Ответ. .

Бесконечно убывающая геометрическая прогрессия – это такая бесконечная геометрическая прогрессия (), у которой знаменатель .

Сумму бесконечно убывающей геометрической прогрессии находят по формуле:

.

Пример 11. Запишите периодическую дробь 0,4545…=0,(45) как обыкновенную.

Решение. Запишем периодическую дробь в виде бесконечной суммы обыкновенных дробей: .

Слагаемые представляют собой бесконечно убывающую геометрическую прогрессию со знаменателем и первым членом , а полученная сумма – это сумма этой прогрессии.

Используя формулу суммы бесконечно убывающей геометрической прогрессии, получим: .

Ответ. .

Пример 12. Найдите бесконечно убывающую геометрическую прогрессию, если , а сумма .

Решение. Используя формулу суммы бесконечно убывающей геометрической прогрессии найдем:

.

Ответ. :: ; ; ; …

Пример 13. Найдите бесконечно убывающую геометрическую прогрессию, если ее сумма равна , а сумма ее первых четырех членов равна .

Решение. Из условия задачи запишем систему:

.

Подставим правую часть первого уравнения во второе уравнение:

И .

Тогда найдем два значения :

1) ; 2) .

Ответ. 1) :: ; ; ; ; …; 2) :: ; ; ; ; … .

Пример 14. Сумма бесконечно убывающей геометрической прогрессии , а сумма квадратов всех ее членов . Найти четвертый член прогрессии.

Решение. Найдем знаменатель прогрессии, которая состоит из квадратов членов: ; ; ; …; ; … : .

Тогда составим систему уравнений: .

Возведем первое уравнение в квадрат: .

Разделим второе уравнение системы на первое:

Тогда ; .

Ответ. .

< Предыдущая   Следующая >

Сумма первых N членов конечной геометрической прогрессии Калькулятор

✖Первый член конечной GP — это значение, соответствующее первому члену конечной геометрической прогрессии.ⓘ Первый срок конечного GP [a]

+10%

-10%

✖Общее отношение конечной GP — это отношение любого члена к предыдущему члену геометрической прогрессии, в которой не существует бесконечной суммы членов.ⓘ Общий коэффициент конечного GP [r]

+10%

-10%

✖Номер индекса n-го члена конечной ГП — это значение n для n-го члена или положение n-го члена в конечной геометрической прогрессии.ⓘ Номер индекса n-го члена конечной ГП [n]

+10%

-10%

✖Сумма первых N членов конечной ГП — это сумма членов, начиная с первого по n-й член данной конечной геометрической прогрессии. ⓘ Сумма первых N членов конечной геометрической прогрессии [Sn]

⎘ копия

👎

Формула

сбросить

👍

Сумма первых N членов конечной геометрической прогрессии Решение

ШАГ 0: Сводка предварительного расчета

ШАГ 1. Преобразование входов в базовый блок

Первый срок конечного GP: 3 —> Конверсия не требуется
Общий коэффициент конечного GP: 2 —> Конверсия не требуется
Номер индекса n-го члена конечной ГП: 4 —> Конверсия не требуется

ШАГ 2: Оцените формулу

ШАГ 3: Преобразуйте результат в единицу вывода

45 —> Конверсия не требуется

< 2 Конечная геометрическая прогрессия Калькуляторы

Сумма первых N членов конечной геометрической прогрессии формула

Сумма первых N членов конечной ЗП = (Первый срок конечного GP*((Общий коэффициент конечного GP^Номер индекса n-го члена конечной ГП)-1))/(Общий коэффициент конечного GP-1)
Sn = (a*((r^n)-1))/(r-1)

Что такое геометрическая прогрессия?

В математике геометрическая прогрессия или просто GP, также известная как геометрическая последовательность, представляет собой последовательность чисел, в которой каждый член после первого находится путем умножения предыдущего на фиксированное действительное число, называемое обыкновенным отношением. Например, последовательность 2, 6, 18, 54,… является геометрической прогрессией со знаменателем 3. Если сумма всех членов прогрессии является конечным числом или если существует бесконечная сумма прогрессии, то мы скажем, это бесконечная геометрическая прогрессия или бесконечная GP. А если бесконечной суммы прогрессии не существует, то это конечная геометрическая прогрессия или конечная ВП. Если абсолютное значение общего отношения больше 1, то ЗП будет Конечным ЗП, а если оно меньше 1, то ЗП будет Бесконечным ЗП.

Share

Copied!

геометрия — Визуальная интуиция для суммы КОНЕЧНОГО геометрического ряда

спросил

Изменено 9 месяцев назад

Просмотрено 1к раз

$\begingroup$

Меня интересуют интуитивные визуальные объяснения суммы конечного геометрического ряда.

Я знаю, что есть несколько довольно «интуитивных» объяснений (в том числе и на этом сайте) , но я не видел ни одного визуального интуитивного понимания.

Если кто-нибудь знает о них и поделится ими, я был бы очень признателен!

Спасибо!


Конечный геометрический ряд . Все ответы до сих пор относились к бесконечному случаю.

Спасибо!

  • последовательности и серии
  • геометрия
  • суммирование
  • мягкий вопрос
  • интуиция
$\endgroup$

3

$\begingroup$

Вот еще одно доказательство (скопировано с блестящего.org) бесконечного ряда, но для произвольного $r<1$. Интересно, можно ли его адаптировать для конечного случая, если подумать о такой трапеции вместо треугольника...

$\endgroup$

3

$\begingroup$

Если кто-то выживет, ограничиваясь только натуральными числами, предлагаю иллюстрацию с помощью деревьев.

Каждому из $n$ членов геометрического ряда со знаменателем $r$ соответствует уровень в (совершенном) $r$-арном дереве $T_n$. Сумма $s$ определяется количеством узлов в $T_n$.

Основная идея приходит из сравнения $T_n$ с $T_{n+1}$, т.е. сравнения ряда из $n$ и $n+1$ терминов: 9{n+1}-1}{r-1}$

Итого , рассматривая следующий член ряда, вы увеличиваете свою сумму на известную величину (шаг 2), что соответствует умножению оригинала на известную величину (шаг 3).

$\endgroup$

$\begingroup$

Смотрите эти изображения:

Это графическое объяснение суммы геометрической прогрессии отношения $\frac{1}{2}$.

$\endgroup$

2

$\begingroup$

Нарисуйте отрезок длиной в одну единицу.

Галочка в первой трети слева.

С правой стороны отметьте первую треть слева.

С правой стороны отметьте первую треть слева.

С правой стороны отметьте первую треть слева.

Когда вы закончите, у вас будет бесконечная сумма для $a=\frac13,r=\frac23$. 94$$.

Теперь представьте себе прямоугольник со сторонами 1 и 2 . Теперь возьмите еще два таких прямоугольника и поставьте их на соседние позиции, они образуют большой прямоугольник со сторонами 2 и 2 . Теперь снова возьмите два больших прямоугольника, чтобы повторить процесс до 4 раз. (См. схему). Теперь все, что вам нужно, это найти общую площадь этих прямоугольников. Здесь у двух последовательных прямоугольников одна сторона общая, а площадь в два раза больше.

Теперь давайте сделаем еще один шаг и нарисуем следующий прямоугольник, площадь которого будет в два раза больше предыдущего. Теперь мы хотим проверить, больше или меньше эта площадь, чем S. 95-2=32-2=30}$$

Когда вы думаете об обобщении этого метода, нам нужно на $\color{red}{k}$ больше предыдущих прямоугольников. Или мы должны разделить эту область на один шаг дальше на k частей, и мы будем использовать 1 часть, чтобы снова скрыть, поэтому у нас есть k-1 частей для разделения, и в конечном итоге мы останемся с наименьшим прямоугольником. Значит, площадь равна k-1 раз.

$$ S(k-1)=A_{n+1}-A_1$$

$\endgroup$

$\begingroup$

Метод треугольника/трапеции, вероятно, является наиболее традиционным, однако существует еще один метод, основанный на его упрощении. 9n$$

Я полагаю, что кто-то уже придумал этот точный метод, однако я не смог найти аналогичный метод нигде в Интернете.

$\endgroup$

7.4.1: Суммы конечного геометрического ряда

  1. Последнее обновление
  2. Сохранить как PDF
  • Идентификатор страницы
    14795
  • Нахождение суммы конечного геометрического ряда

    Вы копите деньги на летний лагерь. Вы вносите 100 долларов первого числа каждого месяца на свой сберегательный счет. Счет растет со скоростью 0,5% в месяц. Сколько денег на вашем счету в первый день 9 числа месяца?


    Сумма конечного геометрического ряда

    Мы обсудили, как использовать калькулятор для нахождения суммы любого ряда при условии, что мы знаем n 9{п-1}\).

    Наконец, решим следующую задачу.

    Первого числа каждого года Чарли вносит 1000 долларов на свой инвестиционный счет. Счет растет со скоростью 8% в год. Сколько денег на счету в первый день 11 -го года.

    Сначала рассмотрим, что здесь происходит в первый день каждого года. В первый день первого года вносится 1000 долларов. В первый день второго года вносится 1000 долларов, а ранее внесенные 1000 долларов приносят 8% годовых или увеличиваются в 1,08 раза (108%). В первый день третьего года вносится еще 1000 долларов, депозит предыдущего года приносит 8% годовых, а первоначальный депозит приносит 8% годовых на два года (мы умножаем на 1,08 9). 0200 2 ):

    Сумма за год 1: 1000

    Сумма за год 2: 1000 + 1000(1,08)

    Сумма за год 3: 1000 + 1000(1,08) + 1000(1,08) 9 0200 2

    Сумма Год 4: 1000 + 1000(1,08) + 1000(1,08) 2 + 1000(1,08) 3

    \(\ \четверка\четверка\четверка\четверка\)⋮

    Сумма Год 11: 1 000 + 1000 (1,08) + 1000(1,08) 2 + 1000(1,08) 3 + … + 1000(1,08) 9 + 1000(1,08) 10

    ∗ В этом ряду 11 слагаемых, потому что в первый день 11 9{11}\right)}{1-1.08}=16645,48746 \приблизительно \$ 16 645,49\)


    Примеры

    Пример 1

    9 месяц.

    Решение

    В этой серии 9 терминов, потому что в первый день 9 -го месяца вы вносите свой последний депозит, а первоначальный депозит приносит проценты в течение 8 месяцев.

    Эта серия геометрическая. Первый член равен 100, обыкновенное отношение равно 1,005 и n = 9.{n-1}\)

    Пример 4

    Первого числа каждого месяца Сэм вносит 50 долларов на счет, который ежемесячно приносит 0,5% годовых. {48},\\
    \quad \uparrow \quad\quad\quad\quad\quad\quad\quad\quad\quad\quad\quad\quad\quad\quad\quad\quad\quad\quad\quad\quad\quad\quad\ quad\quad\quad\quad\quad\uparrow\\
    \text {последний депозит} \quad\quad\quad\quad\quad\quad\quad\quad\quad\quad\quad\quad\quad\quad\quad \quad\quad\quad\quad \text { первый депозит }
    \end{aligned}\)

    Обратите внимание, что на первый депозит начисляются проценты в течение 48 месяцев, а на последний депозит проценты не начисляются. Теперь мы можем найти сумму, используя \(\ a_{1}=50\), \(\ r=1,005\) и \(\ n=49{7} a_{n}=-\frac{63}{2}\) и \(\ r=-\frac{1}{2}\)

    Решите следующие текстовые задачи, используя формулу суммы геометрического ряда.

    1. Бабушка и дедушка Сапны вносят 1200 долларов на сберегательный счет колледжа в день ее -го дня рождения 5  года. Они продолжают вносить этот депозит на день рождения каждый год, пока не сделают последний депозит в день ее рождения 18   года. Если счет приносит 5% годовых, сколько остается после окончательного депозита?
    2. Джереми хочет накопить 10 000 долларов за пять лет. Если он вносит ежегодные депозиты первого числа каждого года и на счете зарабатывается 4,5% годовых, сколько он должен вносить каждый год, чтобы иметь 10 000 долларов на счете после окончательного депозита первого из 6 год. Округлите ответ до ближайших 100 долларов.

    Ответы на проблемы с обзором

    Чтобы просмотреть ответы на обзор, откройте этот PDF-файл и найдите раздел 11.10.


    Словарь

    Срок Определение
    индукция Индукция — это метод математического доказательства, обычно используемый для установления того, что данное утверждение верно для всех положительных целых чисел.
    серия Серия — это сумма членов последовательности.

    Модуль числа с x: Уравнения с модулями. Модули

    1.2 Решение линейных уравнений с модулем

    Для начала, стоит вспомнить, что такое модуль числа. Итак, абсолютной величиной или модулем числа называется само число х, если х положителен, число (-х), если х отрицателен, или нуль, если х=0. Значение модуля может быть только положительным.

    Чтобы понять решение параметрических уравнений, содержащих знак модуля, лучше всего продемонстрировать решение наглядно, т.е. привести примеры:

    Пример 1. Решить уравнение |x-2|=b.

    Так как, по определению модуля, |x-2| , то при b<0 данное уравнение решений не имеет. Если b=0, то уравнение имеет решение х=2.

    Если b>0, то решениями уравнения являются числа x=2+b и x=2-b.

    Ответ: при b<0 решений нет, при b=0 х=2, при b>0 х=2+b и x=2-b.

    Пример 2. Решить уравнение |x-a|=|x-4|. Удобнее всего данное уравнение решить методом интервалов, для двух случаев:

    a ;

    4 .

    1. Первый интервал:

        ;

    Второй интервал:

        , т. е. если а<4, то  .

    Третий интервал:

     а=4, т.е. если а=4, то  .

    2. Первый интервал:

     а=4,  .

    В торой интервал:

         a>4,т.е. если 4<а, то 

    Третий интервал:

       

    Ответ: при а=4 х-любое;, при а<4  .

    Пример 3. Для каждого значения параметра а найти все значения х, удовлетворяющие уравнению |x+3|– a| x – 1| =4.

    Рассмотрим 3 промежутка: 1)  , 2)  , 3)   и решим исходное уравнение на каждом промежутке.

    1.  ,  .

    При а=1 уравнение не имеет решений, но при а 1 уравнение имеет корень  . Теперь надо выяснить, при каких а х попадает на промежуток x< – 3, т.е.  ,  ,  ,  . Следовательно, исходное уравнение на x< – 3 имеет один корень   при  , а на остальных а корней не имеет.

    2.  .  .

    При а= – 1 решением уравнения является любое х; но мы решаем на промежутке  . Если а 1, то уравнение имеет один корень х=1.

    3.  .  .

    При а=1 решением является любое число, но мы решаем на  . Если а 1, то х=1.

    Ответ: при    ; при а= – 1   и при а 1 х=1; при а=1   и при а 1 х=1.

    1.3 Решение квадратных уравнений с параметром

    Для начала напомню, что квадратное уравнение – это уравнение вида  , где а, b и с – числа, причем, а 0.

    Условия параметрических квадратных уравнений могут быть различны, но для решений всех их нужно применять свойства обыкновенного квадратного уравнения  :

    а) Если D>0, а>0, то уравнение имеет два действительных различных корня, знаки которых при с>0 одинаковые и противоположны по знаку коэффициента b, а при с<0, причем по абсолютной величине больше тот, знак которого противоположен коэффициенту b.

    б) Если D=0, а>0, то уравнение имеет два действительных и равных между собой корня, знак которых противоположен знаку коэффициента b.

    в) Если D<0, а>0, то уравнение не имеет действительных корней.

    Аналогично можно представить свойства корней при а<0. Кроме того, в квадратных уравнениях справедливы следующие утверждения:

    Если поменять местами коэффициенты а и

    с, то корни полученного квадратного уравнения будут обратны корням данного.

    Если поменять знак коэффициента b, корни полученного квадратного уравнения будут противоположны корням данного.

    Если коэффициенты а и с разных знаков, то уравнение имеет действительные корни.

    Пример1. Найти все значения параметра а, для которых квадратное уравнение  : а) имеет два различных корня; б) не имеет корней; в) имеет два равных корня.

    Данное уравнение по условию является квадратным, поэтому а -1. Рассмотрим дискриминант данного уравнения:

    При а>-1 уравнение имеет два различных корня, т.к. D>0, при a<-1 уравнение корней не имеет, т.к. D<0, а двух одинаковых корней это уравнение иметь не может, т.к. D=0 при а=-1, а это противоречит условию задачи.

    Пример2. Решить уравнение 

    При а=0 уравнение является линейным 2х+1=0, которое имеет единственное решение х=-0.5. А при а 0, уравнение является квадратным и его дискриминант D=4-4a.

    При а>1 D<0 поэтому уравнение корней не имеет. При а=1 D=0, поэтому уравнение имеет два совпадающих корня  =-1.

    При a<1, но а 0, D>0 и данное уравнение имеет два различных корня

     ;  .

    Ответ:   и   при a<1, но а 0; х=-0.5 при а=0;  =-1 при а=1.

    Пример3. Корни уравнения   таковы, что  . Найдите а.

    По теореме Виета   и  . Возведём обе части первого равенства в квадрат:  . Учитывая, что , а  , получаем:   или  ,    . Проверка показывает, что все значения   удовлетворяют условию.

    Ответ: 

    Модуль

    Модулем положительного числа называют само это число; модулем отрицательного числа называют число, ему противоположное; модуль нуля равен нулю.

    \(|a|=\begin{cases} a, \;\; если \;  a>0 \\ 0, \; если\;\; a=0\\ -a,\; если \;\;  a<0  \end{cases}\)

    Второе название модуля – «абсолютное значение действительного числа».

    Фактически модуль делает всё, что находится внутри него положительным. Поэтому чтобы правильно его раскрыть, необходимо сначала выяснить знак выражения внутри него:

    — если подмодульное выражение положительно, модуль просто убирается. 4+1\)

    Пример. Вычислить значение выражения \(|7-x|-|x+3|\), при \(x>12\).

    Решение: При любом \(x\) большем \(12\), первое подмодульное выражение будет отрицательно, а второе – положительно. Соответственно, первый модуль будет раскрываться с минусом, а второй – с плюсом (значит перед ним останется минус, который стоял перед ним до раскрытия):

    \(|7-x|-|x+3|=-(7-x)-(x+3)=-7+x-x-3=-10\)

    Ответ: \(-10\)



    Геометрическое определение модуля

    \(|a|\) — это расстояние от \(0\) до числа \(a\) на числовой оси

    Пример. Чему равен \(|5|\)  и \(|-5|\)?

    Представим числовую ось и отметим на ней точки \(5\) и \(-5\). Какое будет расстояние от нуля до этих точек? Очевидно \(5\).


    Значит ответ: \(|5|=5\),   \(|-5|=5\).

    Так как модуль это расстояние, а расстояние не может выражаться отрицательным числом, то он всегда положителен.

    Понимать легче второе определение, но практике удобнее использовать первое.

    Решение простейших уравнений с модулем

    Уравнения вида \(|f|=g\) решается с помощью перехода к совокупности   \( \left[ \begin{gathered}f= g\\  f=-g\end{gathered}\right.\) , при условии, что \(g≥0\).

    Сначала об условии \(g≥0\). Откуда оно берется? Из определения модуля, ведь модуль всегда неотрицателен (то есть, положителен или равен нулю). Поэтому условие \(g≥0\) обязательно. Иначе уравнение не будет иметь решения.

    Теперь о совокупности. Почему уравнение распадается на два? Давайте, к примеру, рассмотрим уравнение \(|x|=3\). Какое число под модулем будет равно \(3\)? Конечно \(3\) и \(-3\), потому что \(|3|=3\), \(|-3|=3\). Корни уравнения \(|x|=3\): \(3\) и \(-3\). Логично? Логично! В общем виде получается, что подмодульное выражение \(f\) должно быть равно \(g\) и \(-g\). Иначе равенство не получится.

    Пример.   Решить уравнение:

    \(|x-1|=3x\)

    Найдем ограничения уравнения. Запишем его немного правее от основного решения

                                   \(3x≥0\)
                                   \(x≥0\)

     

    Когда ограничение записано —  можно со спокойной душой решать уравнение. Избавимся от модуля и перейдем к совокупности уравнений

    \( \left[ \begin{gathered}x-1=3x\\ x-1=-3x\end{gathered}\right.\)

     

    Перед нами 2 линейных уравнения. Решаем их с помощью известного заклинания: «иксы влево, числа вправо»

    \( \left[ \begin{gathered}x-3x=1\\ x+3x=1\end{gathered}\right. \)

     

    Приведем подобные слагаемые

    \( \left[ \begin{gathered}-2x=1\\ 4x=1\end{gathered}\right.\)


    Поделим первое уравнение на \(-2\), второе на \(4\).

    \( \left[ \begin{gathered} x=-\frac{1}{2}\\ x=\frac{1}{4}\end{gathered}\right.\)


    Корень \(-\)\(\frac{1}{2}\) – не подходит, т.к. \(x≥0\). Остается корень \(\frac{1}{4}\), его и запишем в ответ

    Ответ:  \(\frac{1}{4}\)

    Решение простейших неравенств с модулем

    Неравенство вида \(|f|< c\) решается с помощью перехода к двойному неравенству   \( -c< f< c\) , при условии, что \(c>0\).

    Начнем опять с условия. Почему \(c>0\)? Потому что, иначе неравенство не будет иметь решения. Здесь все также как в уравнениях. В самом деле, когда, например, модуль икса меньше \(-7\)? Никогда!

    Теперь разберем неравенство \(|x|<3\). Какие иксы нам подойдут? Все от \(-3\) до \(3\). Иначе говоря, икс должен лежать между \(-3\) и \(3\). Это утверждение можно записать вот так \(-3< x <3\) либо системой \(\begin{cases}x<3\\x > -3\end{cases}\). В любом случае ответ будет \(xϵ (-3;3)\).

    Неравенство вида \(|f|>c\) решается с помощью перехода к совокупности неравенств \( \left[ \begin{gathered} f>c\\ f< -c\end{gathered}\right.\), при условии, что \(c≥0\).

    А здесь почему \(c≥0\)? Потому что иначе решать нечего: если \(c\) отрицательно, то модуль абсолютно любого икса нам подойдет. И значит ответ, икс – любое число.

    Теперь о переходе. Рассмотрим неравенство \(|x|>3\). Какие иксы нам подойдут? Все, модуль которых больше трех, то есть от минус бесконечности до \(-3\) и от \(3\) до плюс бесконечности. Записывая системой получим \(\begin{cases}x>3\\x < -3\end{cases}\). Ответ будет \(x ϵ (-∞;-3)⋃(3;∞)\).

    \(|3x-7|≤8\)

    \(|3x-11|≥11\)

    \(-8≤3x-7≤8\) \(|+7\)

    \( \left[ \begin{gathered}3x-11≥11\\ 3x-11≤-11\end{gathered}\right.\)

    \(-1≤3x≤15\)

     

    \( \left[ \begin{gathered}3x≥22\\ 3x≤0\end{gathered}\right.\)

    \(-\frac{1}{3}≤x≤5\)

     

    \( \left[ \begin{gathered}x≥\frac{22}{3}\\ x≤0\end{gathered}\right. \)

    Ответ: \([ -\frac{1}{3};5]\)

     

    Ответ: \( (-\infty;0]\cup [ \frac{22}{3};\infty)\)

    Смотрите также:
    Свойства модуля

    Теория чисел. Модульная арифметика

    ◀ Теория чисел. Алгоритм Евклида ▶

    Содержание

    Пусть \(n\) — положительное целое число. Обозначим множество \([0..n-1]\) через \(\mathbb{Z}_n\).

    Мы считаем два целых числа \(x, y\) одинаковыми, если \(x\) и \(y\) отличаются на a кратно \(n\), и мы записываем это как \(x = y \pmod{n}\) и говорим, что \(x\) и \(y\) конгруэнтны по модулю \(n\). Мы можем опустить \(\pmod{n}\), когда ясно из контекста. Каждое целое число \(x\) конгруэнтно некоторому \(y\) в \(\mathbb{Z}_n\). Когда мы добавляем или вычесть кратные \(n\) из целого числа \(x\), чтобы получить некоторое \(у\в\mathbb{Z}_n\), мы говорим уменьшает \(x\) по модулю \(n\), а \(y\) является остатком .

    Мы могли бы выбрать разные наборы для \(\mathbb{Z}_n\), например мы могли бы добавить \(n\) к каждому элементу, но по умолчанию будет \([0..n-1]\). Элементы в этом конкретном представлении \(\mathbb{Z}_n\) называются наименьшими остатками .

    Пример : \(38 = 3 \pmod{5}\), так как \(38 = 7\умножить на 5 + 3\). \(-3 = 11 \pmod{14}\), так как \(-3 = (-1)\умножить на 14 + 11\).

    Каков наиболее естественный способ выполнения арифметических действий в \(\mathbb{Z}_n\)? Учитывая два элемента \(x, y \in \mathbb{Z}_n\), мы можем складывать, вычитать или умножать их как целые числа, а затем результат будет соответствовать одному из элементов в \(\mathbb{Z}_n\).

    Пример : \(6 + 7 = 1 \pmod{12}\), \(3 \times 20 = 10 \pmod{50}\), \(12 — 14 = 16 \pmod{18}\).

    Эти операции ведут себя так же, как их обычные аналоги. Однако понятия размера нет. Говоря \(0

    Division

    Division явно отсутствует в приведенном выше обсуждении. Если \(y\) делит \(x\) как целые числа, то можно предположить, что мы могли бы использовать обычное определение. Посмотрим, к чему это приведет: у нас есть \(10 = 4 \pmod{6}\). Разделение обоих сторон на \(2\) дает неверное уравнение \(5 = 2 \pmod{6}\).

    Таким образом, мы должны изменить значение деления. Интуитивно деление должно «отменять умножение», т. е. разделить \(x\) на \(y\) означает найти число \(z\) такое, что \(y\) умножить на \(z\) равно \(x\). Проблема выше в том, что существуют разные кандидаты на \(z\): в \(\mathbb{Z}_6\) и 5, и 2 дают 4 при умножении на 2.

    Какой ответ мы должны выбрать для «\(4 / 2\)», \( 5\) или \(2\)? Мы могли бы ввести некоторые произвольные соглашения, такие как выбор наименьшего ответ при рассмотрении наименьшего остатка как целого числа, но тогда деление будет вести себя странно.

    Вместо этого мы требуем уникальности, то есть \(x\), деленное на \(y\) по модулю \(n\), составляет всего определяется, когда существует уникальный \(z \in \mathbb{Z}_n\) такой, что \(x = y z\).

    Мы можем получить условие на \(у\) следующим образом. Предположим, \(z_1 y = z_2 y \pmod {n}\). Тогда по определению это означает для некоторого \(k\) имеем \(y(z_1 — z_2) = k n\). Пусть \(d\) будет наибольшим общий делитель \(n\) и \(y\). Тогда \(n/d\) делит \(z_1 — z_2\) поскольку он не может делить \(y\), мы имеем

    \[ z_1 y = z_2 y \pmod {n} \]

    тогда и только тогда, когда

    \[ z_1 = z_2 \pmod {n/d} . \]

    Таким образом, уникальный \(z\) существует по модулю \(n\), только если наибольшее общее делитель \(y\) и \(n\) равен 1.

    Обратные

    Мы увидим, что существует единственное \(z\) тогда и только тогда, когда можно найти \(w \in \mathbb{Z}_n\) такое, что \(y w = 1 \pmod {n}\). Если такое \(w\) существует, то оно должно быть уникальным: предположим, что \(y w’\) также равно 1. Тогда, умножая обе части \(y w = y w’\) через \(w\) дает \(w y w = w y w’\), откуда следует \(w = w’\) так как \(wy = 1\). Когда он существует, мы называем это уникальное \(w\) 9{-1}\) существует, и если да, то как его найти? Поскольку в \(\mathbb{Z}_n\) всего \(n\) элементов, мы можем умножить каждый элемент по очереди на \(y\) и посмотреть, получим ли мы 1. {-1}\), если существует инверсия \(y\), иначе ответ не определено. 9{-1}) = 2 \pmod{6}\).

    ◀ Теория чиселАлгоритм Евклида ▶

    Содержание


    Бен Линн [email protected] 💡

    Математическая задача: Модуль — вопрос № 4325, алгебра, уравнение

    Найдите x в уравнении по модулю:

    47x = 4 (mod 9)

    Подсказка — прочитайте, какое число 47x разделить на 9 (по модулю 9) дает остаток 4.

    Правильный ответ:

    x =  2

    Пошаговое объяснение:

    47x = 9к + 4; k,x−целое число k=(47x−4)/9k1=(47⋅1−4)/9=943=497=4,7778k2=(47⋅2−4)/9=10k3​ =(47⋅ 3−4)/9=9137​=1592​≐15,2222 k4​=(47⋅ 4−4)/9=9184​=2094​≐20,4444 …. x=2


    Нашли ошибку или неточность? Не стесняйтесь

    пишите нам

    . Спасибо!

    Советы по связанным онлайн-калькуляторам

    Вы решаете задачи Диофанта и ищете калькулятор целочисленных уравнений Диофанта?
    У вас есть линейное уравнение или система уравнений и вы ищете ее решение? Или у вас есть квадратное уравнение?

    Чтобы решить эту математическую задачу, вам необходимо знать следующие знания:

    • алгебра
    • уравнение
    • целочисленное уравнение
    • делимость
    • 90 108 основные функции
    • по модулю
    • числа
    • целые числа
    • натуральные числа
    Уровень задачи:
    • практика для 14-летних
    • старшая школа
    • Большое число
      Какой остаток при делении 10 на 9 до 47 — 111?
    • Напоминание и частное
      Даны числа A = 135, B = 315. Найдите наименьшее натуральное число R, большее единицы, так, чтобы отношения R:A, R:B были с остатком 1.
    • Делимость
      Является ли число 761082 точно делится на 9? (результат — целое число и/или остаток равен нулю)
    • Остаток 33031
      Найдите число, которое при делении на 28 дает соотношение 606 и остаток 23.
    • Остаток 34441
      Найдите остаток после деления суммы на 1! +2! +3! +. … . +300! число 13.
    • Неизвестное целое число
      Найдите наименьшее целое число: При делении на 2 остаток равен 1. При делении на 3. Остаток равен 2. При делении на 4. Остаток равен 3. При делении на восемь остаток равен 7. При делении на 9 остаток равен 8.
    • Трехзначное 8002
      Найдите наибольшее трехзначное число, которое дает остаток 1 при делении на три, дает остаток 2 при делении на четыре, дает остаток 3 при делении на пять, и дает остаток 4 при делении на шесть.
    • Делимый 9331
      Число X — наименьшее натуральное число, половина которого делится на три, треть — на четыре, четверть — на одиннадцать, а его половина дает остаток 5 при делении на семь. Найдите это число.
    • Несколько систем счисления
      Найдите значение x, для которого 312 четыре +52 x = 96 десять . Подсказка: четыре, х и десять — это основание заданного числа.
    • Остаток 5594
      Какое число мы разделили на 55, если отношение равно 90,16, а остаток 0,04?
    • Отец 2
      Отец отдал 1/3 своей земли дочери, а оставшуюся часть сыну. Сын жертвует 3/4 своей земли под детскую площадку. На половине оставшейся площади он построил дом, а другую половину продал за 1 миллион рупий. 1) давать сыновьям часть в виде дроби
    • Остатки
      Дан набор чисел { 170; 244; 299; 333; 351; 391; 423; 644}. Разделите эти числа на число 66 и определите множество остатков. В результате запишите сумму этих остатков.
    • Дата игры
      Пусть сейчас вторник. Какой день наступает через 229 дней? Запишите результат в виде числа: 1=понедельник, 2=вторник, 3=среда, 4=четверг, 5=пятница, 6=суббота, 7=воскресенье.
    • Рынок открыт
      Определенный рынок открывается для продаж каждый 7-й день недели.

    Координаты середины отрезка формула в пространстве: Середина отрезка. Координаты середины отрезка

    Математика: Справ. материалы

    Математика: Справ. материалы
      

    Гусев В. А., Мордкович А. Г. Математика: Справ. материалы: Кн. для учащихся.— М.: Просвещение, 1988.— 416 с.

    В книге дано краткое изложение основных разделов школьных курсов алгебры и начал анализа, геометрии. Книга окажет помощь в систематизации и обобщении знаний по математике.



    Оглавление

    СЛОВО К УЧАЩИМСЯ
    ГЛАВА I. ЧИСЛА
    § 1. Натуральные числа
    2. Арифметические действия над натуральными числами.
    3. Деление с остатком.
    4. Признаки делимости.
    5. Разложение натурального числа на простые множители.
    6. Наибольший общий делитель нескольких натуральных чисел.
    7. Наименьшее общее кратное нескольких натуральных чисел.
    8. Употребление букв в алгебре. Переменные.
    § 2. Рациональные числа
    10. Равенство дробей. Основное свойство дроби. Сокращение дробей.
    11. Приведение дробей к общему знаменателю.
    12. Арифметические действия над обыкновенными дробями.
    13. Десятичные дроби.
    14. Арифметические действия над десятичными дробями.
    15. Проценты.
    16. Обращение обыкновенной дроби в бесконечную десятичную периодическую дробь.
    17. Обращение бесконечной десятичной периодической дроби в обыкновенную дробь.
    18. Координатная прямая.
    19. Множество рациональных чисел.
    § 3. Действительные числа
    21. Действительные числа. Числовая прямая.
    22 Обозначения некоторых числовых множеств.
    23. Сравнение действительных чисел.
    25. Числовые промежутки.
    26. Модуль действительного числа.
    27. Формула расстояния между двумя точками координатной прямой.
    28. Правила действий над действительными числами.
    29. Свойства арифметических действий над действительными числами.
    30. Пропорции.
    31. Целая часть числа. Дробная часть числа.
    32. Степень с натуральным показателем.
    33. Степень с нулевым показателем. Степень с отрицательным целым показателем.
    34. Стандартный вид положительного действительного числа.
    35. Определение арифметического корня.
    36. Корень нечетной степени из отрицательного числа.
    37. Степень с дробным показателем.
    38. Свойства степеней с рациональными показателями.
    39. Приближенные значения чисел. Абсолютная и относительная погрешности.
    40. Десятичные приближения действительного числа по недостатку и по избытку.
    41. Правило извлечения квадратного корня из натурального числа.
    42. Понятие о степени с иррациональным показателем.
    43. Свойства степеней с действительными показателями.
    § 4. Комплексные числа
    45. Арифметические операции над комплексными числами.
    46. Алгебраическая форма комплексного числа.
    47. Отыскание комплексных корней уравнений.
    ГЛАВА II. АЛГЕБРАИЧЕСКИЕ ВЫРАЖЕНИЯ
    49. 3.
    112. Построение графика функции y = f(x-m)+n
    113. График квадратичной функции.
    114. Способы построения графика квадратичной функции
    115. Построение графика функции y = f(kx).
    116. Сжатие и растяжение графиков тригонометрических функций.
    117. График гармонического колебания
    ГЛАВА IV. ТРАНСЦЕНДЕНТНЫЕ ВЫРАЖЕНИЯ
    § 12. Преобразование выражений, содержащих переменную под знаком логарифма
    119. Определение логарифма положительного числа по данному основанию.
    120. Свойства логарифмов.
    121. Переход к новому основанию логарифма.
    122. Логарифмирование и потенцирование.
    123. Десятичный логарифм. Характеристика и мантисса десятичного логарифма.
    § 13. Формулы тригонометрии и их использование для преобразования тригонометрических выражений
    125. Формулы сложения и вычитания аргументов.
    126. Формулы приведения.
    127. Соотношения между тригонометрическими функциями одного и того же аргумента.
    128. Формулы двойного угла.
    129. Формулы понижения степени.
    130. Преобразование суммы тригонометрических функций в произведение.
    131. Преобразование произведения тригонометрических функций в сумму.
    132. Преобразование выражения a cos t + b sin t к виду A sin (t + a).
    133. Примеры преобразований выражений, содержащих обратные тригонометрические функции.
    ГЛАВА V. УРАВНЕНИЯ И СИСТЕМЫ УРАВНЕНИЙ
    § 14. Уравнения с одной переменной
    135. Равносильность уравнений.
    136. Линейные уравнения.
    137. Квадратные уравнения.
    138. Неполные квадратные уравнения.
    139. Теорема Виета.
    140. Системы и совокупности уравнений.
    141. Уравнения, содержащие переменную под знаком модуля.
    142. Понятие следствия уравнения. Посторонние корни.
    143. Уравнения с переменной в знаменателе.
    144. Область определения уравнения.
    145. Рациональные уравнения.
    146. Решение уравнения p(x) = 0 методом разложения его левой части на множители.
    147. Решение уравнений методом введения новой переменной.
    148. Биквадратные уравнения.
    149. Решение задач с помощью составления уравнений.
    150. Иррациональные уравнения.
    151. Показательные уравнения.
    152. Логарифмические уравнения.
    153. Примеры решения показательно-логарифмических уравнений.
    154. Простейшие тригонометрические уравнения.
    155. Методы решения тригонометрических уравнений.
    156. Универсальная подстановка (для тригонометрических уравнений).
    157. Метод введения вспомогательного аргумента (для тригонометрических уравнений).
    158. Графическое решение уравнений.
    159. Уравнения с параметром.
    § 15. Уравнения с двумя переменными
    161. График уравнения с двумя переменными.
    162. Линейное уравнение с двумя переменными и его график.
    § 16. Системы уравнений
    164. Решение систем двух уравнений с двумя переменными методом подстановки.
    165. Решение систем двух уравнений с двумя переменными методом сложения.
    167. Графическое решение систем двух уравнений с двумя переменными.
    168. Исследование системы двух линейных уравнений с двумя переменными.
    169. Решение систем двух уравнений с двумя переменными методами умножения и деления.
    170. Системы показательных и логарифмических уравнений.
    171. Системы тригонометрических уравнений с двумя переменными.
    172. Системы трех уравнений с тремя переменными.
    173. Решение задач с помощью составления систем уравнений.
    Глава VI. НЕРАВЕНСТВА
    § 17. Решение неравенств с переменной
    175. Графическое решение неравенств с одной переменной.
    176. Линейные неравенства с одной переменной.
    177. Системы неравенств с одной переменной.
    178. Совокупность неравенств с одной переменной.
    179. Дробно-линейные неравенства.
    180. Неравенства второй степени.
    181. Графическое решение неравенств второй степени.
    182. Неравенства с модулями.
    183. Решение рациональных неравенств методом промежутков.
    184. Показательные неравенства.
    185. Логарифмические неравенства.
    186. Иррациональные неравенства.
    187. Решение тригонометрических неравенств.
    188. Неравенства и системы неравенств с двумя переменными.
    § 18. Доказательство неравенств
    190. Синтетический метод доказательства неравенств.
    191. Доказательство неравенств методом от противного.
    192. Использование неравенств при решении уравнений.
    ГЛАВА VII. ЭЛЕМЕНТЫ МАТЕМАТИЧЕСКОГО АНАЛИЗА
    § 19. Числовые последовательности
    194. Способы задания последовательности.
    195. Возрастание и убывание последовательности.
    196. Определение арифметической прогрессии.
    197. Свойства арифметической прогрессии
    198. Определение геометрической прогрессии.
    199. Свойства геометрической прогрессии.
    200. Понятие о пределе последовательности.
    201. Вычисление пределов последовательностей.
    202. Сумма бесконечной геометрической прогрессии при |q| § 20. Предел функции
    204. Вычисление пределов функции при х->оо.
    205. Предел функции в точке. Непрерывные функции.
    206. Вертикальная асимптота.
    207. Вычисление пределов функций в точке.
    § 21. Производная и ее применения
    209. Определение производной.
    210. Формулы дифференцирования. Таблица производных.
    211. Дифференцирование суммы, произведения, частного.
    212. Сложная функция и ее дифференцирование.
    213. Физический смысл производной.
    214. Вторая производная и ее физический смысл.
    215. Касательная к графику функции.
    216. Применение производной к исследованию функций на монотонность.
    217. Применение производной к исследованию функций на экстремум.
    218. Отыскание наибольшего и наименьшего значений непрерывной функции на отрезке.
    219. Отыскание наибольшего или наименьшего значения непрерывной функции на незамкнутом промежутке.
    220. Задачи на отыскание наибольших или наименьших значений величин.
    221. Применение производной для доказательства тождеств.
    222. Применение производной для доказательства неравенств.
    223. Общая схема построения графика функции.
    § 22. Первообразная и интеграл
    225. Таблица первообразных.
    226. Правила вычисления первообразных.
    227. Интеграл.
    228. Связь между интегралов и первообразной (формула Ньютона—Лейбница).
    229. Правила вычисления интегралов.
    230. Использование интеграла для вычисления площадей плоских фигур.
    ГЕОМЕТРИЯ. ГЛАВА I. ГЕОМЕТРИЧЕСКИЕ ФИГУРЫ НА ПЛОСКОСТИ
    2. Точка. Прямая.
    3. Определения. Аксиомы. Теоремы.
    § 2. Основные свойства простейших геометрических фигур
    5. Луч.
    6. Окружность. Круг.
    7. Полуплоскость.
    8. Угол. Градусная мера угла.
    9. Смежные и вертикальные углы.
    10. Центральные и вписанные углы.
    11. Параллельные прямые.
    12. Признаки параллельности прямых.
    13. Перпендикулярные прямые.
    14. Касательная к окружности.
    15. Треугольники.
    16. Равенство треугольников.
    17. Равнобедренный треугольник.
    18. Сумма углов треугольника.
    19. Прямоугольный треугольник. Теорема Пифагора.
    20. Окружности, вписанные в треугольник и описанные около треугольника.
    § 3. Геометрические построения на плоскости
    22. Простейшие задачи на построение.
    23. Геометрическое место точек на плоскости.
    § 4. Четырехугольники
    25. Параллелограмм.
    26. Прямоугольник. Ромб. Квадрат.
    27. Трапеция.
    § 5. Многоугольники
    29. Выпуклые многоугольники.
    30. Правильные многоугольники.
    31. Длина окружности.
    § 6. Решение треугольников
    33. Соотношения между сторонами и углами в прямоугольном треугольнике.
    34. Теорема косинусов. Теорема синусов.
    35. Решение треугольников.
    § 7. Площади плоских фигур
    37. Площади многоугольников.
    38. Площади подобных фигур.
    39. Площадь круга.
    ГЛАВА II. Прямые и плоскости в пространстве
    § 9. Параллельность прямых и плоскостей
    42. Параллельность прямой и плоскости.
    43. Параллельные плоскости.
    § 10. Перпендикулярность прямых и плоскостей
    45. Перпендикуляр и наклонная к плоскости.
    46. Перпендикулярность плоскостей.
    ГЛАВА III. ТЕЛА В ПРОСТРАНСТВЕ
    § 11. Многогранники
    48. Многогранные углы. Многогранники.
    49. Призма. Параллелепипед. Куб.
    50. Пираприда.
    51. Правильные многогранники.
    § 12. Тела вращения
    53. Конус.
    54. Шар.
    § 13. Изображение пространственных фигур на плоскости
    56. Ортогональное проектирование.
    57. Геометрическое место точек в пространстве.
    § 14. Объемы тел
    59. Объем параллелепипеда, призмы и пирамиды.
    60. Объем цилиндра и конуса.
    61. Общая формула объемов тел вращения.
    § 15. Площади поверхностей тел
    63. Понятие площади поверхности.
    64. Площади поверхностей тел вращения.
    ГЛАВА IV. ДЕКАРТОВЫ КООРДИНАТЫ
    § 16. Координаты на плоскости и в пространстве
    66. Координаты середины отрезка.
    § 17. Уравнения фигур на плоскости
    68. Пересечение двух окружностей.
    69. Уравнение прямой.
    70. Пересечение прямой и окружности.
    § 18. Уравнения фигур в пространстве
    72. Уравнение сферы.
    73. Взаимное расположение сферы и плоскости.
    74. Пересечение двух сфер.
    ГЛАВА V. РЕОБРАЗОВАНИЯ ФИГУР
    76. Понятие движения.
    § 20. Подобие фигур
    78. Подобные фигуры.
    ГЛАВА VI. ВЕКТОРЫ
    80. Понятие вектора.
    81. Координаты вектора.
    § 22. Операции над векторами
    83. Умножение вектора на число. Коллинеарные векторы.
    84. Скалярное произведение векторов.
    ПРИЛОЖЕНИЯ
    ГЕОМЕТРИЯ

    Расстояние между двумя точками. Середина отрезка. Координаты середины отрезка. Тема 4

    1. Тема 1-11. Расстояние между двумя точками. Середина отрезка. Координаты середины отрезка. Уравнение прямой на плоскости.

    Раздел III. Аналитическая геометрия
    Тема 1-11.
    Расстояние между двумя точками. Середина
    отрезка. Координаты середины отрезка.
    Уравнение прямой на плоскости. Уравнение
    прямой в пространстве. Уравнение
    плоскости. Расстояние от точки до плоскости.
    Расстояние между плоскостями. Расстояние
    от точки до прямой на плоскости. Расстояние
    от точки до прямой в пространстве. Угол
    между плоскостями. Угол между прямой и
    плоскостью

    2. Расстояние между двумя точками — это длина отрезка, что соединяет эти точки.

    Расстояние между двумя точками — это
    длина отрезка, что соединяет эти точки.
    • Формула вычисления расстояния между двумя
    точками A(xa, ya) и B(xb, yb) на плоскости:
    AB = √(xb — xa)2 + (yb — ya)2
    • Формула вычисления расстояния между двумя
    точками A(xa, ya, za) и B(xb, yb, zb) в пространстве:
    AB = √(xb — xa)2 + (yb — ya)2 + (zb — za)2

    3. Середина отрезка — это точка, которая лежит на отрезке и находится на равном расстоянии от конечных точек.

    Середина отрезка — это точка, которая
    лежит на отрезке и находится на равном
    расстоянии от конечных точек.
    • Формула вычисления координат середины
    отрезка с концами A(xa, ya) и B(xb, yb) на
    плоскости:
    x a + xb
    ya + yb
    xc =
    yc =
    2
    2
    • Формула вычисления координат середины
    отрезка с концами A(xa, ya, za) и B(xb, yb, zb) в
    пространстве:
    xa + xb
    ya + yb
    za + zb
    xc =
    yc =
    zc =
    2
    2
    2

    4.

    Прямая (прямая линия) — это бесконечная линия, по которой проходит кратчайший путь между любыми двумя её точками.Прямая (прямая линия) — это
    бесконечная линия, по которой
    проходит кратчайший путь между
    любыми двумя её точками.
    • Любую прямую на плоскости можно
    задать уравнением прямой первой степени
    вида
    A x + B y + C = 0,
    где A и B не могут быть одновременно равны
    нулю.

    5. Уравнение прямой с угловым коэффициентом

    • Общее уравнение прямой при B≠0 можно
    привести к виду
    y = k x + b,
    где k — угловой коэффициент равный тангенсу
    угла, образованного данной прямой и
    положительным направлением оси ОХ.

    6. Уравнение прямой в отрезках на осях

    • Если прямая пересекает оси OX и OY в
    точках с координатами (a, 0) и (0, b), то она
    может быть найдена используя
    формулу уравнения прямой в отрезках
    x
    a
    +
    y
    b
    =1

    7. Уравнение прямой, проходящей через две различные точки на плоскости

    • Если прямая проходит через две точки
    A(x1, y1) и B(x2, y2), такие
    что x1 ≠ x2 и y1 ≠ y2 то уравнение
    прямой можно найти, используя
    следующую формулу
    x -x1
    x2 -x1
    =
    y -y1
    y2 — y1

    8.

    Параметрическое уравнение прямой на плоскости• Параметрические уравнения прямой могут
    быть записаны следующим образом
    x = l t +x0
    y = m t + y0
    где (x0, y0) — координаты точки лежащей на
    прямой,
    {l,m} — координаты направляющего вектора
    прямой.

    9. Каноническое уравнение прямой на плоскости

    • Если известны координаты точки A(x0, y0)
    лежащей на прямой и направляющего
    вектора n ={l;m}, то уравнение прямой
    можно записать в каноническом виде,
    используя следующую формулу
    x -x0
    l
    =
    y — y0
    m

    10. Уравнение прямой, проходящей через две различные точки в пространстве

    • Если прямая проходит через две точки
    A(x1,y1,z1) и B(x2,y2,z2), такие что
    x1 ≠ x2, y1 ≠ y2 и z1 ≠ z2 то уравнение
    прямой можно найти используя
    следующую формулу
    x -x1
    x2 -x1
    =
    y -y1
    y2 -y1
    =
    z -z1
    z2 — z1

    11. Параметрическое уравнение прямой в пространстве

    • Параметрические уравнения прямой могут
    быть записаны следующим образом
    x = l t +x0
    y = m t + y0
    z = n t + z0
    где (x0, y0, z0) — координаты точки лежащей на
    прямой,
    {l; m; n} — координаты направляющего вектора
    прямой.

    12. Каноническое уравнение прямой в пространстве

    • Если известны координаты точки A(x0, y0, z0)
    лежащей на прямой и направляющего
    вектора n={l;m;n}, то уравнение прямой
    можно записать в каноническом виде,
    используя следующую формулу
    x -x0
    l
    =
    y -y0
    m
    =
    z -z0
    n

    13. Прямая как линия пересечения двух плоскостей

    • Если прямая является пересечением двух
    плоскостей, то ее уравнение можно задать
    следующей системой уравнений
    A1x + B1y + C1z + D1 = 0
    A2x + B2y + C2z + D2 = 0
    при условии, что не имеет место равенство
    A1
    A2
    =
    B1
    B2
    =
    C1
    C2
    .

    14. Плоскость — есть поверхность, полностью содержащая, каждую прямую, соединяющую любые её точки.

    Плоскость — есть поверхность, полностью
    содержащая, каждую прямую,
    соединяющую любые её точки.
    • Любую плоскость можно
    задать уравнением плоскости первой
    степени вида
    Ax+By+Cz+D=0
    где A, B и C не могут быть одновременно
    равны нулю.

    15. Уравнение плоскости в отрезках

    • Если плоскость пересекает оси OX, OY и OZ в
    точках с координатами (a, 0, 0), (0, b, 0) и (0,
    0, с), то она может быть найдена, используя
    формулу уравнения плоскости в отрезках
    x
    a
    +
    y
    b
    +
    z
    c
    =1

    16. Уравнение плоскости, проходящей через точку, перпендикулярно вектору нормали

    • Чтобы составить уравнение плоскости, зная
    координаты точки плоскости M(x0, y0, z0) и
    вектора нормали плоскости n = {A; B; C} можно
    использовать следующую формулу.
    A(x — x0) + B(y — y0) + C(z — z0) = 0

    17. Уравнение плоскости, проходящей через три заданные точки, не лежащие на одной прямой

    • Если заданы координаты трех точек
    A(x1, y1, z1), B(x2, y2, z2) и C(x3, y3, z3),
    лежащих на плоскости, то уравнение
    плоскости можно найти по следующей
    формуле
    x — x1 y — y1 z — z1
    x2 — x1 y2 — y1 z2 — z1
    x3 — x1 y3 — y1 z3 — z1
    =0

    18. Расстояние от точки до плоскости — равно длине перпендикуляра, опущенного из точки на плоскость.

    Расстояние от точки до плоскости —
    равно длине перпендикуляра,
    опущенного из точки на плоскость.
    • Если задано уравнение плоскости Ax + By +
    Cz + D = 0, то расстояние от точки M(Mx, My,
    Mz) до плоскости можно найти, используя
    следующую формулу:
    |A·Mx + B·My + C·Mz + D|
    d=
    √A2 + B2 + C2

    19. Расстояние между плоскостями — равно длине перпендикуляра, опущенного с одной плоскости на другую.

    Расстояние между плоскостями — равно
    длине перпендикуляра, опущенного с
    одной плоскости на другую.
    • Если заданы уравнения параллельных
    плоскостей Ax + By + Cz + D1 = 0 и
    Ax + By + Cz + D2 = 0, то расстояние между
    плоскостями можно найти, используя
    следующую формулу
    |D2 — D1|
    d=
    √A2 + B2 + C2

    20. Расстояние от точки до прямой — равно длине перпендикуляра, опущенного из точки на прямую.

    Расстояние от точки до прямой — равно
    длине перпендикуляра, опущенного из
    точки на прямую.
    • Если задано уравнение прямой Ax + By + C = 0,
    то расстояние от точки M(Mx, My) до прямой
    можно найти, используя следующую формулу
    |A·Mx + B·My + C|
    d=
    √A2 + B2

    21.

    Расстояние от точки до прямой — равно длине перпендикуляра, опущенного из точки на прямую.Расстояние от точки до прямой —
    равно длине перпендикуляра,
    опущенного из точки на прямую.
    • Если s = {m; n; p} — направляющий вектор
    прямой l, M1(x1, y1, z1) — точка лежащей на
    прямой, тогда расстояние от точки
    M0(x0, y0, z0) до прямой l можно найти,
    используя формулу
    d=
    |M0M1×s|
    |s|
    • Двугранный угол между плоскостями равен углу
    образованному нормальными векторами этих
    плоскостей.
    • Двугранный угол между плоскостями равен углу
    образованному прямыми l1 и l2, лежащими в
    соответствующих плоскостях и перпендикулярными
    линии пересечения плоскостей.
    • Если заданы уравнения плоскостей A1x + B1y + C1z +
    D1 = 0 и A2x + B2y + C2z + D2 = 0, то угол между
    плоскостями можно найти, используя следующую
    формулу
    |A1·A2 + B1·B2 + C1·C2|
    cos α =
    √A12 + B12 + C12√A22 + B22 + C22

    23. Угол между прямой и плоскостью — это угол между прямой и ее проекцией на эту плоскость.

    Угол между прямой и плоскостью — это угол
    между прямой и ее проекцией на эту
    плоскость.
    • Если в пространстве заданы направляющий
    вектор прямой L s = {l; m; n} и уравнение
    плоскости Ax + By + Cz + D = 0, то угол
    между этой прямой и плоскостью можно
    найти используя формулу
    |A·l+B· m+C· n|
    sin φ =
    √A2 + B2 + C2 · √l2 + m2 + n2

    Пояснение к уроку: Точки, средние точки и расстояния в пространстве

    В этом объяснении мы научимся найдите координаты точки в 3D, расстояние между двумя точками в 3D, а также координаты средней и конечной точек в 3D, используя формулу.

    Мы уже должны знать, как найти все это в двух измерениях. Любая точка в двух измерениях будет иметь 𝑥- и 𝑦-координату и может быть записана в виде (𝑥,𝑦). Каждое из действительных чисел в упорядоченной паре представляет собой смещение этой точки от начала координат, другими словами, положительное или отрицательное расстояние от точки (0,0).

    Если две точки 𝐴 и 𝐵 имеют координаты (𝑥,𝑦) и (𝑥,𝑦) соответственно, то мы можем вычислить их середину по формуле 𝑥+𝑥2,𝑦+𝑦2.

    Если две точки 𝐴 и 𝐵 имеют координаты (𝑥,𝑦) и (𝑥,𝑦) соответственно, то мы можем вычислить расстояние между ними, используя формулу расстояния, полученную из теоремы Пифагора, (𝑥−𝑥)+(𝑦−𝑦).

    В этом объяснении мы исследуем, как мы можем расширить эти формулы, чтобы включить третью координату при работе с точками в трех измерениях.

    Определение: Координаты точки в трехмерном пространстве

    Любая точка в трех измерениях будет иметь 𝑥-, 𝑦-, и 𝑧-координат и может быть записан в виде (𝑥,𝑦,𝑧). Каждое из действительных чисел в упорядоченной тройке дает расстояние от начала координат, измеренное вдоль соответствующей оси.

    В нашем первом примере мы рассмотрим, в какой плоскости лежит точка, одна из координат которой равна нулю.

    Пример 1. Определение плоскости, в которой находится заданная координата

    В какой из следующих координатных плоскостей находится точка (−7,−8,0) ложь?

    1. 𝑥𝑦
    2. 𝑥𝑧
    3. 𝑦𝑧

    Ответ

    Мы знаем, что точка в 3D будет иметь 𝑥-, 𝑦- и 𝑧-координаты. В этом вопросе 𝑥=−7, 𝑦=−8 и 𝑧=0.

    Поскольку 𝑧-координата равна нулю, точка находится на нулевом расстоянии от начала координат в 𝑧-направлении. Это означает, что он будет лежать на 𝑥𝑦-плоскости. В самом деле, любая точка с координаты (𝑥,𝑦,0) будут лежать на этой плоскости.

    Таким образом, мы можем заключить, что точка (−7,−8,0) лежит на 𝑥𝑦-плоскости.

    Определение: Три координатные плоскости

    Любая точка с координатами (𝑥,𝑦,0) будет лежать на 𝑥𝑦-плоскости.

    Аналогично, любая точка с координатами (𝑥,0,𝑧) будет лежать на 𝑥𝑧-плоскость, и любая точка с координатами (0,𝑦,𝑧) будет лежать на 𝑦𝑧-плоскости.

    В нашем следующем вопросе мы рассмотрим, как мы можем определить координаты точки в трех измерениях.

    Пример 2. Нахождение координат заданной точки в 3D

    Определить координаты точки 𝐴.

    Ответ

    Любая точка в трех измерениях будет иметь 𝑥-, 𝑦- и 𝑧-координаты и может быть записана в виде (𝑥,𝑦,𝑧).

    Двигаясь от начала координат, мы проходим 3 единицы в положительном 𝑥-направлении, −3 единицы в 𝑦-направлении и, наконец, 3 единицы в 𝑧-направлении.

    Это означает, что 𝑥=3, 𝑦=−3 и 𝑧=3.

    Координаты точки 𝐴 равны (3,−3,3).

    Напомним, что формула средней точки в двух измерениях просто говорит нам найти среднее значение двух точек. Находим среднее значение 𝑥-координат и среднее значение 𝑦-координат. Теперь мы расширим эту идею на три измерения, найдя также среднее значение 𝑧-координат.

    Чтобы найти среднее любых двух чисел, мы складываем их, а затем делим их сумму на два.

    Определение: середина двух точек в трехмерном пространстве

    Если две точки 𝐴 и 𝐵 имеют координаты (𝑥,𝑦,𝑧) и (𝑥,𝑦,𝑧) соответственно, то можно вычислить их среднюю точку, используя следующую формулу: 𝑥+𝑥2,𝑦+𝑦2,𝑧+𝑧2.

    В нашем следующем примере мы будем использовать эту формулу для определения середины двух точек в пространстве.

    Пример 3: Нахождение координат средней точки в 3D

    Баллы 𝐴 и 𝐵 имеют координаты (8,−8,−12) и (−8,5,−8) соответственно. Определить координаты середины 𝐴𝐵.

    Ответ

    Чтобы найти середину двух точек в трех измерениях, мы будем использовать формулу для вычисления середины координат (𝑥,𝑦,𝑧) и (𝑥,𝑦,𝑧): 𝑥+𝑥2,𝑦+𝑦2,𝑧+𝑧2. 

    Пусть точка 𝐴 имеет координаты (𝑥,𝑦,𝑧), а точка 𝐵 имеет координаты (𝑥 ,𝑦,𝑧 ).

    Середина между точками 𝐴 и 𝐵 равна =8+(−8)2,−8+52,−12+(−8)2=02,−32,−202=0,−32 ,−10.

    Координаты середины 𝐴𝐵 составляют 0,−32,−10.

    В нашем следующем примере мы будем использовать формулу средней точки для определения конечной точки по средней точке двух точек в пространстве и другой конечной точке.

    Пример 4: Нахождение координат конечной точки отрезка линии по координатам середины и координатам начальной точки

    Учитывая, что точка (0,17,−10) является средней точкой 𝐴𝐵 и это 𝐴(−19,7,14), каковы координаты 𝐵?

    Ответ

    Чтобы найти середину двух точек в трех измерениях, мы будем использовать формулу для вычисления середины координат (𝑥,𝑦,𝑧) и (𝑥,𝑦,𝑧) : 𝑥+𝑥2,𝑦+𝑦2,𝑧+𝑧2.

    Мы знаем, что точка 𝐴 имеет координаты (−19,7,14) и пусть точка 𝐵 имеет координаты (𝑥,𝑦,𝑧). Середина между этими двумя точками имеет координаты (0,17,−10).

    Подставив эти значения в формулу, получим (0,17,−10)=−19+𝑥2,7+𝑦2,14+𝑧2.

    Затем мы можем приравнять отдельные компоненты, что даст нам решение трех уравнений.

    Во-первых, 𝑥-координата дает нам 0=−19+𝑥2.

    Умножая обе части уравнения на 2, получаем 0=−19+𝑥.

    Итак, 19=𝑥.

    Во-вторых, 𝑦-координата дает нам 17=7+𝑦2.

    Умножив обе части уравнения на 2, мы получим 34=7+𝑦.

    Итак, 27=𝑦.

    Наконец, 𝑧-координата дает нам −10=14+𝑧2.

    Умножая обе части уравнения на 2, мы получаем −20=14+𝑧.

    Итак, −34=𝑧.

    Координаты точки 𝐵 равны (19,27,−34).

    В двух измерениях мы можем вычислить расстояние между двумя точками, используя адаптацию теоремы Пифагора. Это утверждает, что 𝑎+𝑏=𝑐, где 𝑐 — длина самой длинной стороны, известной как гипотенуза, прямоугольного треугольника.

    Если две точки 𝐴 и 𝐵 имеют координаты (𝑥,𝑦) и (𝑥,𝑦) соответственно, то можно вычислить расстояние между ними по следующей формуле: (𝑥−𝑥)+(𝑦−𝑦). 

    Теперь мы рассмотрим, как мы можем вычислить расстояние между двумя точками в три измерения.

    Рассмотрим трехмерную прямоугольную призму 𝐴𝐵𝐶𝐷𝐸𝐹𝐺𝐻, нарисованную ниже, и предположим, что мы хотим пройти от самого нижнего левого переднего угла, 𝐴, до самого верхнего правого заднего угла, 𝐺.

    Сначала рассмотрим треугольник 𝐴𝐵𝐹 в основании призмы. Теорема Пифагора говорит нам, что 𝐴𝐹=𝐴𝐵+𝐵𝐹.

    Итак, 𝐴𝐹=√𝑥+𝑦.

    Теперь делаем еще один треугольник 𝐴𝐹𝐺, с основанием вдоль 𝐴𝐹 и высотой 𝐹𝐺.

    Мы можем снова использовать теорему Пифагора так, чтобы 𝐴𝐺=𝐴𝐹+𝐹𝐺. Подставляя длины 𝐴𝐹 и 𝐹𝐺, мы видим, что 𝐴𝐺=√𝑥+𝑦+𝑧.

    Следовательно, 𝐴𝐺=√𝑥+𝑦+𝑧.

    Определение: расстояние между двумя точками в трехмерном пространстве

    Если две точки 𝐴 и 𝐵 имеют координаты (𝑥,𝑦,𝑧) и (𝑥,𝑦,𝑧), соответственно, то мы можем рассчитать расстояние между ними по следующей формуле: (𝑥−𝑥)+(𝑦−𝑦)+(𝑧−𝑧). теоремы Пифагора в трех измерениях; мы находим сумму квадратов разницы между каждой координатой и затем квадратный корень этого ответа.

    В наших последних двух вопросах мы вычислим кратчайшее расстояние между точкой и одной из осей, а также расстояние между двумя точками в пространстве.

    Пример 5. Нахождение расстояния между двумя точками по их координатам в трех измерениях

    Нахождение расстояния между двумя точками 𝐴(−7,12,3) и 𝐵(−4,−1,−8).

    Ответ

    Чтобы вычислить расстояние между двумя точками в трех измерениях, мы будем использовать следующую формулу, где две точки 𝐴 и 𝐵 имеют координаты (𝑥,𝑦,𝑧) и (𝑥,𝑦,𝑧) соответственно: (𝑥−𝑥)+(𝑦−𝑦)+(𝑧−𝑧).

    Пусть точка 𝐴 имеет координаты (𝑥,𝑦,𝑧), а точка 𝐵 имеет координаты (𝑥,𝑦,𝑧).

    Расстояние между ними равно =√(−4−(−7))+(−1−12)+(−8−3)=√(3)+(−13)+(−11)=√9+169+121=√299 .

    Расстояние между двумя точками 𝐴(−7,12,3) и 𝐵(−4,−1,−8) составляет √299 единиц длины.

    Пример 6: Нахождение расстояния между точкой и осью в 3D

    Каково расстояние между точкой (19,5,5) и осью 𝑥?

    Ответ

    Мы знаем, что любая точка будет лежать на оси 𝑥, если и ее 𝑦-координата, и ее 𝑧-координата равны нулю. Это означает, что мы можем определить точку на оси 𝑥 как (𝑥,0,0).

    Признаем, что искомое расстояние — это перпендикулярное расстояние от точки до оси 𝑥, что означает проекцию точки на 𝑥-ось будет в точке (19,0,0).

    Расстояние между двумя точками можно рассчитать по формуле следующим образом √(19−19)+(5−0)+(5−0)=√0+(5)+(5)=√50=5√2.

    Расстояние между точка (19,5,5) и ось 𝑥 составляют 5√2 единиц длины.

    Мы закончим это объяснение повторением некоторых ключевых моментов.

    Ключевые точки

    • Любая точка в трех измерениях имеет координаты, записанные в виде (𝑥,𝑦,𝑧).
    • Если 𝑧-координата равна нулю, то мы знаем, что точка лежит в 𝑥𝑦-плоскости; если 𝑦-координата равна нулю, то мы знаем, что точка лежит в 𝑥𝑧-плоскость; а если 𝑥-координата равна нулю, то мы знаем, что точка лежит в 𝑦𝑧-плоскости.
    • Если и 𝑦-координата, и 𝑧-координата равны нулю, то точка лежит на 𝑥-оси; если и 𝑥-координата, и 𝑧-координата равна нулю, то точка лежит на 𝑦-оси; и если и 𝑥-координата, и 𝑦-координата равны равна нулю, то точка лежит на оси 𝑧.
    • Середина двух точек с координатами (𝑥,𝑦,𝑧) и (𝑥,𝑦,𝑧) лежит в точке 𝑥+𝑥2,𝑦+𝑦2,𝑧+𝑧2.
    • Мы также можем использовать формулу средней точки для вычисления конечной точки отрезка, учитывая среднюю точку и другую конечную точку.
    • Расстояние между двумя точками с координатами (𝑥,𝑦,𝑧) и (𝑥,𝑦,𝑧) это равно (𝑥−𝑥)+(𝑦−𝑦)+(𝑧−𝑧).

    Центр отрезка.

    Произношение: /ˈmɪdˌpɔɪnt/ Объяснение

    Середина — это точка равноудаленный между двумя точками. Середина находится на отрезке, соединяющем две точки и делит отрезок ровно пополам. Точное математическое определение средней точки:

    Средняя точка A M между точками А и В точка на прямой AB такая, что АМ = МБ .

    Как построить среднюю точку

    Шаг Иллюстрация Описание
    1 Начните с точек A и Б .
    2 Нарисуйте отрезок АВ.
    3 Нарисуйте круг с центром в точке А и радиус AB .
    4 Нарисуйте круг с центром в точке B и радиус AB
    5 Отметить одно пересечение двух кругов как точку C и другой перекресток двух круги как точка D .
    6 Нарисуйте отрезок CD.
    7 Отметить точку пересечения сегмента линии АВ и отрезок CD как М . Точка M — средняя точка.
    Таблица 1: Построение средней точки.

    Как вычислить среднюю точку в одномерном метрическом пространстве

    Нажмите на синие точки и перетащите их, чтобы изменить фигуру.

    Что произойдет, если B окажется слева от A?
    Манипулятивное 8 — Расчет средней точки в одном измерении Создано с помощью GeoGebra.

    Формула средней точки в одномерном пространстве между А и В есть . Нажать на синие точки в манипуляции 1 и перетащите их, чтобы изменить фигуру.

    Как вычислить среднюю точку в метрическом двумерном пространстве

    Нажмите на синие точки и перетащите их, чтобы изменить фигуру.

    Манипулятивное 9 — Расчет средней точки в двух измерениях Создано с помощью GeoGebra.

    Середина делит отрезок ровно пополам. Этот факт можно использовать найти формулу для середины отрезка в двумерном пространстве. Евклидово пространство, такое как декартова система координат. X-координата средняя точка будет на полпути между x-координатами двух точек, и y-координата средней точки будет на полпути между y-координатами две точки. Формула середины отрезка с концами и является .

    Как вычислить среднюю точку в метрическом n-мерном пространстве

    Алгоритм вычисления конечной точки в двумерном пространство может быть обобщено для n-мерного пространства. Учитывая две точки и середина .

    Доказательство: Если

    M является средней точкой АБ, затем утра = MB

    Это доказательство является доказательством абзаца или неофициальным доказательством.

    Определение середины отрезка таково, что 902:01 AM = МБ . Другими словами, длины двух отрезков равны. По определению конгруэнтности АМ конгруэнтно МБ тогда и только тогда, когда AM и MB имеют одинаковую меру. Так как по определению середины АМ и МБ имеют одинаковую меру, утра = МБ.

    Ссылки

    1. МакАдамс, Дэвид Э.. Словарь всех математических слов, середина . 2-й классный выпуск 20150108-4799968. стр. 117. Life is a Story Problem LLC. 8 января 2015. Купить книгу

    Дополнительная информация

    • Евклид Александрийский. Элементы . Университет Кларка. 06.09.2018. https://mathcs.clarku.edu/~djoyce/elements/elements.html.

    Цитируйте эту статью как:

    МакАдамс, Дэвид Э. Середина . 25.04.2019. Вся энциклопедия математических слов. ООО «Жизнь — это проблема истории». https://www.allmathwords.org/en/m/midpoint.html.

    Авторы изображений

    • Все изображения и манипуляции принадлежат Дэвиду МакАдамсу, если не указано иное. Все изображения Дэвида МакАдамса защищены авторским правом © Life is a Story Problem LLC и находятся под лицензией Creative Commons Attribution-ShareAlike 4.0 International License.

    История изменений

    25.

    Квантили нормального распределения таблица: Таблица. Функция распределения вероятностей стандартного нормального закона. Таблица квантилей стандартного нормального закона распределения.

    Таблица. Функция распределения вероятностей стандартного нормального закона. Таблица квантилей стандартного нормального закона распределения.

    Раздел недели: Скоропись физического, математического, химического и, в целом, научного текста, математические обозначения. Математический, Физический алфавит, Научный алфавит.


    Поиск на сайте DPVA

    Поставщики оборудования

    Полезные ссылки

    О проекте

    Обратная связь

    Ответы на вопросы.

    Оглавление

    Таблицы DPVA.ru — Инженерный Справочник



    Адрес этой страницы (вложенность) в справочнике dpva.ru:  главная страница / / Техническая информация/ / Математический справочник / / Теория вероятностей. Математическая статистика. Комбинаторика. / / Таблица. Функция распределения вероятностей стандартного нормального закона. Таблица квантилей стандартного нормального закона распределения.

    Поделиться:   

    Таблица. Функция распределения вероятностей стандартного нормального закона.

    • Пример использования: на пересечении строки 1.3 и столбца 0,02 находим Ф(1,32)= 0,9049
    • Посмотреть: Таблица квантилей стандартного нормального закона распределения.
    Таблица. Функция распределения стандартного нормального закона.

    t

    . 00

    .01

    .02

    .03

    .04

    .05

    .06

    .07

    .08

    .09

    0

    .5000 .5040 .5080 .5120 .5160 .5199 .5239 .5279 .5319 .5359

    .1

    .5398 .5438 .5478 .5517 .5557 . 5596 .5636 .5675 .5714 .5753

    .2

    .5793 .5832 .5871 .5910 .5948 .5987 .6026 .6064 .6103 .6141

    .3

    .6179 .6217 .6255 .6293 .6331 .6368 .6406 .6443 .6480 .6517

    .4

    .6554 .6591 .6628 .6664 .6700 .6736 .6772 .6808 . 6844 .6879

    .5

    .6915 .6950 .6985 .7019 .7054 .7088 .7123 .7157 .7190 .7224

    .6

    .7257 .7291 .7324 .7357 .7389 .7422 .7454 .7486 .7517 .7549

    .7

    .7580 .7611 .7642 .7673 .7704 .7734 .7764 .7794 .7823 .7852

    . 8

    .7881 .7910 .7939 .7967 .7995 .8023 .8051 .8078 .8106 .8133

    .9

    .8159 .8186 .8212 .8238 .8264 .8289 .8315 .8340 .8365 .8389

    1.0

    .8413 .8438 .8461 .8485 .8508 .8531 .8554 .8577 .8599 .8621

    1.1

    .8643 .8665 . 8686 .8708 .8729 .8749 .8770 .8790 .8810 .8830

    1.2

    .8849 .8869 .8888 .8907 .8925 .8944 .8962 .8980 .8997 .9015

    1.3

    .9032 .9049 .9066 .9082 .9099 .9115 .9131 .9147 .9162 .9177

    1.4

    .9192 .9207 .9222 .9236 .9251 . 9265 .9279 .9292 .9306 .9319

    1.5

    .9332 .9345 .9357 .9370 .9382 .9394 .9406 .9418 .9429 .9441

    1.6

    .9452 .9463 .9474 .9484 .9495 .9505 .9515 .9525 .9535 .9545

    1.7

    .9554 .9564 .9573 .9582 .9591 .9599 .9608 .9616 . 9625 .9633

    1.8

    .9641 .9649 .9656 .9664 .9671 .9678 .9686 .9693 .9699 .9706

    1.9

    .9713 .9719 .9726 .9732 .9738 .9744 .9750 .9756 .9761 .9767

    2.0

    .9772 .9778 .9783 .9788 .9793 .9798 .9803 .9808 .9812 .9817

    2. 1

    .9821 .9826 .9830 .9834 .9838 .9842 .9846 .9850 .9854 .9857

    2.2

    .9861 .9864 .9868 .9871 .9875 .9878 .9881 .9884 .9887 .9890

    2.3

    .9893 .9896 .9898 .9901 .9904 .9906 .9909 .9911 .9913 .9916

    2.4

    .9918 .9920 . 9922 .9925 .9927 .9929 .9931 .9932 .9934 .9936

    2.5

    .9938 .9940 .9941 .9943 .9945 .9946 .9948 .9949 .9951 .9952

    2.6

    .9953 .9955 .9956 .9957 .9959 .9960 .9961 .9962 .9963 .9964

    2.7

    .9965 .9966 .9967 .9968 .9969 . 9970 .9971 .9972 .9973 .9974

    2.8

    .9974 .9975 .9976 .9977 .9977 .9978 .9979 .9979 .9980 .9981

    2.9

    .9981 .9982 .9982 .9983 .9984 .9984 .9985 .9985 .9986 .9986

    3.0

    .9987 .9987 .9987 .9988 .9988 .9989 .9989 .9989 . 9990 .9990

    3.1

    .9990 .9991 .9991 .9991 .9992 .9992 .9992 .9992 .9993 .9993

    3.2

    .9993 .9993 .9994 .9994 .9994 .9994 .9994 .9995 .9995 .9995

    3.3

    .9995 .9995 .9995 .9996 .9996 .9996 .9996 .9996 .9996 .9997

    3. 4

    .9997 .9997 .9997 .9997 .9997 .9997 .9997 .9997 .9997 .9998

    Таблица квантилей стандартного нормального закона распределения. Квантили обозначены : Ф(uα)=α

    • Посмотреть:Таблица. Функция распределения стандартного нормального закона.
    Таблица квантилей стандартного нормального закона распределения.

    ?

    uα

    ?

    uα

    ?

    uα

    . 50

    0

    .91

    1.341

    .995

    2.576

    .55

    .126

    .92

    1.405

    .999

    3.090

    .60

    .253

    .93

    1.476

    .9995

    3.291

    .65

    .385

    .94

    1.555

    .9999

    3.719

    . 70

    .524

    .95

    1.645

    .99995

    3.891

    .75

    .674

    .96

    1.751

    .99999

    4.265

    .80

    .842

    .97

    1.881

    .999995

    4.417

    .85

    1.036

    .98

    2.054

    .999999

    4.753

    . 90

    1.282

    .99

    2.326

    .9999999

    5.199

    Поиск в инженерном справочнике DPVA. Введите свой запрос:

    Дополнительная информация от Инженерного cправочника DPVA, а именно — другие подразделы данного раздела:

    Поиск в инженерном справочнике DPVA. Введите свой запрос:

    Если Вы не обнаружили себя в списке поставщиков, заметили ошибку, или у Вас есть дополнительные численные данные для коллег по теме, сообщите , пожалуйста.
    Вложите в письмо ссылку на страницу с ошибкой, пожалуйста.

    Коды баннеров проекта DPVA. ru
    Начинка: KJR Publisiers

    Консультации и техническая
    поддержка сайта: Zavarka Team

    Проект является некоммерческим. Информация, представленная на сайте, не является официальной и предоставлена только в целях ознакомления. Владельцы сайта www.dpva.ru не несут никакой ответственности за риски, связанные с использованием информации, полученной с этого интернет-ресурса. Free xml sitemap generator

    Прикладная статистика: Исследование зависимостей

    Прикладная статистика: Исследование зависимостей
      

    Айвазян С. А. и др. Прикладная статистика: Исследование зависимостей: Справ. изд. / С. А. Айвазян, И. С. Енюков, Л. Д. Мешалкин; Под ред. С. А. Айвазяна. — М.: Финансы и статистика, 1985. — 487 с.

    Данная книга является логическим продолжением справочного издания «Прикладная статистика: Основы моделирования и первичная обработка данных», вышедшего в 1983 г. В ней рассматриваются методы корреляционного, регрессионного и дисперсионного анализа. Приводятся их алгоритмы и обзор программного обеспечения.

    Для статистиков, экономистов, социологов, программистов.



    Оглавление

    ПРЕДИСЛОВИЕ
    Введение. СТАТИСТИЧЕСКОЕ ИССЛЕДОВАНИЕ ЗАВИСИМОСТЕЙ СОДЕРЖАНИЕ, ЗАДАЧИ, ОБЛАСТИ ПРИМЕНЕНИЯ
    В.2. Какова конечная прикладная цель статистического исследования зависимостей?
    В.3. Математический инструментарий
    В.4. Некоторые типовые задачи практики
    В.5. Основные типы зависимостей между количественными переменными
    В.6. Основные этапы статистического исследования зависимостей
    ВЫВОДЫ
    Раздел I. АНАЛИЗ СТРУКТУРЫ И ТЕСНОТЫ СТАТИСТИЧЕСКОЙ СВЯЗИ МЕЖДУ ИССЛЕДУЕМЫМИ ПЕРЕМЕННЫМИ (корреляционный анализ)
    1. 1.1. Понятие индекса корреляции.
    1.1.2. Коэффициент корреляции как измеритель степени тесноты связи в двумерных нормальных схемах.
    1.1.3. Распределение выборочного коэффициента корреляции и проверка гипотезы о статистической значимости линейной связи.
    1.1.4. Влияние ошибок измерения на величину коэффициента корреляции.
    1.1.5. Измерение степени тесноты связи при нелинейной зависимости.
    1.2. Анализ частных («очищенных») связей
    1.2.2. Частные коэффициенты корреляции и их выборочные значения.
    1.2.3. Статистические свойства выборочных частных коэффициентов корреляции (проверка на статистическую значимость их отличия от нуля, доверительные интервалы).
    1.3. Анализ множественных связей
    1.3.2. Множественный коэффициент корреляции и его свойства (общий случай).
    1.3.3. Вычисление и свойства множественного коэффициента корреляции в рамках линейных нормальных моделей.
    1.3.4. Примеры.
    ВЫВОДЫ
    Глава 2. АНАЛИЗ СТАТИСТИЧЕСКОЙ СВЯЗИ МЕЖДУ ПОРЯДКОВЫМИ (ОРДИНАЛЬНЫМИ) ПЕРЕМЕННЫМИ
    2. 1. Ранговая корреляция
    2.1.2. Понятие ранговой корреляции.
    2.1.3. Основные задачи статистического анализа связей между ранжировками.
    2.1.4. Вероятностные пространства ранжировок, генерируемые порядковыми переменными [14, гл. 4, 5].
    2.2. Анализ и измерение парных ранговых статистических связей
    2.2.1. Ранговый коэффициент корреляции Спирмэна.
    2.2.2. Ранговый коэффициент корреляции Кендалла.
    2.2.3. Обобщенная формула для парного коэффициента корреляции и связь между коэффициентами Спирмэна и Кендалла.
    2.2.4. Статистические свойства выборочных характеристик парной ранговой связи.
    2.3. Анализ множественных ранговых связей
    2.3.2. Проверка статистической значимости выборочного значения коэффициента конкордации.
    2.3.3. Использование коэффициента конкордации в решении основных задач статистического анализа ранговых связей.
    2.3.4. Примеры.
    ВЫВОДЫ
    Глава 3. АНАЛИЗ СВЯЗЕЙ МЕЖДУ КЛАССИФИКАЦИОННЫМИ (НОМИНАЛЬНЫМИ) ПЕРЕМЕННЫМИ
    3.1. Таблицы сопряженности
    3. 1.2. Логарифмически-линейная. параметризация таблиц сопряженности.
    3.1.3. Проверка гипотез.
    3.1.4. Меры связи между строками и столбцами таблицы.
    3.2. Приписывание численных значений качественным переменным (дуальное шкалирование)
    3.2.1. Методическое место дуального шкалирования.
    3.2.2. Максимизация F-отношения суммы квадратов отклонений между объектами к полной сумме квадратов отклонений.
    3.2.3. Двойственность в определении V и W.
    3.2.4. Максимизация коэффициента корреляции.
    3.2.5. Изучение оптимального решения.
    3.2.6. Таблицы «объект—многомерный отклик».
    ВЫВОДЫ
    Глава 4. АНАЛИЗ СТРУКТУРЫ СВЯЗЕЙ МЕЖДУ КОМПОНЕНТАМИ МНОГОМЕРНОГО ВЕКТОРА
    4.1.1. Цепи Маркова.
    4.1.3. Математические задачи, связанные с изучением распределений с ДСЗ.
    4.2. Распределение с древообразной структурой зависимостей
    4.2.1. Предварительные сведения из теории графов.
    4.2.2. Распределения с древообразной структурой зависимостей (ДСЗ).
    4.3. Оценка графа структуры зависимостей компонент нормального вектора
    4. 3.2. Построение графа структуры зависимостей по корреляционной матрице.
    4.3.3. Асимптотика Колмогорова — Деева.
    4.4. R(k)-распределения
    4.4.1. Основные определения. Начнем с обобщения понятия распределения с ДСЗ.
    4.4.2. Нормальное R(k)-распределение.
    4.4.3. Восстановление графа структуры зависимостей.
    4.5. Структура связей нормального вектора (общий случай)
    4.5.1. Марковская тройка. Структура многомерного вектора.
    4.5.2. Информационная интерпретация структуры связей.
    4.5.3. Использование структуры для представления распределения в виде композиции более простых распределений.
    ВЫВОДЫ
    Раздел II. ИССЛЕДОВАНИЕ ВИДА ЗАВИСИМОСТИ МЕЖДУ КОЛИЧЕСТВЕННЫМИ ПЕРЕМЕННЫМИ (регрессионный анализ)
    5.1. Функция регрессии как условное среднее и ее интерпретация в рамках многомерной нормальной модели
    5.2. Функция «дельта»-регрессии как решение оптимизационной задачи
    5.3. Взаимоотношения различных регрессий
    ВЫВОДЫ
    Глава 6. ВЫБОР ОБЩЕГО ВИДА ФУНКЦИИ РЕГРЕССИИ
    6. 1. Использование априорной информации о содержательной сущности анализируемой зависимости
    6.2. Предварительный анализ геометрической структуры исходных данных
    6.2.1. Содержание геометрического анализа парных корреляционных полей.
    6.2.2. Учет и формализация «гладких» свойств искомой функции регрессии.
    6.2.3. Некоторые вспомогательные преобразования, линеаризующие исследуемую парную зависимость.
    6.3. Математико-статистические методы в задаче параметризации модели регрессии
    6.3.1. Компромисс между сложностью регрессионной модели и точностью ее оценивания.
    6.3.2. Поиск модели, наиболее устойчивой к варьированию состава выборочных данных, на основании которых она оценивается.
    6.3.3. Статистические критерии проверки гипотез об общем виде функции регрессии.
    ВЫВОДЫ
    Глава 7. ОЦЕНИВАНИЕ НЕИЗВЕСТНЫХ ЗНАЧЕНИЙ ПАРАМЕТРОВ, ЛИНЕЙНО ВХОДЯЩИХ В УРАВНЕНИЕ РЕГРЕССИОННОЙ ЗАВИСИМОСТИ
    7.1. Метод наименьших квадратов
    7.1.2. Свойства мнк-оценок.
    7.1.3. Ортогональная матрица плана.
    7.1.4. Параболическая регрессия и система ортогональных полиномов Чебышева.
    7.1.5. Обобщенный мнк.
    7.2. Функции потерь, отличные от квадратичной
    7.2.1. Функция потерь.
    7.2.3. Функции потерь, имеющие горизонтальную асимптоту.
    7.2.4. Эв-регрессия («лямбда»-регрессия).
    7.2.5. Минимизация систематической ошибки.
    7.3. Байесовское оценивание
    7.3.1. Введение априорной плотности распределения параметров.
    7.3.2. Апостериорное распределение параметров.
    7.3.3. Повторная выборка из той же совокупности.
    7.4. Многомерная регрессия
    7.4.1. Случай известной ковариационной матрицы ошибок.
    7.4.3. Эв-оценки.
    7.4.4. Использование многомерной регрессии для параметризации многомерных распределений.
    7.5. Оценивание параметров при наличии погрешностей в предикторных переменных (конфлюэнтный анализ)
    7.5.1. Основные типы задач конфлюэнтного анализа.
    7.5.2. Модифицированный мнк для схемы активного эксперимента.
    7.5.3. Пассивные наблюдения.
    7.5.4. Некоторые принципиальные отличия регрессионных задач (7.83) и (7.84).
    7.5.5. Неявное задание отклика.
    7.6. Оценивание в регрессионных моделях со случайными параметрами (регрессионные задачи второго рода)
    7.6.2. Случай, когда средние значения и ковариационная матрица оцениваемых параметров известны (требуется оценить параметры).
    7.6.3. Случай, когда известна только ковариационная матрица (требуется оценить параметры).
    7.6.4. Случай неизвестных.
    ВЫВОДЫ
    Глава 8. ОЦЕНИВАНИЕ ПАРАМЕТРОВ РЕГРЕССИИ В УСЛОВИЯХ МУЛЬТИКОЛЛИНЕАРНОСТИ И ОТБОР СУЩЕСТВЕННЫХ ПРЕДИКТОРОВ
    8.1. Явление мультиколлинеарности и его влияние на мнк-оценки
    8.2. Регрессия на главные компоненты
    8.3. Смещенное оценивание коэффициентов регрессии
    8.4. Редуцированные оценки для стандартной модели линейной регрессии
    8.4.2. Редуцированная оценка Мейера — Уилке.
    8.5. Оценки, связанные с ортогональным разложением
    8.5.1. Оптимальное взвешивание вклада главных компонент.
    8. 5.2. Оценка оптимальных вкладов главных компонент.
    8.6. Вопросы точности вычислительной реализации процедур линейного оценивания
    8.6.1. Два метода получения мнк-оценок.
    8.6.2. Оценки величин возмущений для решений центрированной и соответствующей ей нормальной системы уравнений.
    8.6.3. Центрирование и нормирование матрицы данных.
    8.6.4. Вычисление элементов ковариационной матрицы.
    8.7. Отбор существенных переменных в задачах линейной регрессии
    8.7.1. Влияние отбора переменных на оценку уравнения регрессии.
    8.7.2. Критерии качества уравнения регрессии.
    8.7.3. Схемы генерации наборов переменных.
    8.7.4. Пошаговые процедуры генерации наборов.
    8.7.5. Оператор симметричного выметания.
    8.7.6. Методические аспекты использования процедур отбора существенных предикторных переменных.
    ВЫВОДЫ
    Глава 9. ВЫЧИСЛИТЕЛЬНЫЕ АСПЕКТЫ МЕТОДА НАИМЕНЬШИХ КВАДРАТОВ
    9.1. Итерационные методы поиска оценок метода наименьших квадратов (мнк-оценок)
    9.1.2. Алгоритмы квазиградиентного типа.
    9.2. Градиентный спуск
    9.3. Метод Ньютона
    9.4 Метод Ньютона-Гаусса и его модификации
    9.4.2. Обсуждение скорости сходимости процедуры.
    9.4.3. Рекомендации по правилу остановки итерационной процедуры.
    9.5. Методы, не использующие вычисления производных
    9.6. Способы нахождения начального приближения
    9.7. Вопросы существования и единственности мнк-оценки
    ВЫВОДЫ
    Глава 10. НЕПАРАМЕТРИЧЕСКАЯ, ЛОКАЛЬНО-ПАРАМЕТРИЧЕСКАЯ И КУСОЧНАЯ АППРОКСИМАЦИЯ РЕГРЕССИОННЫХ ЗАВИСИМОСТЕЙ
    10.1. Непараметрическое оценивание регрессии
    10.2. Локальная параметрическая аппроксимация регрессии в одномерном случае
    10.3. Кусочно-параметрическая (сплайновая) техника аппроксимации регрессионных зависимостей
    10.3.1. Определение одномерных сплайнов.
    10.3.2. Выбор порядка сплайна, числа и положения узлов.
    10.3.3. Оценка параметров и проверка гипотез.
    10.3.4. Билинейные сплайны.
    ВЫВОДЫ
    Глава II. ИССЛЕДОВАНИЕ точности СТАТИСТИЧЕСКИХ ВЫВОДОВ в РЕГРЕССИОННОМ АНАЛИЗЕ
    11. 1 Линейный (относительно оцениваемых параметров) нормальный вариант идеализированной схемы регрессионной зависимости
    11.1.2. Решение основных задач по оценке точности регрессионной модели.
    11.1.3. Случаи линейной (по предикторным переменным) и полиномиальной регрессии.
    11.2. Нелинейный нормальный вариант идеализированной схемы регрессионной зависимости
    11.2.2. Решение основных задач по оценке точности нелинейной регрессионной модели.
    11.3. Исследование точности регрессионной модели в реалистической ситуации
    ВЫВОДЫ
    Глава 12. СТАТИСТИЧЕСКИЙ АНАЛИЗ АВТОРЕГРЕССИОННЫХ ДИНАМИЧЕСКИХ ЗАВИСИМОСТЕЙ
    12.1. Дискретные динамические модели
    12.2. Авторегрессия первого порядка
    12.3. Авторегрессия произвольного порядка
    ВЫВОДЫ
    Раздел III. ИССЛЕДОВАНИЕ ЗАВИСИМОСТИ КОЛИЧЕСТВЕННОГО РЕЗУЛЬТИРУЮЩЕГО ПОКАЗАТЕЛЯ ОТ ОБЪЯСНЯЮЩИХ ПЕРЕМЕННЫХ СМЕШАННОЙ ПРИРОДЫ
    Глава 13. ДИСПЕРСИОННЫЙ И КОВАРИАЦИОННЫЙ АНАЛИЗ
    13.1. Классификация моделей дисперсионного анализа по способу организации исходных данных
    13. 2. Однофакторный дисперсионный анализ
    13.3. Полный двухфакторный дисперсионный анализ
    13.4. Модели дисперсионного анализа со случайными факторами
    13.5. Ковариационный анализ (КА) и проблема статистического исследования смесей многомерных распределений
    13.6. Влияние нарушений основных предположений
    ВЫВОДЫ
    Раздел IV. СИСТЕМЫ ОДНОВРЕМЕННЫХ УРАВНЕНИЙ И ПРОГРАММНОЕ ОБЕСПЕЧЕНИЕ АППАРАТА СТАТИСТИЧЕСКОГО ИССЛЕДОВАНИЯ ЗАВИСИМОСТЕЙ
    14.1. Системы одновременных уравнений
    14.2. Спецификация модели и проблема идентифицируемости
    14.3. Рекурсивные системы
    14.4. Двух- и трехшаговый методы наименьших квадратов
    14.5. Метод неподвижной точки
    14.6. Сравнение методов
    ВЫВОДЫ
    Глава 15. ПРОГРАММНОЕ ОБЕСПЕЧЕНИЕ СТАТИСТИЧЕСКОГО ИССЛЕДОВАНИЯ ЗАВИСИМОСТЕЙ
    ПРИЛОЖЕНИЯ. МАТЕМАТИКО-СТАТИСТИЧЕСКИЕ ТАБЛИЦЫ
    Таблица П.1. Значения функции плотности стандартного нормального закона распределения
    Таблица П.2. Значения функции стандартного нормального распределения
    ИСПОЛЬЗУЕМЫЕ В КНИГЕ ОБОЗНАЧЕНИЯ
    СПИСОК ЛИТЕРАТУРЫ

    Стандартная таблица нормального распределения

    изображения / normal-dist. js

    Это колоколообразная кривая стандартного нормального распределения.
    Это нормальное распределение со средним значением 0 и стандартным отклонением 1.

    Показывает процент населения:

    • между 0 и Z (опция «0 до Z»)
    • меньше Z (опция «До Z»)
    • больше, чем Z (опция «Z и далее»)

    Отображает только значения до 0,01%

    Вы также можете использовать приведенную ниже таблицу. В таблице показана область от 0 до Z.

    Вместо одной ДЛИННОЙ таблицы мы поместили « 0.1 » вниз, а затем « 0.01 «. (Пример использования ниже)

    З 0,00 0,01 0,02 0,03 0,04 0,05 0,06 0,07 0,08 0,09
    0,0 0,0000 0,0040 0,0080 0,0120 0,0160 0,0199 0,0239 0,0279 0,0319 0,0359
    0,1 0,0398 0,0438 0,0478 0,0517 0,0557 0,0596 0,0636 0,0675 0,0714 0,0753
    0,2 0,0793 0,0832 0,0871 0,0910 0,0948 0,0987 0,1026 0,1064 0,1103 0,1141
    0,3 0,1179 0,1217 0,1255 0,1293 0,1331 0,1368 0,1406 0,1443 0,1480 0,1517
    0,4 0,1554 0,1591 0,1628 0,1664 0,1700 0,1736 0,1772 0,1808 0,1844 0,1879
    0,5 0,1915 0,1950 0,1985 0,2019 0,2054 0,2088 0,2123 0,2157 0,2190 0,2224
    0,6 0,2257 0,2291 0,2324 0,2357 0,2389 0,2422 0,2454 0,2486 0,2517 0,2549
    0,7 0,2580 0,2611 0,2642 0,2673 0,2704 0,2734 0,2764 0,2794 0,2823 0,2852
    0,8 0,2881 0,2910 0,2939 0,2967 0,2995 0,3023 0,3051 0,3078 0,3106 0,3133
    0,9 0,3159 0,3186 0,3212 0,3238 0,3264 0,3289 0,3315 0,3340 0,3365 0,3389
    1,0 0,3413 0,3438 0,3461 0,3485 0,3508 0,3531 0,3554 0,3577 0,3599 0,3621
    1. 1 0,3643 0,3665 0,3686 0,3708 0,3729 0,3749 0,3770 0,3790 0,3810 0,3830
    1,2 0,3849 0,3869 0,3888 0,3907 0,3925 0,3944 0,3962 0,3980 0,3997 0,4015
    1,3 0,4032 0,4049 0,4066 0,4082 0,4099 0,4115 0,4131 0,4147 0,4162 0,4177
    1,4 0,4192 0,4207 0,4222 0,4236 0,4251 0,4265 0,4279 0,4292 0,4306 0,4319
    1,5 0,4332 0,4345 0,4357 0,4370 0,4382 0,4394 0,4406 0,4418 0,4429 0,4441
    1,6 0,4452 0,4463 0,4474 0,4484 0,4495 0,4505 0,4515 0,4525 0,4535 0,4545
    1,7 0,4554 0,4564 0,4573 0,4582 0,4591 0,4599 0,4608 0,4616 0,4625 0,4633
    1,8 0,4641 0,4649 0,4656 0,4664 0,4671 0,4678 0,4686 0,4693 0,4699 0,4706
    1,9 0,4713 0,4719 0,4726 0,4732 0,4738 0,4744 0,4750 0,4756 0,4761 0,4767
    2,0 0,4772 0,4778 0,4783 0,4788 0,4793 0,4798 0,4803 0,4808 0,4812 0,4817
    2. 1 0,4821 0,4826 0,4830 0,4834 0,4838 0,4842 0,4846 0,4850 0,4854 0,4857
    2.2 0,4861 0,4864 0,4868 0,4871 0,4875 0,4878 0,4881 0,4884 0,4887 0,4890
    2,3 0,4893 0,4896 0,4898 0,4901 0,4904 0,4906 0,4909 0,4911 0,4913 0,4916
    2,4 0,4918 0,4920 0,4922 0,4925 0,4927 0,4929 0,4931 0,4932 0,4934 0,4936
    2,5 0,4938 0,4940 0,4941 0,4943 0,4945 0,4946 0,4948 0,4949 0,4951 0,4952
    2,6 0,4953 0,4955 0,4956 0,4957 0,4959 0,4960 0,4961 0,4962 0,4963 0,4964
    2,7 0,4965 0,4966 0,4967 0,4968 0,4969 0,4970 0,4971 0,4972 0,4973 0,4974
    2,8 0,4974 0,4975 0,4976 0,4977 0,4977 0,4978 0,4979 0,4979 0,4980 0,4981
    2,9 0,4981 0,4982 0,4982 0,4983 0,4984 0,4984 0,4985 0,4985 0,4986 0,4986
    3,0 0,4987 0,4987 0,4987 0,4988 0,4988 0,4989 0,4989 0,4989 0,4990 0,4990

     

    Пример: Процент населения от 0 до 0,45

    Начните со строки 0,4 и читайте дальше до 0,45: есть значение 0,1736

    А 0,1736 — это 17,36%

    Итак, 17,36% населения находятся в диапазоне от 0 до 0,45 стандартных отклонений от среднего.

    Поскольку кривая симметрична, одну и ту же таблицу можно использовать для значений, идущих в любом направлении, поэтому отрицательный 0,45 также имеет площадь 0,1736

    Пример: Процент населения Z Между −1 и 2

    От от −1 до 0 то же, что и от 0 до +1 :

    В строке для 1,0, первый столбец 1,00, есть значение 0,3413

    От 0 до +2 :

    В строке для 2,0, первый столбец 2,00, есть значение 0,4772

    Добавьте два, чтобы получить сумму от -1 до 2:

    0,3413 + 0,4772 = 0,8185

    И 0,8185 9002 3 равно 81,85%

    Таким образом, 81,85% населения находятся между -1 и +2 стандартных отклонения от среднего.

     

    1486, 1487, 1488, 1489, 1490, 1491, 3846, 3847, 3848, 3849

    Квантиль — это ключ к пониманию распределения вероятностей

    Если вы когда-нибудь чувствовали себя запутанными при использовании распределения вероятностей, эта статья для вас.

    Agnieszka Kujawska, PhD

    ·

    Читать

    Опубликовано в

    ·

    13 мин чтения

    ·

    Июн 26, 2021

    Photo by Joshua Earle on Unsplash

    Вы много раз встречались с распределением вероятностей. Вы знаете, что есть несколько разных типов. Но в глубине души вы чувствуете смущение, когда вам нужно использовать его на практике. В чем, черт возьми, разница между распределением вероятностей и кумулятивным распределением вероятностей? Должен ли я проверять уровень достоверности или альфу по оси X или Y? Если да, то эта статья для вас. В конце концов, вы будете чувствовать себя комфортно, используя распределения вероятностей для дискретных или непрерывных случайных величин. Давайте погрузимся в это!

    В этой статье мы рассмотрим следующие темы:

    1. Функция плотности вероятности (PDF)
    2. Функция массы вероятности (PMF)
    3. Кумулятивное распределение вероятностей (CDF)
      3. 1 Кумулятивное распределение вероятностей для ДИСКРЕТНЫХ случайных величин (CMF) 9000 5 3.2 Кумулятивное распределение вероятностей для НЕПРЕРЫВНЫХ случайных величин (CDF)
    4. Сводка распределений вероятностей
    5. Функция квантилей
    6. Спасибо за чтение и ссылки

    Распределение плотности вероятности нормального распределения — это то, о чем люди чаще всего думают, когда слышат слово «распределение». Он имеет специфическую форму колокола:

    PDF стандартного нормального распределения (нулевое среднее и стандартное отклонение 1). Источник: изображение автора

    Функция плотности вероятности (PDF) сопоставляет значение с его плотностью вероятности [1]. Это понятие похоже на физику, где плотность вещества — это его масса на единицу объема. Например, 1 литр воды весит примерно 1 кг, поэтому плотность воды составляет примерно 1 кг/л или 1000 кг/м³. Аналогично, плотность вероятности измеряет вероятность на единицу х .

    PDF относится к непрерывной случайной величине , что означает, что переменная может принимать любое значение в пределах определенного диапазона действительных чисел. Random показывает неопределенность того, какие значения может принимать переменная. Это дает бесконечное количество возможностей, например 0,1, но также и 0,101, 0,1001 и т. д. Таким образом, вероятность того, что непрерывная случайная величина будет равна заданному значению, равна нулю.

    Вероятность на графике PDF представлена ​​площадью под кривой плотности. Площадь под точкой равна нулю. Вот почему PDF используется для проверки вероятности того, что случайная величина попадает в заданный диапазон значений, а не для принятия какого-либо конкретного значения. Например, какова вероятность того, что мы потеряем деньги, инвестируя в фонд так, что доходность будет отрицательной? Здесь мы рассматриваем все доходы меньше нуля.

    Интуитивно PDF представляет собой линию, описывающую гистограмму. Например, мы хотим разделить 992 участника эксперимента на возрастные группы (0–10, 11–20 и т. д.). Мы подсчитываем, сколько участников попадает в каждую группу, и представляем это в виде столбцов на гистограмме:

    Гистограмма из 992 участников, разделенных на возрастные группы. Данные, полученные из нормального распределения. Изображение автора.

    Насколько высока вероятность того, что человек, которого мы случайно выберем, будет членом данной возрастной группы? Во-первых, мы должны преобразовать распределение частот в распределение вероятностей. Это означает вычисление плотности вероятности на основе количества участников в каждой группе. Так как бары имеют прямоугольную форму и площадь под функцией плотности вероятности всегда равна 1, мы можем использовать упрощенное уравнение:

    Для частот, представленных на предыдущем графике, мы имеем:

    Теперь мы можем построить наши данные, используя плотности вместо подсчетов по оси Y. Красная кривая соединяет расчетные точки и обозначает функцию плотности вероятности:

    PDF-график автора. Красная линия — функция плотности вероятности.

    Но обратите внимание, что я сгенерировал данные для этого графика из нормального распределения. Вот почему PDF и гистограмма так хорошо подходят. PDF имеет «закрытую» форму, что требует предварительного определения распределения и параметров (среднее значение и стандартное отклонение в случае нормального распределения). Гистограмма использует необработанные данные, поэтому она показывает реальное распределение. Это позволяет обнаруживать аномалии, особенно при большом количестве баров.

    Интересуют другие параметры, используемые для описания распределения (математическое ожидание, дисперсия, асимметрия и эксцесс)? Перейти сюда:

    Статистические моменты в интервью по науке о данных

    Основная математика для специалистов по данным, объясненная с нуля

    в направлении datascience.com

    Ключевые моменты, которые следует помнить из приведенного выше анализа:

    • Вероятность — это площадь под вероятностью кривая плотности (PDF).
    • Вероятность того, что непрерывная случайная величина примет заданное значение, равна нулю. Итак, для заданного значения x мы можем проверить только плотность вероятности, что не очень полезно.
    • Поэтому мы ориентируемся на интервалы значений. Это позволяет нам делать вероятностные утверждения о диапазоне значений. Например, есть вероятность 50%, что участнику будет не менее 40 лет.

    Функция массы вероятности (PMF) относится к дискретным случайным величинам. В отличие от непрерывных случайных величин, дискретные случайные величины могут принимать только счетное число дискретных значений, таких как 0, 1, 2,…. Простыми примерами являются бросание игральной кости, подбрасывание монеты или обнаружение мошеннических транзакций (мошенничество либо есть, либо его нет).

    Подобно непрерывным случайным величинам, мы можем создать гистограмму дискретных данных. Но нет необходимости агрегировать значения в интервалы. Рассмотрим сумму бросков пары игральных костей. Количество результатов конечно, так как значения на обоих кубиках от 1 до 6. На графике ниже показан пример гистограммы для 1000 бросков правильной пары игральных костей:

    Гистограмма сумм при 1000-кратном броске правильной пары игральных костей . Изображение автора.

    Оба кубика являются правильными, что означает, что вероятность выпадения каждого числа от 1 до 6 одинакова, равная 1/6. Таким образом, самая популярная сумма равна 7. Как и в случае с непрерывными случайными величинами, мы можем выразить каждый результат как вероятность.

    Если мы бросим пару кубиков, возможны 36 исходов (по 6 вариантов на каждом кубике). Если сумма равна 2, возможна только одна комбинация: (1,1). Таким образом, вероятность получить сумму, равную 2, равна 1/36 = 0,0278. Аналогично для суммы 12, возможно только для (6,6). Точно так же мы можем рассчитать вероятности других возможных исходов. Результаты, представленные на графике, создают функцию массы вероятности (PMF):

    PMF суммы справедливой пары игральных костей. Изображение автора.

    Подводя итог, мы рассмотрели следующие типы графиков:

    • Гистограмма — это график, показывающий, сколько раз каждый диапазон значений появляется в наборе данных. Он не требует каких-либо предположений о распределении, но мы должны заранее указать количество баров. Гистограмма строится из конечного числа выборок. Сумма значений гистограммы для всех баров равна общему количеству выборок.
    • Функция плотности вероятности (PDF) описывает плотность вероятности непрерывных случайных величин . Вероятность на PDF представляет собой площадь под кривой плотности. Поскольку вероятность данного значения равна нулю для непрерывных случайных величин, PDF используется для проверки вероятности того, что переменная попадает в заданный интервал. Вся площадь под PDF равна единице.
    • Функция массы вероятности (PMF) описывает вероятность дискретных случайных величин . Это означает, что переменная может принимать только счетное число дискретных значений, таких как 0, 1, 2 и т. д. Сумма вероятностей всех дискретных значений в PMF равна единице.

    Хотя все они очень полезны и широко используются в отрасли, есть еще одно важное распределение вероятностей — кумулятивная функция распределения (CDF).

    Кумулятивная функция распределения (CDF) случайной величины X описывает вероятность (шансы) того, что X примет значение, равное или меньшее x. Математически это можно выразить так:

    3.1. Кумулятивная функция распределения ДИСКРЕТНОГО распределения вероятностей (CDF или CMF)

    Взяв предыдущий пример с подбрасыванием правильной пары игральных костей, мы можем спросить: какова вероятность того, что сумма двух игральных костей меньше или равна 3? Нам нужно добавить вероятность суммы, равной 2 (0,0278), и вероятность суммы 3 (0,0556), поэтому совокупная вероятность для x = 3 составляет 0,0278 + 0,0556 = 0,0834. Затем мы повторяем процесс добавления для каждого дискретного значения, чтобы получить кумулятивную функцию распределения дискретного распределения вероятностей:

    Кумулятивную функцию вероятности дискретного распределения вероятностей. Изображение автора.

    Как видно на графике, кумулятивная функция вероятности для максимально возможного исхода равна 1. Поскольку сумма двух игральных костей может принимать только целые значения, график можно выразить с помощью столбцов:

    Накопительная функция вероятности дискретное распределение вероятностей. Изображение автора.

    3.2. Кумулятивная функция распределения НЕПРЕРЫВНОГО распределения вероятностей (CDF)

    Идея CDF для непрерывных переменных такая же, как и для дискретных переменных. Ось Y показывает вероятность того, что X примет значения, равные или меньшие, чем x. Отличие в том, что вероятность меняется даже при небольших перемещениях по оси x.
    В примере с групповым возрастом участников кумулятивная функция распределения выглядит следующим образом:

    Кумулятивная функция распределения непрерывного распределения вероятностей (CDF). Изображение автора.

    Графики ниже сравнивают PDF и CDF нормального распределения с нулевым средним и стандартным отклонением, равным единице:

    PDF и CDF нормального распределения N(0,1). Изображение автора.

    Мы можем сделать вывод, что:

    • CDF является неубывающей функцией. Он показывает вероятность того, что переменная равна или меньше x, поэтому она может увеличиваться только с увеличением значения x.
    • Мы можем проверить вероятность по обоим графикам, но использование CDF более просто. CDF показывает вероятность по оси y, а PDF имеет плотность вероятности по оси y. В случае PDF вероятность представляет собой площадь под кривой PDF.
    • Поскольку нормальное распределение симметрично, CDF при x=0 (что означает среднее значение) составляет 0,5.
    • Функция CDF с левой стороны асимптотична к 0 и 1 с правой стороны графика. Точные значения x зависят от типа распределения и параметров (среднее значение и стандартное отклонение для нормального распределения).

    До сих пор мы рассмотрели три способа описания распределения вероятностей: функция плотности вероятности (PDF), функция массы вероятности (PMF) и кумулятивная функция распределения (CDF). Основные различия между PDF и PMF представлены в таблице ниже:

    Основные различия между PDF и PMF. Изображение автора.

    Кумулятивная функция распределения показывает вероятность того, что X примет максимальное значение x. Он суммирует шансы для всех меньших значений и шансов, равных x. Поскольку ось Y представляет собой вероятность, использование CDF часто более просто, чем для PDF.

    На следующей схеме показаны типичные графики каждого распределения, по часовой стрелке и начиная с верхнего левого угла: PDF, PMF, CMF, CDF. Он обобщает высокоуровневую характеристику и описывает отношения между заданными типами функций распределения.

    Сравнение различных типов дистрибутивов. Изображение автора вдохновлено [1,2].

    Как видно выше, существует некоторая связь между различными способами отображения распределения вероятностей.

    • Для непрерывных случайных величин мы можем легко построить PDF и CDF. Область под PDF — это вероятность, поэтому нам нужно интегрировать, чтобы преобразовать PDF в CDF, или дифференцировать, чтобы перейти от CDF к PDF.
    • Для дискретных случайных величин PMF показывает вероятность, а CDF (CMF) — кумулятивную вероятность. Чтобы получить CMF из PMF, мы должны сложить вероятности до заданного x. Чтобы пойти наоборот (от CMF к PMF), мы должны вычислить разницу между шагами.
    • Если мы разделим все значения на набор бинов (см. примеры с гистограммами выше), мы можем перейти от PDF к виду PMF. Он использует диапазон значений/интервалов и может рассматриваться как аппроксимация PDF. Чтобы перейти от дискретного кумулятивного распределения к непрерывной функции, необходима некоторая форма сглаживания. Это можно сделать, предположив, что данные поступают из определенного непрерывного распределения, такого как нормальное или экспоненциальное, и оценив параметры этого распределения. Изменение дискретной и непрерывной случайной величины в обоих направлениях следует рассматривать как аппроксимацию.

    Позвольте представить суперзвезду распределений — функцию квантилей. Это позволяет использовать распределения для многих практических целей, таких как поиск доверительных интервалов и проверка гипотез.

    Математическое определение состоит в том, что квантильная функция является обратной функцией распределения при α. Он определяет значение случайной величины так, что вероятность того, что переменная меньше или равна этому значению, равна заданной вероятности:

    Где F⁻¹(α) обозначает α-квантиль X.

    Сейчас это может показаться немного загадочным, но при ближайшем рассмотрении сомнения развеются. Предположим, что мы хотим проверить 5% общей площади в нижнем хвосте распределения. Мы называем это нижним 5% квантилем X и записываем как F⁻¹(0,05). Квантиль — это распределение вероятностей, разделенное на области с равной вероятностью. Если рассматривать проценты, то сначала делим раздачу на 100 штук. Когда мы смотрим в PDF, 5-й квантиль — это точка, которая отсекает площадь 5% в нижней части распределения:

    Нижний 5% квантиль для нормального распределения N(0,1). Изображение автора.

    Площадь под PDF слева от красной линии составляет ровно 5% от общей площади под кривой. Это подразумевает вероятность 5%. Первым шагом к рисованию красной линии было вычисление, где заканчивается 0,05 общей площади (здесь x=-1,645). Это можно сделать с помощью программного обеспечения (например, функция qnorm() в R или scipy.stats.norm.ppf() в Python) или вручную с использованием z-таблиц (пример здесь).

    Поскольку CDF имеет вероятность (α) по оси y, проще найти это значение здесь:

    Нижний 5% квантиль для нормального распределения N(0,1). Изображение автора.

    Это показывает, насколько полезны графики CDF. Мы можем использовать CDF в обоих направлениях:

    • Если у нас есть значение z (или значение x, значение на оси x), мы можем проверить вероятность того, что X примет значение, равное или меньшее, чем x. Например, какова вероятность того, что средняя продолжительность пребывания клиента в интернет-магазине составляет полчаса или меньше?
    • Если у нас есть вероятность, мы можем проверить значение, которое отсекает область данной альфы. Например, с 90% уверенности, можно сказать, что клиент проводит в интернет-магазинах не менее X часов.

    В приведенном выше примере мы рассмотрели только односторонний 5% квантиль (нижний хвост). Мы можем сделать то же самое для 5% вероятности с двух сторон. Это означает, что мы ищем 5% общей площади под PDF, но разделены на 2,5% нижнего квантиля (слева) и 2,5% верхнего квантиля (на правой стороне графика).

    Двусторонняя вероятность 5%. Изображение автора.

    Таким образом, квантили являются прямой связью между этими графиками.

    Основываясь на графиках, мы можем сказать, что у нас есть 95% уверенность в том, что истинный параметр (среднее значение) находится между -1,96 и 1,96. Или что существует 5% вероятность того, что оно находится за пределами диапазона от -1,96 до 1,96.

    Приведенная выше интерпретация подчеркивает, что:

    • уровень достоверности говорит нам, насколько вероятно рассматриваемое событие или каковы шансы того, что данный параметр находится в заданном диапазоне значений.
    • альфа или уровень значимости — это вероятность. Мы можем проверить это по оси Y на графике CDF. Альфа — это один минус уровень достоверности.

    Несколько замечаний:

    • Обратная функция Φ⁻¹(α) является α-квантилем
    • Когда α мало, квантиль также называется критическим значением 0010 Некоторые квантили имеют специальные имена . Если мы разделим вероятность на 100 частей, мы получим процентили. Мы можем сказать 5-й процентиль вместо 5% квантиля. 4-квантили называются квартилями и делятся на 4 части с разбивкой по значениям 25%, 50% (медиана) и 75%.
    • Для стандартного нормального распределения (нормальное распределение с нулевым средним значением и стандартным отклонением, равным единице N(0,1)), которое симметрично относительно нуля, мы имеем:

    Это доказано на графиках выше, поскольку мы получаем — 1,96 на нижнем оперении и 1,96 на верхнем оперении.

    Используя квантили, PDF, CDF, мы можем ответить на разные вопросы в зависимости от информации, которой мы владеем, например:

    • Учитывая выборочное среднее, каков диапазон значений, содержащих среднее значение генеральной совокупности, в котором мы достаточно уверены? «Разумно» может принимать различные процентные значения и зависит от цели нашего исследования.
    • С какой степенью уверенности можно сказать, что доходность не будет отрицательной?

    Я рад, что вы дочитали до конца этой статьи. Мы рассмотрели различные типы распределений вероятностей: функция плотности вероятности (PDF), функция массы вероятности (PMF) и кумулятивная функция плотности (CDF). Затем мы обсудили функцию количества. Он связывает различные способы описания дистрибутивов (PDF и CDF) и позволяет нам использовать эти дистрибутивы очень практичным образом. Надеюсь, это было увлекательное путешествие для вас.

    Помните, что самый эффективный способ выучить (математические) навыки — это практика . Так что не ждите, пока вы почувствуете себя «готовым», просто возьмите ручку и бумагу (или ваше любимое программное обеспечение) и попробуйте несколько примеров самостоятельно. Держу за тебя пальцы скрещенными.

    Я буду рад услышать ваши мысли и вопросы в разделе комментариев ниже, связавшись со мной напрямую через мой профиль LinkedIn или по телефону [email protected]. До скорой встречи!

    Вам также может понравиться:

    Statistical Moments in Data Science интервью

    Основная математика для специалистов по данным: объяснение с нуля

    в направлении datascience.

    Производная: еще пара-тройка глупых вопросов: marta_inj — LiveJournal

    ?
    Categories:
    • Наука
    • Образование
    • Cancel

    Все знают, что на ноль делить нельзя. Но почему?
    Потому что не имеет смысла – ответствуют нам школьные учителя.
    Хотя вроде бы смысл есть. Любое число, деленное на ноль, должно быть равно бесконечности.
    А если ноль поделить на ноль? Единице, наверное.
    Вроде логично…

    Но тут выплывает производная.

    Производной функции y = f(x) в точке x называется предел (если он существует и конечен) отношения приращения функции к приращению аргумента при условии, что последнее стремится к нулю.

    Ага, здесь у нас как раз знаменатель стремится к нулю, но сама функция почему-то не к бесконечности стремится, а к другим значениям.

    Например, известно, что производная константы (например, числа) равна нулю. Но что является функцией, а что аргументом в этом случае?..

    Производная указывает нам на скорость изменения функции- так написано во многих учебниках. Число вроде бы не меняется, значит, производная равна нулю… Но всплывает вопрос – а с чего мы взяли, что число не меняется? Потому что это аксиома?
    Если взять единицу длины – в виде палочки и веревочки – можем мы эту единицу длины изменить? Можем растянуть? Можем. Можем сжать? Можем. Выходит, на физическом плане неизменность числа никак не гарантируется, она – эта неизменность — существует лишь теоретически, в виртуальных математических построениях.

    А вот производная икс квадрат – это два икс. Почему?!

    Вот пример (из лекций по физике Цаплина):

    Путь s — скалярная величина, равная полной длине отрезка траектории, пройденной МТ за время движения.

    Уравнение пути

    s = s(t) .

    Перемещение — вектор, проведенный из начального положения (точка А) в конечное (точка В),

    Численные значения и в случае прямолинейного движения совпадают. В случае же криволинейного движения они совпадают только в пределе, т.е. для бесконечно малого перемещения
    .

    Здесь нападает ступор… В пределе совпадают, а по факту нет. Что это значит? Что в микромире путь и перемещение совпадают? А в макромире – нет?..

    Интересное из Википедии:
    • Определение через колебания: функция непрерывна в точке, если её колебание в данной точке равно нулю.
    …..
    Если функция имеет разрыв в данной точке (то есть предел функции в данной точке отсутствует или не совпадает со значением функции в данной точке), то для числовых функций возникает два возможных варианта, связанных с существованием у числовых функций односторонних пределов:
    • если оба односторонних предела существуют и конечны, то такую точку называют точкой разрыва первого рода. К точкам разрыва первого рода относят устранимые разрывы и скачки.
    • если хотя бы один из односторонних пределов не существует или не является конечной величиной, то такую точку называют точкой разрыва второго рода. К точкам разрыва второго рода относят полюса и точки существенного разрыва.

    Tags: Вопросы к науке, Смысл математики

    Subscribe

    • Еще раз про свойства сознания

      Для того, чтобы мы могли найти отличия на картинках, обе эти картинки должны быть рядом в одном поле зрения. Если мы попробуем расположить эти…

    • «Потерянные» участки тела

      Описываю гипотезу, почему мышечный зажим одновременно ускользает от сознания и захватывает соседние и даже отдаленные мышцы, создавая напряжение в…

    • Поговорим про мышечный зажим

      Мышечный зажим — это хронически напряженная мышца, которую мы не замечаем. Не замечаем потому, что её состояние не меняется, а значит, из сферы…

    Photo

    Hint http://pics. livejournal.com/igrick/pic/000r1edq

    2 = \displaystyle {\underbrace {x+x+x+ \ldots +x}_{x \ times}}$

    Теперь пусть

    $ f(x) = \displaystyle {\underbrace {x+x+x+ \ ldots +x}_{x \ times}}$

    затем

    $ f'(x) = \dfrac{d}{dx} \left( \displaystyle {\underbrace {x+x+x+ \ldots + x} _ {x \ times}} \right) $

    $ f ‘(x) = \ displaystyle {\ underbrace {\ dfrac {d} {dx} x + \ dfrac {d} {dx} x + \ ldots + \dfrac{d}{dx} x}_{x \ times}}$

    $ f'(x)=\displaystyle {\underbrace {1 + 1 + \ldots + 1}_{x \ times}} $ 92 = х[х]+х\{х\}$

  • $d/dx {[x²]}= d/dx \left( {x[x] +x \{x\} }\right)$
  • (дифференциация по частям)
  • $= 1\cdot [x]+x \cdot [x]’+ 1\cdot \{x\} + x \cdot \{x\}’$
  • , так как $d/dx (x)=x’=1$ и [x]’ & {x}’ представляют дифференцирование каждого по x.
  • $=[x]+\{x\}+x \left({[x]’+\{x\}’ }\right)$
  • $=x+x (x’)$
  • $=х+х=2х$
    • Yesmanapple прислал свое мнение об этой статье. Взгляни.
    • wnoise предложил эту ссылку:
    • Умножение не повторяется Сложение.
    • Функция наибольшего целого числа

    Предыдущий пост: Решение головоломной задачи Рамануджана

    Следующий пост: Формула Вейна и законы Вейна

    Видео-вопрос: Дифференцирование функции по отношению к другой функции с использованием параметрического дифференцирования

    Используя параметрическое дифференцирование, определите производную от 5𝑥³ + 𝑥² − 2 относительно 4𝑥² + 8.

    Стенограмма видео

    Используя параметрическое дифференцирование, определите производную пяти 𝑥 в кубе плюс 𝑥 в квадрате минус два относительно четырех 𝑥 в квадрате плюс восемь.

    Напомним, что если бы у нас было 𝑦 равно некоторой функции 𝑓 от 𝑥 и 𝑧 равно некоторой функции 𝑔 от 𝑥, то по цепному правилу. Производная от 𝑦 по 𝑧 равна производной от 𝑦 по 𝑥, умноженной на производную от 𝑥 по 𝑧. Затем, установив 𝑥 равным обратному 𝑔 𝑧, мы получим, что производная 𝑥 по 𝑧 равна единице, деленной на производную 𝑧 по 𝑥, используя нашу теорему об обратной функции. Используя это, мы можем вычислить производную от 𝑦 по 𝑧, сначала вычислив производную от 𝑦 по 𝑥, а затем разделив ее на производную от 𝑧 по 𝑥.

    Вопрос требует, чтобы мы вычислили производную пяти 𝑥 в кубе плюс 𝑥 в квадрате минус два. И он хочет, чтобы мы сделали это относительно четырех 𝑥 в квадрате плюс восемь. Итак, если мы установим 𝑦 равным пяти 𝑥 в кубе плюс 𝑥 в квадрате минус два и 𝑧 равным четырем 𝑥 в квадрате плюс восемь, то d𝑦 d𝑧 — это то, что вопрос хочет, чтобы мы вычислили. Это производная от пяти 𝑥 в кубе плюс 𝑥 в квадрате минус два относительно четырех 𝑥 в квадрате плюс восемь. И мы имеем, что это равно производной от 𝑦 по 𝑥, деленной на производную от 𝑧 по 𝑥.

    Теперь мы можем вычислить и то, и другое. Производная от 𝑦 по 𝑥 равна производной пяти 𝑥 в кубе плюс 𝑥 в квадрате минус два по 𝑥. Мы можем дифференцировать этот термин за термином. Чтобы дифференцировать пять 𝑥 в кубе, мы умножаем его на показатель степени, а затем уменьшаем показатель степени на единицу. Это дает нам 15𝑥 в квадрате. Мы делаем то же самое, чтобы дифференцировать 𝑥 в квадрате. Мы умножаем на показатель степени двух, а затем уменьшаем показатель степени на единицу, что дает нам два 𝑥.

    Наконец, производная любой константы просто равна нулю. Таким образом, производная от отрицательных двух просто равна нулю. Итак, мы показали, что d𝑦 d𝑥 равно 15𝑥 в квадрате плюс два 𝑥. Мы можем сделать то же самое, чтобы вычислить производную от 𝑧 по 𝑥. Это производная от четырех 𝑥 в квадрате плюс восемь относительно 𝑥.

    Чтобы дифференцировать четыре 𝑥 в квадрате, мы умножаем на показатель степени двух, а затем уменьшаем показатель степени на единицу, что дает нам восемь 𝑥. А затем, чтобы продифференцировать постоянную восемь, мы просто получаем ноль, что дает нам, что производная от 𝑧 по 𝑥 равна восьми 𝑥. Подстановка их в наше уравнение дает нам, что производная от 𝑦 по отношению к 𝑧 равна 15𝑥 в квадрате плюс два 𝑥, все деленные на восемь 𝑥.

    Решите уравнение x 2 3x 2 0: Решите уравнение: а) x^2 + 3x + 2 = 0; б) x^2 – 4x – 5 = 0; в) x^2 – 7x + 12 = 0 ; г) x^2 + 5x – 6 = 0.

    2
    Функция — Квадрат x
    ctg(x)
    Функция — Котангенс от x
    arcctg(x)
    Функция — Арккотангенс от x
    arcctgh(x)
    Функция — Гиперболический арккотангенс от x
    tg(x)
    Функция — Тангенс от x
    tgh(x)
    Функция — Тангенс гиперболический от x
    cbrt(x)
    Функция — кубический корень из x
    gamma(x)
    Гамма-функция
    LambertW(x)
    Функция Ламберта
    x! или factorial(x)
    Факториал от x
    DiracDelta(x)
    Дельта-функция Дирака
    Heaviside(x)
    Функция Хевисайда
    Интегральные функции:
    Si(x)
    Интегральный синус от x
    Ci(x)
    Интегральный косинус от x
    Shi(x)
    Интегральный гиперболический синус от x
    Chi(x)
    Интегральный гиперболический косинус от x
    В выражениях можно применять следующие операции:
    Действительные числа
    вводить в виде 7. 3
    — возведение в степень
    x + 7
    — сложение
    x — 6
    — вычитание
    15/7
    — дробь

    Другие функции:
    asec(x)
    Функция — арксеканс от x
    acsc(x)
    Функция — арккосеканс от x
    sec(x)
    Функция — секанс от x
    csc(x)
    Функция — косеканс от x
    floor(x)
    Функция — округление x в меньшую сторону (пример floor(4.5)==4.0)
    ceiling(x)
    Функция — округление x в большую сторону (пример ceiling(4.5)==5.0)
    sign(x)
    Функция — Знак x
    erf(x)
    Функция ошибок (или интеграл вероятности)
    laplace(x)
    Функция Лапласа
    asech(x)
    Функция — гиперболический арксеканс от x
    csch(x)
    Функция — гиперболический косеканс от x
    sech(x)
    Функция — гиперболический секанс от x
    acsch(x)
    Функция — гиперболический арккосеканс от x

    Постоянные:
    pi
    Число «Пи», которое примерно равно ~3.

    Порядок базисного минора матрицы онлайн: Онлайн калькулятор. Ранг матрицы

    Решение высшей математики онлайн

    ‹— Назад

    В этом разделе рассмотрим еще одну важную числовую характиристику матрицы, связанную с тем, насколько ее строки (столбцы) зависят друг от друга.

            Определение 14.10   Пусть дана матрица размеров и число , не превосходящее наименьшего из чисел и : . Выберем произвольно строк матрицы и столбцов (номера строк могут отличаться от номеров столбцов). Определитель матрицы, составленной из элементов, стоящих на пересечении выбранных строк и столбцов, называется минором порядка матрицы .         

            Пример 14.9   Пусть .

    Минором первого порядка является любой элемент матрицы. Так 2, ,  — миноры первого порядка.

    Миноры второго порядка:

    1. возьмем строки 1, 2, столбцы 1, 2, получим минор ;
    2. возьмем строки 1, 3, столбцы 2, 4, получим минор ;
    3. возьмем строки 2, 3, столбцы 1, 4, получим минор

    Миноры третьего порядка:

    строки здесь можно выбрать только одним способом,

    1. возьмем столбцы 1, 3, 4, получим минор ;
    2. возьмем столбцы 1, 2, 3, получим минор .

            

            Предложение 14.23   Если все миноры матрицы порядка равны нулю, то все миноры порядка , если такие существуют, тоже равны нулю.

            Доказательство.     Возьмем произвольный минор порядка . Это определитель матрицы порядка . Разложим его по первой строке. Тогда в каждом слагаемом разложения один из множителей будет являться минором порядка исходной матрицы. По условию миноры порядка равны нулю. Поэтому и минор порядка будет равен нулю.     

            Определение 14.11   Рангом матрицы называется наибольший из порядков миноров матрицы , отличных от нуля. Ранг нулевой матрицы считается равным нулю.         

    Единое, стандартное, обозначение ранга матрицы отсутствует. Следуя учебнику  [1], мы будем обозначать его .

            Пример 14. 10   Матрица примера 14.9 имеет ранг 3, так как есть минор третьего порядка, отличный от нуля, а миноров четвертого порядка нет.

    Ранг матрицы равен 1, так как есть ненулевой минор первого порядка (элемент матрицы ), а все миноры второго порядка равны нулю.

    Ранг невырожденной квадратной матрицы порядка равен , так как ее определитель является минором порядка и у невырожденной матрицы отличен от нуля.         

            Предложение 14.24   При транспонировании матрицы ее ранг не меняется, то есть .

            Доказательство.     Транспонированный минор исходной матрицы будет являться минором транспонированной матрицы , и наоборот, любой минор является транспонированным минором исходной матрицы . При транспонировании определитель (минор) не меняется ( предложение 14.6). Поэтому если все миноры порядка в исходной матрице равны нулю, то все миноры того же порядка в тоже равны нулю. Если же минор порядка в исходной матрице отличен от нуля, то в есть минор того же порядка, отличный от нуля. Следовательно, .     

            Определение 14.12   Пусть ранг матрицы равен . Тогда любой минор порядка , отличный от нуля, называется базисным минором.         

            Пример 14.11   Пусть . Определитель матрицы равен нулю, так как третья строка равна сумме первых двух. Минор второго порядка, расположенный в первых двух строках и первых двух столбцах, равен . Следовательно, ранг матрицы равен двум, и рассмотренный минор является базисным.

    Базисным минором является также минор, расположенный, скажем, в первой и третьей строках, первом и третьем столбцах: . Базисным будет минор во второй и третьей строках, первом и третьем столбцах: .

    Минор в первой и второй строках, втором и третьем столбцах равен нулю и поэтому не будет базисным. Читатель может самостоятельно проверить, какие еще миноры второго порядка будут базисными, а какие нет.         

    Так как столбцы (строки) матрицы можно складывать, умножать на числа, образовывать линейные комбинации, то можно ввести определения линейной зависимости и линейной независимости системы столбцов (строк) матрицы. Эти определения аналогичны таким же определениям 10.14, 10.15 для векторов.

            Определение 14.13   Система столбцов (строк) называется линейно зависимой, если существует такой набор коэффициентов, из которых хотя бы один отличен от нуля, что линейная комбинация столбцов (строк) с этими коэффициентами будет равна нулю.         

            Определение 14.14   Система столбцов (строк) является линейно независимой, если из равенства нулю линейной комбинации этих столбцов (строк) следует, что все коэффициенты этой линейной комбинации равны нулю.         

    Верно также следующеее предложение, аналогичное предложению 10.6.

            Предложение 14.25   Система столбцов (строк) является линейно зависимой тогда и только тогда, когда один из столбцов (одна из строк) является линейной комбинацией других столбцов (строк) этой системы.    

    Сформулируем теорему, которая называется теорема о базисном миноре.

            Теорема 14.2   Любой столбец матрицы является линейной комбинацией столбцов, проходящих через базисный минор.     

    Доказательство можно найти в учебниках по линейной алгебре, например, в [1], [3].

            Предложение 14.26   Ранг матрицы равен максимальному числу ее столбцов, образующих линейно независимую систему.

            Доказательство.     Пусть ранг матрицы равен . Возьмем столбцы, проходящие через базисный минор. Предположим, что эти столбцы образуют линейно зависимую систему. Тогда один из столбцов является линейной комбинацией других. Поэтому в базисном миноре один столбец будет линейной комбинацией других столбцов. По предложениям 14.15 и 14.18 этот базисный минор должен быть равен нулю, что противоречит определению базисного минора. Следовательно, предположение о том, что столбцы, проходящие через базисный минор, линейно зависимы, не верно. Итак, максимальное число столбцов, образующих линейно независимую систему, больше либо равно .

    Предположим, что столбцов образуют линейно независимую систему. Составим из них матрицу . Все миноры матрицы являются минорами матрицы . Поэтому базисный минор матрицы имеет порядок не больше . По теореме о базисном миноре, столбец, не проходящий через базисный минор матрицы , является линейной комбинацией столбцов, проходящих через базисный минор, то есть столбцы матрицы образуют линейно зависимую систему. Это противоречит выбору столбцов, образующих матрицу . Следовательно, максимальное число столбцов, образующих линейно независимую систему, не может быть больше . Значит, оно равно , что и утверждалось.     

            Предложение 14.27   Ранг матрицы равен максимальному числу ее строк, образующих линейно независимую систему.

            Доказательство.     По предложению 14.24 ранг матрицы при транспонировании не меняется. Строки матрицы становятся ее столбцами. Максимальное число новых столбцов транспонированной матрицы, (бывших строк исходной) образующих линейно независимую систему, равно рангу матрицы.     

            Предложение 14.28   Если определитель матрицы равен нулю, то один из его столбцов (одна из строк) является линейной комбинацией остальных столбцов (строк).

            Доказательство.     Пусть порядок матрицы равен . Определитель является единственным минором квадратной матрицы, имеющим порядок . Так как он равен нулю, то . Следовательно, система из столбцов (строк) является линейно зависимой, то есть один из столбцов (одна из строк) является линейной комбинацией остальных.     

    Результаты предложений 14.15, 14.18 и 14.28 дают следующую теорему.

            Теорема 14.3   Определитель матрицы равен нулю тогда и только тогда, когда один из ее столбцов (одна из строк) является линейной комбинацией остальных столбцов (строк).    

    Нахождение ранга матрицы с помощью вычисления всех ее миноров требует слишком большой вычислительной работы. (Читатель может проверить, что в квадратной матрице четвертого порядка 36 миноров второго порядка.) Поэтому для нахождения ранга применяется другой алгоритм. Для его описания потребуется ряд дополнительных сведений.

            Определение 14.15   Назовем элементарными преобразованиями матриц следующие действия над ними:

    1) перестановка строк или столбцов;
    2) умножение строки или столбца на число отличное от нуля;
    3) добавление к одной из строк другой строки, умноженной на число или добавление к одному из столбцов другого столбца, умноженного на число.
            

            Предложение 14.29   При элементарных преобразованиях ранг матрицы не меняется.

            Доказательство.     Пусть ранг матрицы равен ,  — матрица, получившаяся в результате выполнения элементарного преобразования.

    Рассмотрим перестановку строк. Пусть  — минор матрицы , тогда в матрице есть минор , который или совпадает с , или отличается от него перестановкой строк. И наоборот, любому минору матрицы можно сопоставить минор матрицы или совпадающий с , или отличающийся от него порядком строк. Поэтому из того, что в матрице все миноры порядка равны нулю, следует, что в матрице тоже все миноры этого порядка равны нулю. И так как в матрице есть минор порядка , отличный от нуля, то и в матрице тоже есть минор порядка , отличный от нуля, то есть .

    Рассмотрим умножение строки на число , отличное от нуля. Минору из матрицы соответствует минор из матрицы или совпадающий с , или отличающийся от него только одной строкой, которая получается из строки минора умножением на число, отличное от нуля. В последнем случае . Во всех случаях или и одновременно равны нулю, или одновременно отличны от нуля. Следовательно, .

    Пусть к -ой строке матрицы прибавлена ее -ая строка, умноженная на число . Рассмотрим миноры порядка в матрице . Если через минор не проходит -ая строка, то он совпадает с минором , расположенным в тех же строках и столбцах в матрице , и следовательно, равен нулю.

    Если через минор проходят и -ая и -ая строки, то он получается из минора , расположенного в тех же строках и столбцах матрицы , прибавлением к -ой строке минора -ой строки, умноженной на . По свойству определителя . Следовательно, .

    Пусть через минор проходит -ая строка и не проходит -ая. Тогда отличается от -ой строкой. Эта строка в является строкой , к которой добавлены элементы -ой строки, умноженные на . По свойствам определителей , где  — минор порядка матрицы , стоящий в -ой строке и в тех же строках, что и минор , исключая -ую, а знак » » связан с возможным изменением порядка строк. Так как все миноры порядка в матрице равны нулю, то .

    Итак, в матрице все миноры порядка равны нулю. Следовательно, , то есть при выполнении элементарного преобразования третьего типа ранг не может повыситься. Предположим, что , и . Тогда в матрице к -ой строке прибавим -ую строку, умноженную на число . В результате получим исходную матрицу . По только что доказанному . Получили противоречие: . Предположение не верно, следовательно, .     

    Алгоритм вычисления ранга матрицы похож на алгоритм вычисления определителя и заключается в том, что с помощью элементарных преобразований матрица приводится к простому виду, для которого найти ранг не представляет труда. Так как при каждом преобразовании ранг не менялся, то, вычислив ранг преобразованной матрицы, мы тем самым находим ранг исходной матрицы.

    Алгоритм нахождения ранга матрицы.

    Пусть требуется вычислить ранг матрицы размеров . Если матрица нулевая, то по определению . В противном случае с помощью перестановки строк и столбцов матрицы добиваемся того, чтобы в левом верхнем углу матрицы стоял ненулевой элемент. Итак, считаем, что .

    Первую строку оставляем без изменений. Ко второй строке прибавляем первую, умноженную на число . В результате вторая строка принимает вид

    Затем к третьей строке прибавляем первую строку, умноженную на число . В результате третья строка принимает вид

    Процесс продолжаем до тех пор, пока не получим нуль на первом месте в последней строке.

    Преобразованная матрица имеет вид

    Если все строки, начиная со второй, в полученной матрице нулевые, то ее ранг равен 1, так как есть минор первого порядка, отличный от нуля . В противном случае перестановкой строк и столбцов матрицы с номерами, большими единицы, добиваемся, чтобы второй элемент второй строки был отличен от нуля. Итак, считаем, что .

    Первую и вторую строки оставляем без изменений. К третьей строке прибавляем вторую, умноженную на число . В результате получим, что второй элемент третьей строки равен нулю. Затем к четвертой строке прибавляем вторую, умноженную на число , и т.д. В результате получаем матрицу

    Если все строки, начиная с третьей, нулевые, то , так как минор . В противном случае перестановкой строк и столбцов с номерами, большими двух, добиваемся, чтобы третий элемент третьей строки был отличен от нуля. Далее, добавлением третьей строки, умноженной на соответствующие числа, к строкам с большими номерами получаем нули в третьем столбце, начиная с четвертого элемента, и т.д.

    На каком-то этапе мы придем к матрице, у которой все строки, начиная с -ой , равны нулю (или отсутствуют при ), а минор в первых строках и первых столбцах является определителем треугольной матрицы с ненулевыми элементами на диагонали. Ранг такой матрицы равен . Следовательно, .     

            Замечание 14.15   В предложенном алгоритме нахождения ранга матрицы все вычисления должны производиться без округлений. Сколь угодно малое изменение хотя бы в одном из элементов промежуточных матриц может привести к тому, что полученный ответ будет отличаться от ранга исходной матрицы на несколько единиц.         

            Замечание 14.16   Если в исходной матрице элементы были целыми числами, то и вычисления удобно производить без использования дробей. Поэтому на каждом этапе целесообразно умножать строки на такие числа, чтобы при вычислениях дроби не возникали.         

            Пример 14.12   Найдите ранг матрицы .

    Решение. Первую строку оставляем без изменений. Чтобы избежать появления дробей, умножим вторую, третью и четвертую строки на 2:

    Первую строку умножим на и прибавим ко второй. Получим строку . Первую строку умножим на и прибавим к третьей. Получим строку . Первую строку умножим на и прибавим к четвертой. Получим строку . В итоге имеем матрицу

    Вторую строку оставляем без изменений. К третьей строке прибавляем вторую, умноженную на 2. Получим строку . К четвертой строке прибавляем вторую. Получим нулевую строку. Преобразованная матрица имеет вид

    Поменяем местами третий и четвертый столбцы:

    Базисный минор матрицы стоит в первых трех столбцах и первых трех строках, . Следовательно, .         

            Замечание 14.17   В приведенном примере вычисления были бы проще, если сначала четвертый столбец сделать первым и четвертую строку сделать первой. Но для того, чтобы догадаться об этом, нужно анализировать вопросы делимости чисел, что достаточно сложно описать в алгоритме, пригодном для всех случаев.         

    Математика, вышка, высшая математика, математика онлайн, вышка онлайн, онлайн математика, онлайн решение математики, ход решения, процес решения, решение, задачи, задачи по математике, математические задачи, решение математики онлайн, решение математики online, online решение математики, решение высшей математики, решение высшей математики онлайн, матрицы, решение матриц онлайн, векторная алгебра онлайн, решение векторов онлайн, система линейных уравнений, метод Крамера, метод Гаусса, метод обратной матрицы, уравнения, системы уравнений, производные, пределы, интегралы, функция, неопределенный интеграл, определенный интеграл, решение интегралов, вычисление интегралов, решение производных, интегралы онлайн, производные онлайн, пределы онлайн, предел функции, предел последовательности, высшие производные, производная неявной функции

    Ранг матрицы. Лекция 2.2 — презентация онлайн

    Похожие презентации:

    Ранг матрицы

    Определители матриц. Обратная матрица, ранг матрицы

    Обратная матрица

    Линейная алгебра. Ранг матрицы. (Тема 2)

    Ранг матрицы. Собственные числа и собственные векторы

    Системы линейных уравнений. Ранг матрицы

    Матрицы. Элементарные преобразования и действия над матрицами

    Метод Гаусса решения систем линейных уравнений. Ранг матрицы. Исследование систем линейных уравнений

    Линейная алгебра. Ранг матрицы. Метод Гаусса решения систем линейных уравнений. Лекция 5

    Матрицы. Операции над матрицами. Элементарные преобразования. Приведение к ступенчатому виду. Ранг матрицы

    *Лекция 2.2
    © материалы подготовлены к.ф.-м.н., доц. Н.А. Фоменко
    Ранг матрицы.
    Определение. В матрице порядка m n минор порядка r
    называется базисным, если он не равен нулю, а все
    миноры порядка r+1 и выше равны нулю, или не
    существуют вовсе, т.е. r совпадает с меньшим из чисел m
    или n.
    Замечание. В матрице может быть несколько
    различных базисных миноров, имеющих одинаковый
    порядок.
    Определение. Порядок базисного минора матрицы
    называется рангом матрицы и обозначается Rg А.
    Замечание.
    Очень
    важным
    свойством
    элементарных преобразований матриц является то, что
    они не изменяют ранг матрицы.
    © материалы подготовлены к.ф.-м.н., доц. Н.А. Фоменко
    Определение. Матрицы, полученные в результате
    элементарного
    преобразования,
    называются
    эквивалентными.
    Замечание. Надо отметить, что равные матрицы и
    эвивалентные матрицы — понятия совершенно различные.
    Теорема. Наибольшее число линейно независимых
    столбцов в матрице равно числу линейно независимых
    строк. Более того, это число равно рангу матрицы А.
    Т.к. элементарные преобразования не изменяют ранг
    матрицы, то можно существенно упростить процесс
    нахождения ранга матрицы.
    Замечание.
    Ранг
    ступенчатой
    количеству ее ненулевых строк.
    матрицы,
    равен
    © материалы подготовлены к.ф.-м.н., доц. Н.А. Фоменко
    Пример. Найти ранг матрицы
    Решение. С помощью элементарных преобразований
    приведем матрицу A к трапецеидальному виду и первым
    шагом поменяем первую и вторую строчки местами:
    1 2 2 0
    3 1 5 2
    1 3 1
    2
    8 1 13 4
    3
    1 2
    1 l2 3l1 0 5
    ~
    7 l3 l1 0 5
    4 l4 8l1 0 15
    2 0 3
    1 2 10
    1 2 10 l3 l2
    3 4 28 l4 3l2
    © материалы подготовлены к. ф.-м.н., доц. Н.А. Фоменко
    1
    0
    0
    0
    2 2
    0
    3
    3
    1 2 2 0
    5 1 2 10
    ~ 0 5 1 2 10
    0 0
    0
    0
    0 0 0 10 2
    0 0 10 2
    Ранг последней матрицы, являющейся трапецеидальной,
    равен 3; следовательно, rg A = 3.
    Из определения ранга следует, что матрица
    является
    невырожденной в том и только в том случае, если rgА = n.
    © материалы подготовлены к.ф.-м.н., доц. Н.А. Фоменко
    Решение произвольных систем линейных уравнений.
    Определение. Система m уравнений с n неизвестными
    в общем виде записывается следующим образом:
    (1)
    a11 x1 a12 x2 … a1n xn b1
    a x a x … a x b
    21 1 22 2
    2n n
    2
    ………………………………………..
    Определение. Если
    система имеет хотя бы одно решение, то
    1 x1 am 2 x2 … amn xn bm
    она называется amсовместной.
    Если система не имеет ни
    одного решения, то она называется несовместной.
    Определение. Если b1, b2, …,bm = 0, то система
    называется однородной. Однородная система всегда
    совместна, т.к. всегда имеет нулевое решение .
    © материалы подготовлены к.ф.-м.н., доц. Н.А. Фоменко
    Теорема Кронекера – Капелли.
    Теорема: Система совместна тогда и только тогда,
    когда ранг матрицы системы равен рангу расширенной
    матрицы.
    RgA = Rg
    Доказательство:
    Очевидно, что система (1) может быть записана в виде:
    a11
    a12
    a1n b1
    a
    a
    a
    b2
    21
    22
    2n
    x1
    x
    … xn
    … 2 …
    … …
    a
    a
    a
    m1
    m2
    mn bm
    © материалы подготовлены к.ф.-м.н., доц. Н.А. Фоменко
    1) Если решение существует, то столбец свободных членов
    есть линейная комбинация столбцов матрицы А, а значит
    добавление этого столбца в матрицу, т.е. переход А
    не
    изменяют ранга.
    2) Если RgA = Rg , то это означает, что они имеют один и тот
    же базисный минор. Столбец свободных членов – линейная
    комбинация столбцов базисного минора, т.е. верна запись,
    приведенная выше.
    Метод Гаусса.
    При решении методом Гаусса расширенную матрицу системы (1)
    элементарными преобразованиями приводят к треугольному
    виду.
    Если то система решений не имеет.
    Если то система имеет единственное решение.
    Если то система имеет множество решений.
    © материалы подготовлены к.ф.-м.н., доц. Н.А. Фоменко
    Пример. Решить систему методом Гаусса
    Решение.
    1 4 7 3
    1 4 7 3
    0
    10
    19
    8
    ~
    0
    10
    19
    8
    .
    0 7 13 5 10l 7l 0 0 3 6
    2
    3
    RgA = Rg =3, следовательно система совместна, и так как
    ранг совпадает с количеством неизвестных, то система
    имеет единственное решение.
    © материалы подготовлены к.ф.-м.н., доц. Н.А. Фоменко
    Последней ступенчатой матрице, соответствует
    следующая СЛАУ, равносильная исходной системе:
    Из последнего уравнения находим , подставив его во
    второе уравнение, найдем
    и , наконец, подставив
    найденные и в первое уравнение, найдем :
    Следует иметь в виду, что при решении СЛАУ методом
    Гаусса перестановка столбцов приводит к перенумерации
    неизвестных.
    © материалы подготовлены к.ф.-м.н., доц. Н.А. Фоменко
    Пример . Решить систем уравнений:
    x1 2x 2 3x 3 x 4 4x 5 1,
    a) 3×1 4x 2 x 3 2x 4 2,
    2x
    3x 3 x 4 x 5 6;
    1
    Решение.
    1 2 3 1 4 1
    3
    4
    1
    2
    0
    2
    l2 3l1 ~
    2 0 3 1 1 6 l 2l
    1
    3
    1 2 3 1 4 1
    0
    10
    10
    5
    12
    5
    ~
    0 4 3 1 7 4 5l 2l
    2
    3
    1 2 3 1 4 1
    ~ 0 10 10 5 12 5
    0 0
    5 5 11 30
    следовательно система совместна и имеет множество решений.
    Замечание.Неизвестные базисные,
    свободных неизвестных равно , где )
    свободные (количество
    © материалы подготовлены к.ф.-м.н., доц. Н.А. Фоменко
    Перенесем свободные неизвестные в правую часть:
    Степень свободы системы равна двум, значит решение
    системы выразится через два параметра. Положив и решив
    систему из трех уравнений с неизвестными найдем
    где
    произвольные числа.
    © материалы подготовлены к.ф.-м.н., доц. Н.А. Фоменко
    Запишем общее решение системы
    19
    19
    6
    c
    c
    1
    2
    6
    1
    5
    5
    x1
    1
    17
    17
    11
    x 11 1
    c
    c
    2
    1
    2
    2 2
    5
    5 2
    2
    X x 3
    c1 c2
    11
    1
    11
    6 c c 6
    1
    2
    x
    4
    5
    0
    5
    1
    x
    0
    0
    5
    c1
    0
    c2
    1
    © материалы подготовлены к. ф.-м.н., доц. Н.А. Фоменко
    Пример . Решить систему уравнений:
    Решение.
    в результате преобразований появилась
    следовательно, система несовместна.
    строка
    © материалы подготовлены к.ф.-м.н., доц. Н.А. Фоменко

    English     Русский Правила

    Минор матрицы – формула, определение, примеры

    Минор матрицы относится к каждому элементу матрицы и равен части матрицы, оставшейся после исключения строки и столбца, содержащего этот конкретный элемент. Новая матрица, образованная минорами каждого элемента данной матрицы, называется минором матрицы.

    Минор матрицы широко используется при нахождении ее определителя, сопряженной и обратной матрицы. Давайте узнаем больше о миноре матрицы в приведенном ниже содержании.

    1. Что такое минор матрицы?
    2. Как найти минор матрицы?
    3. Применение минора матрицы
    4. Решенные примеры на миноре матрицы
    5. Практические вопросы
    6. Часто задаваемые вопросы о миноре матрицы

    Что такое минор матрицы?

    Минор матрицы для определенного элемента в матрице определяется как матрица, полученная после удаления строки и столбца матрицы, в которой находится этот конкретный элемент. Здесь минор элемента \(a_{ij}\) обозначается как \(M_{ij}\). Например, для данной матрицы A минор \(a_{12}\) является частью матрицы после исключения первой строки и второго столбца матрицы. \(A = \left[\begin{массив}{ccc}
    а_{11} и а_{12} и а_{13} \\
    а_{21} и а_{22} и а_{23} \\
    а_{31} и а_{32} и а_{33}
    \end{array}\right] \)

    Минор элемента \(a_{12}\) выглядит следующим образом.

    \(M_{12} = \left[\begin{array}{ccc} a_{21} & a_{23} \\
    а_{31} и а_{33}
    \end{array}\right] \)

    Аналогично, мы можем взять миноры матрицы и сформировать минорную матрицу M данной матрицы A как:

    \(M = \left[\begin{массив}{ccc}
    М_{11} и М_{12} и М_{13} \\
    М_{21} и М_{22} и М_{23} \\
    М_{31} и М_{32} и М_{33}
    \конец{массив}\справа] \)

    Как найти минор матрицы?

    Есть три простых шага, чтобы найти минор матрицы.

    • Сначала идентифицируйте и исключите строку и столбец, содержащие определенный элемент в матрице.
    • В качестве второго шага сформируйте новую меньшую матрицу из оставшихся элементов, чтобы представить минор определенного элемента матрицы.
    • Наконец, найдите определитель минора каждого элемента матрицы и сформируйте новую матрицу, содержащую минорные значения соответствующих элементов.

    Создает минор матрицы.

    \(A =\begin{bmatrix}a_{11} & a_{12}&a_{13}\\a_{21}&a_{22}&a_{23}\\a_{31}&a_{32}&a_{ 33}\end{bmatrix}\)

    Младший из \(a_{11} = M_{11} =\left|\begin{array}{ll}
    а_{22} и а_{23} \\
    а_{32} и а_{33}
    \конец{массив}\право| = a_{22}.a_{33} — a_{23}.a_{32}\)

    Младший из \(a_{23} = M_{23} =\left|\begin{array}{ll}
    а_{11} и а_{12} \\
    а_{31} и а_{32}
    \конец{массив}\право| = a_{11}.a_{32} — a_{12}.a_{31}\)

    Младший из \(a_{32} = M_{23} =\left|\begin{array}{ll}
    а_{11} и а_{13} \\
    а_{21} и а_{23}
    \конец{массив}\право| = a_{11}. a_{23} — a_{13}.a_{21}\)

    Аналогично можно найти минор каждого элемента матрицы A. Далее можно образовать минор матрицы A, записав минор каждого элемента в матричном массиве.

    Минор матрицы A = \(\begin{bmatrix}M_{11} & M_{12}&M_{13}\\M_{21}&M_{22}&M_{23}\\M_{31}&M_{32 }&M_{33}\end{bmatrix}\)

    Применение минора матрицы

    Минор матрицы полезен для нахождения кофакторов элементов матрицы, что полезно для нахождения сопряженной матрицы и обратной матрицы. Также минор матрицы используется при вычислении определителя матрицы. Попробуем теперь понять следующие важные применения минора матрицы. 9{i+j}) M_{ij}\)

    Матрица образована из кофакторов элементов матрицы и называется кофакторной матрицей.

    Матрица кофакторов = \(\left[\begin{array}{ccc}
    С_{11} и С_{12} и С_{13} \\
    С_{21} и С_{22} и С_{23} \\
    C_{31} и C_{32} и C_{33}
    \end{array}\right] \)

    Эта матрица сомножителей относится к приведенной ниже матрице A.

    Определитель матрицы

    Определитель матрицы представляет собой суммарное значение и рассчитывается с использованием матрица. Определитель матрицы равен сумме произведений элементов определенной строки или столбца с их соответствующими кофакторами. Скажем, рассмотрим матрицу A. 9{1 + 3} \left|\begin{matrix}a_{21}&a_{22}\\a_{31}&a_{32}\end{matrix}\right|\)

    Примыкание к матрице

    сопряженную матрицу 3 x 3 можно получить, выполнив два простых шага. Сначала нам нужно найти матрицу кофакторов данной матрицы, а затем транспонировать матрицу этой матрицы кофакторов, чтобы получить сопряженную матрицу. Рассмотрим следующую матрицу A.

    \(A = \begin{bmatrix} a_{11}&a_{12}&a_{13}\\a_{21}&a_{22}&a_{23}\\a_{31 }&a_{32}&a_{33}\end{bmatrix}\)

    Кофакторная матрица \(A = \begin{bmatrix} A_{11}&A_{12}&A_{13}\\A_{21}&A_{22}&A_{23}\\A_{31}&A_{32} &A_{33}\end{bmatrix}\).

    Adj A = транспонирование матрицы кофакторов = транспонирование \(\begin{bmatrix} A_{11}&A_{12}&A_{13}\\A_{21}&A_{22}&A_{23}\\A_{31 }&A_{32}&A_{33}\end{bmatrix}\) =\(\begin{bmatrix} A_{11}&A_{21}&A_{31}\\A_{12}&A_{22}&A_{32} \\A_{13}&A_{23}&A_{33}\end{bmatrix}\)

    Обратная матрица 9{1 + 3} \left|\begin{matrix}a_{21}&a_{22}\\a_{31}&a_{32}\end{matrix}\right|\)

    Adj A = транспонирование матрицы кофакторов = Транспонирование \(\begin{bmatrix} A_{11}&A_{12}&A_{13}\\A_{21}&A_{22}&A_{23}\\A_{31}&A_{32}&A_{33} \end{bmatrix}\) =\(\begin{bmatrix} A_{11}&A_{21}&A_{31}\\A_{12}&A_{22}&A_{32}\\A_{13}&A_{23 }&A_{33}\end{bmatrix}\)

    A -1 = \(\dfrac{1}{|A|}\). \(\begin{bmatrix} A_{11}&A_{21}&A_{31}\\A_{12}&A_{22}&A_{32}\\A_{13}&A_{23}&A_{33}\end{ bматрица}\)

    Связанные темы

    Следующие связанные темы помогут лучше понять концепцию минора матрицы.

    • Квадратная матрица
    • Типы матриц
    • Матричная формула
    • Транспонирование матрицы

    Часто задаваемые вопросы о миноре матрицы

    Что такое минор матрицы?

    Минор матрицы относится к каждому элементу матрицы и равен части матрицы, оставшейся после исключения строки и столбца, содержащего этот элемент. Минор матрицы определен только для квадратной матрицы. Минор элемента ‘a’ в матрице A = \(\begin{bmatrix}a & b\\c&d\end{bmatrix}\) равен d.

    Как найти миноры матрицы?

    Есть два простых шага, чтобы найти минор матрицы. Сначала идентифицируйте и исключите строку и столбец, который содержит конкретный элемент в матрице. Затем сформируйте новую меньшую матрицу из оставшихся элементов, чтобы представить минор конкретного элемента матрицы.

    Минор элемента ‘e’ в матрице A = \(\begin{bmatrix}a&b & c\\d&e&f\\g&h&i\end{bmatrix}\) равен M = \(\begin{bmatrix}a & c\\g&i\end{bmatrix}\).

    Как найти миноры матрицы 2 × 2?

    Для матрицы порядка 2 × 2 вида A = \(\begin{pmatrix}a & b\\c&d\end{pmatrix}\) минор матрицы A = \(\begin{pmatrix}d & c\\b&a\end{pmatrix}\). Минор определенного элемента в матрице равен оставшемуся элементу после исключения строки и столбца, содержащего этот конкретный элемент.

    В чем разница между минорами матрицы и кофактором матрицы?

    Минор элемента \(a_{ij}\) обозначается как \(M_{ij}\). Кофактор матрицы получается из минора матрицы и равен произведению (-1) 9{i+j}) M_{ij}\).

    Для чего используются миноры матрицы?

    Минор матрицы полезен для нахождения кофакторов элементов матрицы. Миноры матрицы используются для нахождения значения определителя матрицы. Кроме того, эти миноры и кофакторы матрицы можно использовать для нахождения определителя матрицы, сопряженного к матрице и обратного к матрице.

    Ранг матрицы по минорному методу

    Ранг матрицы по минорному методу :

    Здесь мы рассмотрим несколько примеров задач на знание метода нахождения ранга матрицы методом миноров.

    Ранг матрицы A определяется как порядок старшего ненулевого минора матрицы A. Он обозначается символом ρ (A). Ранг нулевой матрицы определяется равным 0.

    Примечание

    (i) Если матрица содержит хотя бы один ненулевой элемент, то ρ (A) ≥ 1

    (ii) Ранг единичной матрицы I n равен n.

    (iii) Если ранг матрицы A равен r, то существует по крайней мере один минор матрицы A порядка r, который не равен нулю, и каждый минор матрицы A порядка r + 1 и выше (если есть) равен нулю .

    (iv) Если A является матрицей размера m × n, то ρ (A) ≤ min {m, n} = минимум m, n.

    (v) Квадратная матрица A порядка n имеет обратную тогда и только тогда, когда ρ (A) = n.

    Вопрос 1 :

    Решение :

    Тогда A — матрица порядка 2×2. Итак, ρ (A) min {2, 2} = 2. Наивысший порядок миноров A равен 2 . Существует только один минор третьего порядка A .

      =  4 — 4 

    |A| =  0

    Ранг данной матрицы будет меньше 2.

    Следовательно, ранг данной матрицы равен 1.

    Вопрос 2 :

    Решение :

    Тогда A — матрица порядка 3 × 2. Итак, ρ (A) min {3, 2} = 2. Наивысший порядок миноров A равен 2 .

    В приведенной выше матрице есть четыре второстепенные матрицы 2 x 2. Находя определители, получаем

    Поскольку минор матрицы 2 x 2 не равен нулю, ранг данной матрицы равен 2.

    Вопрос 3 :

    Решение :

    Тогда A — матрица порядка 2 × 4. Таким образом, ρ (A) min {2, 4} = 2. Наивысший порядок миноров матрицы A равен 2 .

    В приведенной выше матрице есть четыре второстепенные матрицы 2 x 2.

    Ранг данной матрицы составляет 2.

    Вопрос 4:

    Решение:

    Тогда A — матрица порядка 3 × 3. SO ρ (a) min {3, 3} = 3. Высший порядок миноров A равен 3 .

    Находя определитель данной матрицы, получаем

      =  1(-4 + 6) + 2(-2 + 30) + 3(2 — 20)

      =  1(2) + 2(28) + 3( -18)

      =  2 + 56 — 54

      =   58 — 54

    |A| = 4 ≠ 0

    Следовательно, ранг данной матрицы равен 3.

    Вопрос 5:

    Решение:

    Тогда A — матрица порядка 3 × 4. Итак, ρ (A)  мин {3 , 4} = 3. Наивысший порядок миноров A равен 3 .

    Найдя определитель данной матрицы, получим

     0(0–4)–1(0–32) + 2(0–16)

     = 0–1(–32) + 2(–16)

     = 32–32

     = 0

      =  1(8-0) — 2(4-3) + 1(0-4)

      =  8 — 2(1) + 1(-4)

      =  8 — 2 — 4

      =  8 — 6

      =  2 ≠ 0

    Следовательно, ранг данной матрицы равен 3.

    Мы надеемся, что после изучения вышеизложенного учащиеся поняли, что такое «ранг матрицы методом минора».

    Односторонние соответственные накрест лежащие углы: Углы при пересечении двух прямых

    Признак параллельности прямых через внутренние односторонние углы. Параллельные прямые, признаки и условия параллельности прямых

    Страница 1 из 2

    Вопрос 1. Докажите, что две прямые, параллельные третьей, параллельны.
    Ответ. Теорема 4.1. Две прямые, параллельные третьей, параллельны.
    Доказательство. Пусть прямые a и b параллельны прямой c. Допустим, что a и b не параллельны (рис. 69). Тогда они не пересекаются в некоторой точке C. Значит, через точку C проходят две прямые, параллельные прямой c. Но это невозможно, так как через точку, не лежащую на данной прямой, можно провести не более одной прямой, параллельной данной. Теорема доказана.

    Вопрос 2. Объясните, какие углы называются внутренними односторонними. Какие углы называются внутренними накрест лежащими?
    Ответ. Пары углов, которые образуются при пересечении прямых AB и CD секущей AC, имеют специальные названия.
    Если точки B и D лежат в одной полуплоскости относительно прямой AC, то углы BAC и DCA называются внутренними односторонними (рис. 71, а).
    Если точки B и D лежат в разных полуплоскостях относительно прямой AC, то углы BAC и DCA называются внутренними накрест лежащими (рис. 71, б).


    Рис. 71

    Вопрос 3. Докажите, что если внутренние накрест лежащие углы одной пары равны, то внутренние накрест лежащие углы другой пары тоже равны, а сумма внутренних односторонних углов каждой пары равна 180°.
    Ответ. Секущая AC образует с прямыми AB и CD две пары внутренних односторонних и две пары внутренних накрест лежащих углов. Внутренние накрест лежащие углы одной пары, например угол 1 и угол 2, являются смежными внутренним накрест лежащим углам другой пары: угол 3 и угол 4 (рис. 72).


    Рис. 72

    Поэтому если внутренние накрест лежащие углы одной пары равны, то внутренние накрест лежащие углы другой пары тоже равны.
    Пара внутренних накрест лежащих углов, например угол 1 и угол 2, и пара внутренних односторонних углов, например угол 2 и угол 3, имеют один угол общий – угол 2, а два других угла смежные: угол 1 и угол 3.
    Поэтому если внутренние накрест лежащие углы равны, то сумма внутренних углов равна 180°. И обратно: если сумма внутренних накрест лежащих углов равна 180°, то внутренние накрест лежащие углы равны. Что и требовалось доказать.

    Вопрос 4. Докажите признак параллельности прямых.
    Ответ. Теорема 4.2 (признак параллельности прямых). Если внутренние накрест лежащие углы равны или сумма внутренних односторонних углов равна 180°, то прямые параллельны.
    Доказательство. Пусть прямые a и b образуют с секущей AB равные внутренние накрест лежащие углы (рис. 73, а). Допустим, прямые a и b не параллельны, а значит, пересекаются в некоторой точке C (рис. 73, б).


    Рис. 73

    Секущая AB разбивает плоскость на две полуплоскости. В одной из них лежит точка C. Построим треугольник BAC 1 , равный треугольнику ABC, с вершиной C 1 в другой полуплоскости. По условию внутренние накрест лежащие углы при параллельных a, b и секущей AB равны. Так как соответствующие углы треугольников ABC и BAC 1 с вершинами A и B равны, то они совпадают с внутренними накрест лежащими углами. Значит, прямая AC 1 совпадает с прямой a, а прямая BC 1 совпадает с прямой b. Получается, что через точки C и C 1 проходят две различные прямые a и b. А это невозможно. Значит, прямые a и b параллельны.
    Если у прямых a и b и секущей AB сумма внутренних односторонних углов равна 180°, то, как мы знаем, внутренние накрест лежащие углы равны. Значит, по доказанному выше, прямые a и b параллельны. Теорема доказана.

    Вопрос 5. Объясните, какие углы называются соответственными. Докажите, что если внутренние накрест лежащие углы равны, то соответственные углы тоже равны, и наоборот.

    Ответ. Если у пары внутренних накрест лежащих углов один угол заменить вертикальным ему, то получится пара углов, которые называются соответственными углами данных прямых с секущей. Что и требовалось объяснить.
    Из равенства внутренних накрест лежащих углов следует равенство соответственных углов, и наоборот. Допустим, у нас есть две параллельные прямые (так как по условию внутренние накрест лежащие углы равны) и секущая, которые образуют углы 1, 2, 3. Углы 1 и 2 равны как внутренние накрест лежащие. А углы 2 и 3 равны как вертикальные. Получаем: \(\angle\)1 = \(\angle\)2 и \(\angle\)2 = \(\angle\)3. По свойству транзитивности знака равенства следует, что \(\angle\)1 = \(\angle\)3. Аналогично доказывается и обратное утверждение.
    Отсюда получается признак параллельности прямых по соответственным углам. Именно: прямые параллельны, если соответственные углы равны. Что и требовалось доказать.

    Вопрос 6. Докажите, что через точку, не лежащую на данной прямой, можно провести параллельную ей прямую. Сколько прямых, параллельных данной, можно провести через точку, не лежащую на этой прямой?

    Ответ. Задача (8). Даны прямая AB и точка C, не лежащая на этой прямой. Докажите, что через точку C можно провести прямую, параллельную прямой AB.
    Решение. Прямая AC разбивает плоскость на две полуплоскости (рис. 75). Точка B лежит в одной из них. Отложим от полупрямой CA в другую полуплоскость угол ACD, равный углу CAB. Тогда прямые AB и CD будут параллельны. В самом деле, для этих прямых и секущей AC углы BAC и DCA внутренние накрест лежащие. А так как они равны, то прямые AB и CD параллельны. Что и требовалось доказать.
    Сопоставляя утверждение задачи 8 и аксиомы IX (основного свойства параллельных прямых), приходим к важному выводу: через точку, не лежащую на данной прямой, можно провести параллельную ей прямую, и только одну.

    Вопрос 7. Докажите, что если две прямые пересекаются третьей прямой, то внутренние накрест лежащие углы равны, а сумма внутренних односторонних углов равна 180°.

    Ответ. Теорема 4.3 (обратная теореме 4.2). Если две параллельные прямые пересекаются третьей прямой, то внутренние накрест лежащие углы равны, а сумма внутренних односторонних углов равна 180°.
    Доказательство. Пусть a и b – параллельные прямые и c – прямая, пересекающая их в точках A и B. Проведём через точку A прямую a 1 так, чтобы внутренние накрест лежащие углы, образованные секущей c с прямыми a 1 и b, были равны (рис. 76).
    По признаку параллельности прямых прямые a 1 и b параллельны. А так как через точку A проходит только одна прямая, параллельная прямой b, то прямая a совпадает с прямой a 1 .
    Значит, внутренние накрест лежащие углы, образованные секущей с
    параллельными прямыми a и b, равны. Теорема доказана.

    Вопрос 8. Докажите, что две прямые, перпендикулярные третьей, параллельны. Если прямая перпендикулярна одной из двух параллельных прямых, то она перпендикулярна и другой.
    Ответ. Из теоремы 4.2 следует, что две прямые, перпендикулярные третьей, параллельны.
    Предположим, что две какие-либо прямые перпендикулярны третьей прямой. Значит, эти прямые пересекаются с третьей прямой под углом, равным 90°.
    Из свойства углов, образованных при пересечении параллельных прямых секущей, следует, что если прямая перпендикулярна одной из параллельных прямых, то она перпендикулярна и другой.

    Вопрос 9. Докажите, что сумма углов треугольника равна 180°.

    Ответ. Теорема 4.4. Сумма углов треугольника равна 180°.
    Доказательство. Пусть ABC – данный треугольник. Проведём через вершину B прямую, параллельную прямой AC. Отметим на ней точку D так, чтобы точки A и D лежали по по разные стороны от прямой BC (рис. 78).
    Углы DBC и ACB равны как внутренние накрест лежащие, образованные секущей BC с параллельными прямыми AC и BD. Поэтому сумма углов треугольника при вершинах B и C равна углу ABD.
    А сумма всех трёх углов треугольника равна сумме углов ABD и BAC. Так как эти углы внутренние односторонние для параллельных AC и BD и секущей AB, то их сумма равна 180°. Теорема доказана.

    Вопрос 10. Докажите, что у любого треугольника по крайней мере два угла острые.
    Ответ. Действительно, допустим, что у треугольника только один острый угол или вообще нет острых углов. Тогда у этого треугольника есть два угла, каждый из которых не меньше 90°. Сумма этих двух углов уже не меньше 180°. А это невозможно, так как сумма всех углов треугольника равна 180°. Что и требовалось доказать.

    Признаки параллельности двух прямых

    Теорема 1. Если при пересечении двух прямых секущей:

      накрест лежащие углы равны, или

      соответственные углы равны, или

      сумма односторонних углов равна 180°, то

    прямые параллельны (рис.1).

    Доказательство. Ограничимся доказательством случая 1.

    Пусть при пересечении прямых а и b секущей АВ накрест лежащие углы равны. Например, ∠ 4 = ∠ 6. Докажем, что а || b.

    Предположим, что прямые а и b не параллельны. Тогда они пересекаются в некоторой точке М и, следовательно, один из углов 4 или 6 будет внешним углом треугольника АВМ. Пусть для определенности ∠ 4 — внешний угол треугольника АВМ, а ∠ 6 — внутренний. Из теоремы о внешнем угле треугольника следует, что ∠ 4 больше ∠ 6, а это противоречит условию, значит, прямые а и 6 не могут пересекаться, поэтому они параллельны.

    Следствие 1 . Две различные прямые на плоскости, перпендикулярные одной и той же прямой, параллельны (рис. 2).

    Замечание. Способ, которым мы только что доказали случай 1 теоремы 1, называется методом доказательства от противного или приведением к нелепости. Первое название этот способ получил потому, что в начале рассуждения делается предположение, противное (противоположное) тому, что требуется доказать. Приведением к нелепости он называется вследствие того, что, рассуждая на основании сделанного предположения, мы приходим к нелепому выводу (к абсурду). Получение такого вывода заставляет нас отвергнуть сделанное вначале допущение и принять то, которое требовалось доказать.

    Задача 1. Построить прямую, проходящую через данную точку М и параллельную данной прямой а, не проходящей через точку М.

    Решение. Проводим через точку М прямую р перпендикулярно прямой а (рис. 3).

    Затем проводим через точку М прямую b перпендикулярно прямой р. Прямая b параллельна прямой а согласно следствию из теоремы 1.

    Из рассмотренной задачи следует важный вывод:
    через точку, не лежащую на данной прямой, всегда можно провести прямую, параллельную данной .

    Основное свойство параллельных прямых состоит в следующем.

    Аксиома параллельных прямых. Через данную точку, не лежащую на данной прямой, проходит только одна прямая, параллельная данной.

    Рассмотрим некоторые свойства параллельных прямых, которые следуют из этой аксиомы.

    1) Если прямая пересекает одну из двух параллельных прямых, то она пересекает и другую (рис.4).

    2) Если две различные прямые параллельны третьей прямой, то они параллельны (рис.5).

    Справедлива и следующая теорема.

    Теорема 2. Если две параллельные прямые пересечены секущей, то:

      накрест лежащие углы равны;

      соответственные углы равны;

      сумма односторонних углов равна 180°.

    Следствие 2. Если прямая перпендикулярна к одной из двух параллельных прямых, то она перпендикулярна и к другой (см. рис.2).

    Замечание. Теорема 2 называется обратной теореме 1. Заключение теоремы 1 является условием теоремы 2. А условие теоремы 1 является заключением теоремы 2. Не всякая теорема имеет обратную, т. е. если данная теорема верна, то обратная теорема может быть неверна.

    Поясним это на примере теоремы о вертикальных углах. Эту теорему можно сформулировать так: если два угла вертикальные, то они равны. Обратная ей теорема была бы такой: если два угла равны, то они вертикальные. А это, конечно, неверно. Два равных угла вовсе не обязаны быть вертикальными.

    Пример 1. Две параллельные прямые пересечены третьей. Известно, что разность двух внутренних односторонних углов равна 30°. Найти эти углы.

    Решение. Пусть условию отвечает рисунок 6.

    Они не пересекаются, сколько бы их ни продолжали. Параллельность прямых на письме обозначают так: AB || С E

    Возможность существования таких прямых доказывается теоремой.

    Теорема.

    Через всякую точку, взятую вне данной прямой, можно провести параллельную этой прямой . С D , что возможно. Прямая CE параллельна AB .

    Для доказательства допустим противное, т.е., что CE пересекается с AB в некоторой точке M . Тогда из точки M к прямой С D мы имели бы два различных перпендикуляра M D и , что невозможно. Значит, CE не может пересечься с AB , т.е. С E параллельна AB .

    Следствие.

    Два перпендикуляра (С E и DB ) к одной прямой (С D ) параллельны.

    Аксиома параллельных линий.

    Через одну и ту же точку нельзя провести двух различных прямых, параллельных одной и той же прямой.

    Так, если прямая С D , проведенная через точку С параллельна прямой AB , то всякая другая прямая С E , проведенная через ту же точку С , не может быть параллельна AB , т.е. она при продолжении пересечется с AB .

    Доказательство этой не вполне очевидной истины оказывается невозможным. Ее принимают без доказательства, как необходимое допущение (postulatum).

    Следствия.

    1. Если прямая (С E ) пересекается с одной из параллельных (СВ ), то она пересекается и с другой (AB ), потому что в противном случае через одну и ту же точку С проходили бы две различные прямые, параллельные AB , что невозможно.

    2. Если каждая из двух прямых (A и B ) параллельны одной и той же третьей прямой (С ) , то они параллельны между собой.

    Действительно, если предположить, что A и B пересекаются в некоторой точке M , то тогда через эту точку проходили бы две различные прямые, параллельные С , что невозможно.

    Теорема .

    Если прямая перпендикулярна к одной из параллельных прямых, то она перпендикулярна и к другой параллельной . С D .

    Перпендикуляр E F , пересекаясь с AB , непременно пересечет и С D . Пусть точка пересечения будет H .

    Предположим теперь, что С D не перпендикулярна к EH . Тогда какая-нибудь другая прямая, например HK , будет перпендикулярна к EH и, следовательно через одну и ту же точку H будут проходить две прямые параллельные AB : одна С D , по условию, а другая HK по доказанному раньше. Так как это невозможно, то нельзя допустить, что СВ была не перпендикулярна к EH .

    Класс: 2

    Цель урока:

    • сформировать понятие о параллельности 2-х прямых, рассмотреть первый признак параллельности прямых;
    • выработать умение применять признак при решении задач.

    Задачи:

    1. Образовательные: повторение и закрепление изученного материала, формирование понятия о параллельности 2-х прямых, доказательство 1-го признака параллельности 2-х прямых.
    2. Воспитательные: воспитывать умение аккуратно вести записи в тетради и соблюдать правила построения чертежей.
    3. Развивающие задачи: развитие логического мышления, памяти, внимания.

    Оборудование урока:

    • мультимедийный проектор;
    • экран, презентации;
    • чертёжные инструменты.

    Ход урока

    I. Организационный момент.

    Приветствие, проверка готовности к уроку.

    II. Подготовка к активной УПД.

    Этап 1.

    На первом уроке геометрии мы рассматривали взаимное расположение 2-х прямых на плоскости.

    Вопрос. Сколько общих точек могут иметь две прямые?
    Ответ. Две прямые могут иметь либо одну общую точку, либо не имеют не одной общей точки.

    Вопрос. Как будут расположены относительно друг друга 2-е прямые, если они имеют одну общую точку?
    Ответ. Если прямые имеют одну общую точку, то они пересекаются

    Вопрос. Как расположены 2-е прямые относительно друг друга, если они не имеют общих точек?
    Ответ. То в этом случае данные прямые не пересекаются.

    Этап 2.

    На прошлом уроке Вы получили задание сделать презентацию, где мы встречаемся с непересекающимися прямыми в нашей жизни и в природе. Сейчас мы посмотрим эти презентации и выберем из них лучшие. (В жюри вошли учащиеся, которым в силу низкого интеллекта сложно создать свои презентации.)

    Просмотр презентаций, выполненных учащимися: «Параллельность прямых в природе и жизни», и выбор из них лучших.

    III. Активная УПД (объяснение нового материала).

    Этап 1.

    Рисунок 1

    Определение. Две прямые на плоскости, которые не пересекаются, называются параллельными.

    На данной таблице изображены различные случаи расположения 2-х параллельных прямых на плоскости.

    Рассмотрим, какие отрезки будут параллельными.

    Рисунок 2

    1) Если прямая a параллельна b, то и отрезки AB и CD параллельны.

    2) Отрезок может быть параллелен прямой. Так отрезок MN параллелен прямой a.

    Рисунок 3

    3) Отрезок AB параллелен лучу h. Луч h параллелен лучу k.

    4) Если прямая a перпендикулярна прямой c, и прямая b перпендикулярна прямой c, то прямые a и b параллельны.

    Этап 2.

    Углы, образованные двумя параллельными прямыми и секущей.

    Рисунок 4

    Две параллельные прямые пересекаются третьей прямой в двух точках. При этом образуются восемь углов, обозначенных на рисунке числами.

    Некоторые пары этих углов имеют специальные названия (см. рисунок 4).

    Существует три признака, параллельности двух прямых , связанных с этими углами. На этом уроке мы рассмотрим первый признак .

    Этап 3.

    Повторим материал, необходимый для доказательства этого признака.

    Рисунок 5

    Вопрос. Как называются углы, изображённые на рисунке 5?
    Ответ. Углы AOC и COB называются смежными.

    Вопрос. Какие углы называются смежными? Дайте определение.
    Ответ. Два угла называются смежными, если у них одна сторона является общей, а две другие являются продолжениями друг друга.

    Вопрос. Каким свойством обладают смежные углы?
    Ответ. Смежные углы в сумме дают 180 градусов.
    AOC + COB = 180°

    Вопрос. Как называются углы 1 и 2?
    Ответ. Углы 1 и 2 называются вертикальными.

    Вопрос. Какими свойствами обладают вертикальные углы?
    Ответ. Вертикальные углы равны между собой.

    Этап 4.

    Доказательство первого признака параллельности.

    Теорема. Если при пересечении двух прямых секущей накрест лежащие углы равны, то прямые параллельны.

    Рисунок 6

    Дано: а и b – прямые
    AB – секущая
    1 = 2
    Доказать: a//b.

    1-ый случай.

    Рисунок 7

    Если 1 и 2 прямые, то a перпендикулярен AB, и b перпендикулярен AB, то а//b.

    2-ой случай.

    Рисунок 8

    Рассмотрим случай, когда 1 и 2 не прямые Разделим отрезок AB пополам точкой O.

    Вопрос. Какими будут отрезки AO и OB по длине?
    Ответ. Отрезки AO и OB равны по длине.

    1) Из точки O проведём перпендикуляр к прямой а, ОН перпендикулярен a.

    Вопрос. Каким будет угол 3?
    Ответ. Угол 3 будет прямым.

    2) От точки А на прямой b отложим циркулем отрезок АН 1 = ВН.

    3) Проведём отрезок ОН 1 .

    Вопрос. Какие треугольники образовались в результате доказательства?
    Ответ.
    Треугольник ОНВ и треугольник ОН 1 А.

    Докажем, что они равны.

    Вопрос. Какие углы равны по условию теоремы?
    Ответ. Угол 1 равен углу 2.

    Вопрос. Какие стороны равны по построению.
    Ответ. АО = ОВ и АН 1 = ВН

    Вопрос. По какому признаку равны треугольники?
    Ответ. Треугольники равны по двум сторонам и углу между ними (первый признак равенства треугольников).

    Вопрос. Каким свойством обладают равные треугольники?
    Ответ. В равных треугольниках против равных сторон лежат равные углы.

    Вопрос. Какие углы будут равны?
    Ответ. 5 = 6, 3 = 4.

    Вопрос. Как называются 5 и 6?
    Ответ. Эти углы называются вертикальными.

    Из этого следует, что точки: Н 1 , О, Н лежат на одной прямой.
    Т.к. 3 – прямой, а 3 = 4, то 4 – прямой.

    Вопрос. Как расположены прямые а и b по отношению к прямой НН 1 , если углы 3 и 4 прямые?
    Ответ. Прямые а и b перпендикулярны HH 1 .

    Вопрос. Что мы можем сказать о двух перпендикулярах к одной прямой?
    Ответ. Два перпендикуляра одной прямой параллельны.

    Итак, а//b. Теорема доказана.

    Сейчас я повторю все доказательство сначала, а Вы внимательно меня послушаете постараетесь все понять запомнить.

    IV. Закрепление нового материала.

    Работа по группам с разным уровнем развития интеллекта, с последующей проверкой на экране и на доске. У доски работают 3 ученика (по одному из каждой группы).

    №1 (для учащихся со сниженным уровнем интеллектуального развития).

    Дано: а и b прямые
    с – секущая
    1 = 37°
    7 = 143°
    Доказать: а//b.

    Решение.

    7 = 6 (вертикальные) 6 = 143°
    1 + 4 = 180° (смежные) 4 =180° – 37° = 143°
    4 = 6 = 143°, а они накрест лежащие а//b 5 = 48°, 3 и 5 – накрест лежащие углы, они равны a//b.

    Рисунок 11

    V. Итог урока.

    Итог урока проводится с использованием рисунков 1-8.

    Производится оценка деятельности учащихся на уроке (каждый ученик получает соответствующий смайлик).

    Домашнее задание: учить – стр. 52-53; решить №186 (б, в).

    Параллельность – очень полезное свойство в геометрии. В реальной жизни параллельные стороны позволяют создавать красивые, симметричные вещи, приятные любому глазу, поэтому геометрия всегда нуждалась в способах эту параллельность проверить. О признаках параллельных прямых мы и поговорим в этой статье.

    Определение для параллельности

    Выделим определения, которые необходимо знать для доказательства признаков параллельности двух прямых.

    Прямые называют параллельными, если они не имеют точек пересечения. Кроме того, в решениях обычно параллельные прямые идут в связке с секущей линией.

    Секущей прямой называется прямая, которая пересекает обе параллельные прямые. В этом случае образуются накрест лежащие, соответственные и односторонние углы. Накрест лежащими будут пары углов 1 и 4; 2 и 3; 8 и 6; 7 и 5. Соответственными будут 7 и 2; 1 и 6; 8 и 4; 3 и 5.

    Односторонними 1 и 2; 7 и 6; 8 и 5; 3 и 4.

    При правильном оформлении пишется: «Накрест лежащие углы при двум параллельных прямых а и b и секущей с», потому что для двух параллельных прямых может существовать бесконечное множество секущих, поэтому необходимо указывать, какую именно секущую, вы имеете в виду.

    Также для доказательства понадобится теорема о внешнем угле треугольника, которая гласит, что внешний угол треугольника равен сумме двух углов треугольника несмежных с ним.

    Признаки

    Все признаки параллельных прямых завязаны на знание свойств углов и теорему о внешнем угле треугольника.

    Признак 1

    Две прямые параллельны, если накрест лежащие углы равны.

    Рассмотрим две прямые а и b с секущей с. Накрест лежащие углы 1 и 4 равны. Предположим, что прямые не параллельны. Значит прямые пересекаются и должна быть точка пересечения М. Тогда образуется треугольник АВМ с внешним углом 1. Внешний угол должен быть равен сумме углов 4 и АВМ как несмежных с ним по теореме о внешнем угле в треугольнике. Но тогда получится, что угол 1 больше угла 4, а это противоречит условию задачи, значит, точки М не существует, прямые не пересекаются, то есть параллельны.

    Рис. 1. Рисунок к доказательству.

    Признак 2

    Две прямые параллельны, если соответственные углы при секущей равны.

    Рассмотрим две прямые а и b с секущей с. Соответственные углы 7 и 2 равны. Обратим внимание на угол 3. Он является вертикальным для угла 7. Значит, углы 7 и 3 равны. Значит, углы 3 и 2 также равны, так как

    Рис. 2. Рисунок к доказательству.

    Признак 3

    Две прямые параллельны, если сумма односторонних углов равна 180 градусам.

    Рис. 3. Рисунок к доказательству.

    Рассмотрим две прямые а и b с секущей с. Сумма односторонних углов 1 и 2 равна 180 градусов. Обратим внимание на углы 1 и 7. Они являются смежными. То есть:

    $$

    $$

    Вычтем из первого выражения второе:

    $$(

    $$(

    $$

    $$

    $

    Что мы узнали?

    Мы в подробностях разобрали, какие углы получаются при рассечении параллельных прямых третьей линией, выделили и подробно расписали доказательство трех признаков параллельности прямых.

    Тест по теме

    Оценка статьи

    Средняя оценка: 4.1 . Всего получено оценок: 220.

    Углы при параллельных прямых и секущей. Вертикальные, смежные, односторонние, соответственные, накрест лежащие углы — Студопедия

    Поделись с друзьями: 

    Пусть прямая с пересекает параллельные прямые а и b. При этом образуется восемь углов. Углы при параллельных прямых и секущей так часто используются в задачах, что в геометрии им даны специальные названия.

    Углы 1 и 3 — вертикальные. Очевидно, вертикальные углы равны, то есть
    ∠1 = ∠3,
    ∠2 = ∠4.

    Конечно, углы 5 и 7, 6 и 8 — тоже вертикальные.

    Углы 1 и 2 — смежные, это мы уже знаем. Сумма смежных углов равна 180º.

    Углы 3 и 5 (а также 2 и 8, 1 и 7, 4 и 6) — накрест лежащие. Накрест лежащие углы равны.
    ∠3 = ∠5,
    ∠1 = ∠7,
    ∠2 = ∠8,
    ∠4 = ∠6.

    Углы 1 и 6 — односторонние. Они лежат по одну сторону от всей «конструкции». Углы 4 и 7 — тоже односторонние. Сумма односторонних углов равна180°, то есть
    ∠1 + ∠6 = 180°,
    ∠4 + ∠7 = 180°.

    Углы 2 и 6 (а также 3 и 7, 1 и 5, 4 и 8) называются соответственными.

    Соответственные углы равны, то есть
    ∠2 = ∠6,
    ∠3 = ∠7.

    Углы 3 и 5 (а также 2 и 8, 1 и 7, 4 и 6) называют накрест лежащими.

    Накрест лежащие углы равны, то есть
    ∠3 = ∠5,
    ∠1 = ∠7,
    ∠2 = ∠8,
    ∠4 = ∠6.

    Чтобы применять все эти факты в решении задач ЕГЭ, надо научиться видеть их на чертеже. Например, глядя на параллелограмм или трапецию, можно увидеть пару параллельных прямых и секущую, а также односторонние углы. Проведя диагональ параллелограмма, видим накрест лежащие углы. Это — один из шагов, из которых и состоит решение.

    1. Биссектриса тупого угла параллелограмма делит противоположную сторону в отношении 3:4, считая от вершины тупого угла. Найдите большую сторону параллелограмма, если его периметр равен 88.

    Напомним, что биссектриса угла — это луч, выходящий из вершины угла и делящий угол пополам.

    Пусть ВМ — биссектриса тупого угла В. По условию, отрезки МD и АВ равны 3х и 4х соответственно.

    Рассмотрим углы СВМ и ВМА. Поскольку АD и ВС параллельны, ВМ — секущая, углы СВМ и ВМА являются накрест лежащими. Мы знаем, что накрест лежащие углы равны. Значит, треугольник АВМ — равнобедренный, следовательно, АВ = АМ = 4х.

    Периметр параллелограмма — это сумма всех его сторон, то есть
    7х + 7х + 4х + 4х = 88.
    Отсюда х = 4, 7х = 28.

    Ответ: 28.

    2. Диагональ параллелограмма образует с двумя его сторонами углы 26º и 34º. Найдите больший угол параллелограмма. Ответ дайте в градусах.

    Нарисуйте параллелограмм и его диагональ. Заметив на чертеже накрест лежащие углы и односторонние углы, вы легко получите ответ: 120º.

    3. Чему равен больший угол равнобедренной трапеции, если известно, что разность противолежащих углов равна 50º? Ответ дайте в градусах.

    Мы знаем, что равнобедренной (или равнобокой) называется трапеция, у которой боковые стороны равны. Следовательно, равны углы при верхнем основании, а также углы при нижнем основании.

    Давайте посмотрим на чертеж. По условию, α — β = 50°, то есть α = β + 50°.

    Углы α и β — односторонние при параллельных прямых и секущей, следовательно,
    α + β = 180°.

    Итак, 2β + 50° = 180°
    β = 65°, тогда α = 115°.

    Ответ: 115.

    EGE-Study» Методические материалы» Геометрия: с нуля до C4» Высоты, медианы, биссектрисы треугольника


    Понравилась статья? Добавь ее в закладку (CTRL+D) и не забудь поделиться с друзьями:  


    

    Что такое конгруэнтный угол? (Примеры вопросов)

    ОбзорТранскриптПрактика

    Конгруэнтные углы часто используются в мире архитектуры, строительства, дизайна и искусства. Равные углы имеют одинаковую угловую меру. Например, правильный пятиугольник имеет пять сторон и пять углов, каждый из которых равен 108 градусам. Независимо от размера или масштаба правильного многоугольника, углы всегда будут конгруэнтны.

    Существует множество правил, позволяющих определить, равны ли углы или нет. Например, если два треугольника подобны, их соответствующие углы будут равны. Это означает, что углы, находящиеся в одном и том же совпадающем положении, будут иметь одинаковый угол.

    Еще один распространенный тест на соответствие углов требует набора параллельных прямых и поперечной линии, пересекающей набор параллельных прямых. Например, прямые a и b параллельны, а прямая l является секущей, пересекающей параллельные прямые. Когда возникает такая ситуация, образуется несколько конгруэнтных углов. В этом сценарии образуются четыре основных типа конгруэнтных углов: альтернативные внутренние углы, альтернативные внешние углы, соответствующие углы и вертикальные углы.


    Альтернативные внутренние углы расположены между двумя параллельными линиями, но на противоположных сторонах поперечной. В этом конкретном примере конгруэнтными альтернативными внутренними углами будут ∠2 и ∠6, а также ∠7 и ∠3.

    Аналогично, Альтернативные внешние углы расположены снаружи параллельных линий и на противоположных сторонах поперечных. ∠5 и ∠1 конгруэнтны, а также ∠4 и ∠8.

    Соответствующие углы расположены на одной стороне поперечной и в аналогичном месте. Например, ∠4 и ∠6 — соответствующие углы, поэтому они равны. Другие пары соответствующих углов включают ∠3 и ∠5, ∠1 и ∠7, а также ∠2 и ∠8.

    Вертикальные углы образованы углами, противоположными друг другу. Например, ∠1 и ∠3, ∠7 и ∠5, ∠4 и ∠2, ∠6 и ∠8 — все пары равных углов. Вертикальные углы или противоположные углы обычно используются в качестве доказательства конгруэнтности.

    Другая категория конгруэнтных углов связана с конгруэнтностью треугольника. Правила конгруэнтности треугольников используются, чтобы доказать, конгруэнтны два треугольника или нет. Эти правила учитывают длины сторон и углы треугольников, чтобы определить конгруэнтность. Четыре критерия используются для определения конгруэнтности треугольников, и они имеют удобные названия.

    Например:
    S-S-S относится к двум треугольникам, все стороны которых имеют одинаковую длину. Если это так, то все соответствующие меры углов также будут конгруэнтны.

    S-A-S относится к двум треугольникам, у которых две конгруэнтные стороны и один конгруэнтный угол между ними. Если это так, то все соответствующие углы будут равны.

    Аналогично, A-S-A говорит нам, что два треугольника имеют два конгруэнтных угла и одну конгруэнтную длину стороны между ними. Опять же, если это так, то все соответствующие углы будут равны.

    Наконец, A-A-S относится к двум треугольникам, которые имеют два соответствующих конгруэнтных угла с соответствующей конгруэнтной длиной стороны. Это говорит нам о том, что все соответствующие углы будут равны.

     

    Конгруэнтные углы обычно используются при изучении геометрии и во многих реальных профессиях. Строители, инженеры, строители и художники регулярно используют конгруэнтные углы. Определение конгруэнтности углов является важным навыком, который помогает заложить основу для изучения геометрии.

    Углы окружают нас повсюду. Они используются инженерами, архитекторами и художниками для поиска творческих решений специализированных проблем и создания красивых светильников и произведений искусства. Много раз углы, представленные в формах и пересекающихся линиях, имеют отношения друг к другу, которые мы можем использовать для определения их меры. Одним из таких отношений является конгруэнтность.

    Если мы говорим, что два угла конгруэнтны , мы имеем в виду, что они имеют одинаковую меру в градусах. Например, предположим, что мы построили квадратную комнату, где все углы равны 9 углам.0°. Все четыре его угла будут иметь конгруэнтные углы, потому что все они имеют одинаковую меру.

    Точно так же мы знаем, что в равнобедренном треугольнике два угла имеют одинаковую меру. Следовательно, эти два угла равны. Чтобы обозначить, что некоторые углы конгруэнтны друг другу, мы обычно проводим дугу в углах.

    Конгруэнтные углы действительно начинают появляться, когда мы наблюдаем пересечения прямых линий. Например, если одна прямая пересекает другую, как показано, мы можем видеть, что создаются пары конгруэнтных углов, противоположных друг другу. Поскольку обе линии прямые, углы на каждой стороне в основном зеркальны, так что красные углы конгруэнтны, а синие углы конгруэнтны. Мы называем эти углы вертикальными углами.

    Также полезно отметить, что поскольку линии прямые, два смежных угла вместе образуют прямой угол в 180°. Это свойство пригодится, когда вам нужно найти недостающие углы.

    Давайте рассмотрим пример. Если угол A равен 40°, каковы размеры углов B , C и D ?

    Поскольку обе пересекающиеся прямые прямые, мы знаем, что противоположные углы A и C совпадают. Поэтому C также должны быть 40°. Точно так же углы B и D равны. Их размеры можно определить, используя тот факт, что A + B должны равняться 180°.

    \(A+B=180°\)
     
    \(40°+B=180°\)
     
    \(B=140°\)

     

    So B и 9007 1 D должны быть оба меры 140°.

    Что происходит, когда мы наблюдаем две параллельные линии, которые пересекает третья линия?

    Рассмотрим параллельные прямые p и q и пересекающую их поперечную линию t . Теперь мы наблюдаем два пересечения и всего восемь углов.

    При внимательном рассмотрении этих пересечений видно, что они идентичны друг другу. Поскольку p и q параллельны, их пересечения с t образуют конгруэнтные углы. Таким образом, конгруэнтны не только углы 1 и 3, но и углы 5 и 7 также включены в это же соответствие. Точно так же равны углы 2, 4, 6 и 8.

    Существует четыре классификации типов конгруэнтных углов, присутствующих в этом сценарии: вертикальные углы, соответствующие углы, чередующиеся внешние углы и чередующиеся внутренние углы.

    Вертикальные углы — это углы, которые противоположны друг другу в одной точке пересечения. Это похоже на то, что мы наблюдали в предыдущем примере. Здесь мы видим, что углы 1 и 3 снова равны, и поскольку они лежат друг напротив друга в одном и том же пересечении, они классифицируются как вертикальные углы. Точно так же пары углов 2 и 4, 5 и 7, 6 и 8 являются парами вертикальных углов.

    Соответствующие углы представляют собой пары углов, находящихся в одинаковом относительном положении пересечения каждой параллельной прямой и поперечной. Соответствующие углы равны между собой. Например, углы 2 и 6 являются соответствующими углами, потому что они оба появляются справа от t и над параллельными прямыми. Значит, это равные углы. Другими парами соответствующих углов в этом примере являются углы 1 и 5, 3 и 7, 4 и 8.

    Альтернативные внешние углы — это пары углов, которые появляются на внешней стороне параллельных прямых и на противоположных сторонах поперечной. Альтернативные внешние углы равны друг другу. Углы 1 и 7 являются противоположными внешними углами и, следовательно, равны друг другу. Углы 2 и 8 также являются альтернативными внешними углами, что делает их конгруэнтными друг другу.

    Аналогично, альтернативные внутренние углы представляют собой пары углов, которые появляются внутри параллельных прямых и на противоположных сторонах поперечной. Альтернативные внутренние углы конгруэнтны друг другу. Здесь это будут пары 3 и 5, 4 и 6.

    Мы обсудили, что два или более угла конгруэнтны, если они имеют одну и ту же угловую меру, и мы рассмотрели несколько геометрических приложений конгруэнтных углов. Эти идеи помогают строителям, плотникам и инженерам добиваться совершенства в своей работе при проектировании инструментов, объектов и зданий. Потратьте некоторое время, чтобы самостоятельно решить некоторые примеры задач, и вскоре вы увидите конгруэнтные углы в окружающем вас мире!

    Надеюсь, это видео было полезным. Спасибо за просмотр и удачной учебы!

    Вопрос №1:

     
    Углы 1 и 2 — соответствующие углы. Если угол 2 равен 67°, то чему равен угол 1?

    164°

    24°

    344°

    67°

    Показать ответ

    Ответ:

    900 04 Когда две параллельные прямые пересекаются секущей, углы, лежащие по одну сторону от этой секущей, и в совпадающих углах будут конгруэнтными. Углы 1 и 2 равны, поэтому оба имеют угловую меру 67°.

    Скрыть Ответ

    Вопрос №2:

     
    Прямая r — это секущая, пересекающая две параллельные прямые s и t . Перечислите все углы, равные углу 6.

    ∠8, ∠3 и ∠2

    ∠8, ∠4 и ∠2

    ∠2

    ∠1, ∠7 , и ∠2

    Показать ответ

    Ответ:

    ∠8 конгруэнтно ∠6, потому что они вертикальные или противоположные углы.
    ∠2 конгруэнтно ∠6, потому что они являются соответствующими углами (на одной стороне поперечной и в соответствующих углах).
    ∠4 конгруэнтно ∠6, потому что они являются альтернативными внутренними углами (альтернативные стороны поперечной и между двумя параллельными прямыми).

    Скрыть ответ

    Вопрос № 3:

     
    Если следующие неправильные четырехугольники равны, угол C должен быть равен какому другому углу?

    ∠Е

    ∠Ф

    ∠G

    ∠H

    Показать ответ

    Ответ:

    Соответственные углы конгруэнтны для конгруэнтных многоугольников. ∠C конгруэнтно ∠G
    ∠D конгруэнтно ∠H
    ∠A конгруэнтно ∠E
    ∠B конгруэнтно ∠F

    Скрыть ответ

    Вопрос №4: 90 013

     
    Город Сиэтл строит пешеходную дорожку, пересекающую пару железнодорожных путей. Пешеходная дорожка представлена ​​поперечной буквой t на изображении ниже. Железнодорожные пути представлены параллельными линиями l и m. Если город хочет, чтобы пешеходная дорожка пересекала рельсы под углом 135° (угол 1), каковы будут значения углов 2, 3 и 4?

    ∠2 = 45° ∠3 = 135° ∠4 = 45°

    ∠2 = 45° ∠3 = 45° ∠4 = 135°

    ∠2 = 145° ∠3 = 4 5° ∠4 = 45 °

    Ϫ2 = 180 ° Ϫ3 = 45 ° ♂4 = 45 °

    Показать ответ

    Ответ:

    ϩ1 и ▲ — это согласованные, потому что это вертикальные углы. Если ∠1 равно 135°, то ∠2 должно быть равно 45°, потому что их сумма должна быть 180°, чтобы образовать прямую линию. Теперь, когда мы знаем, что ∠2 равно 45°, мы также знаем, что ∠3 равно 45°, потому что это вертикальные углы.

    Скрыть ответ

    Вопрос №5:

     
    У Келси есть прямоугольный сад, который она хочет разделить на две равные части по диагонали. Одна секция будет для моркови, а другая для капусты. Она разделяет сад на две треугольные части, как на изображении ниже. Если мера ∠DCA равна 40°, какова мера ∠CAB?

    20°

    30°

    40°

    50°

    Показать ответ

    Ответ:

    Если прямая AC пересекает параллельные прямые DC и AB, то углы DCA и CAB равны. Угол DCA и угол CAB образуют чередующиеся внутренние углы, поэтому угол CAB будет равен 40°.

    Скрыть ответ

    Вернуться к видео о геометрии

    642874

    Углы на прямой | Геометрия прямых линий

    В этой главе вы исследовать отношения между парами углов, которые создается, когда прямые линии пересекаются (встречаются или пересекаются). Вы будете рассмотреть пары углов, образованных перпендикулярными линиями, любыми двумя пересекающимися прямыми и третьей прямой, которая разрезает две параллельные линии. Вы придете к пониманию того, что под вертикально противоположными углами подразумеваются соответствующие углы, параллельные углы и внутренние углы. Вы будете в состоянии определить различные пары углов, а затем использовать свои знания, чтобы помочь вам определить неизвестные углы в геометрических фигурах.

    Углы на прямой

    Сумма углов на прямой

    На рисунках ниже каждый угол присвоена метка от 1 до 5.

    1. Используйте транспортир, чтобы Измерьте размеры всех углов каждой фигуры. Написать свой ответы на каждую фигуру.

      А

      Б

    2. Используйте свои ответы, чтобы заполнить размеры уголков ниже.
        9{\ круг} \)

    Сумма углов, образованная на прямой, равна 180°. (Мы можно сократить это свойство как: \(\угол\)s на прямая. )

    Два угла, сумма величин которых составляет 180°, также называется дополнительных углов, например \(\шляпа{1} + \шляпа{2}\).

    Углы, имеющие общую вершину и общую сторону, считается смежным . Таким образом, \(\шляпа{1} + \шляпа{2}\) также называется дополнительными смежными углами .

    Когда две линии перпендикулярны, их смежные дополнительные углы равны равен 90°.

    На рисунке ниже DC A и DC B смежные дополнительные углы, потому что они рядом друг с другом (смежные), и они в сумме составляют 180° (дополнительно).

    Нахождение неизвестных углов на прямых

    9{\circ} \\ &= \text{______} \end{align}\)


  • Рассчитать размер \(Икс\).


  • Рассчитать размер \(у\).


  • w3.org/1999/xhtml»> Нахождение дополнительных неизвестных углов на прямых

    1. Рассчитать размер из:

      1. \(х\)
      2. \(\шляпа{ECB}\)
    2. Рассчитать размер из:

      1. \(м\)
      2. \(\шляпа{SQR}\)
    3. Рассчитать размер из:

      1. \(х\)
      2. \(\шляпа{HEF}\)
    4. Рассчитать размер из:

      1. \(к\)
      2. \(\шляпа{ТИП}\)
    5. Рассчитать размер из:

      1. \(р\)
      2. \(\шляпа{JKR}\)

    w3.org/1999/xhtml»> Вертикально противоположные углы

    Что такое вертикально противоположные углы?

    1. Используйте транспортир, чтобы Измерьте размеры всех углов на фигуре. Написать свой ответы на рисунке.

    2. Уведомление какие углы равны и как эти углы равны сформировался.

    Вертикально напротив углы ( верт. опп. \(\угол\) с ) это углы, противоположные друг другу, когда две прямые пересекаются. 9{\ circ} && \\ & = \text{______} \\ \\ z &= \text{______} &&[\text{vert. opp.}\angle\text{s}] \end{align}\)


  • Вычислить \(j,~ к\) и \(1\).


  • Вычислить \(а,~ б,~ в\) и \(г\).


  • w3.org/1999/xhtml»> Уравнения с использованием вертикально противоположных углов

    Вертикально противоположные углы всегда равный. Мы можем использовать это свойство для построения уравнения. Тогда мы решить уравнение, чтобы найти значение неизвестного переменная. 9{\circ} \\ &= \text{______} \end{align}\)


  • Рассчитать значение \(т\).


  • Рассчитать значение \(п\).


  • Рассчитать значение \(г\).


  • Рассчитать значение \(у\).


  • Рассчитать значение \(р\).


  • Прямые, пересекаемые секущей

    w3.org/1999/xhtml»> Пары углов, образованные секущей

    пересекает по крайней мере две другие линии.

    При пересечении секущей двух линии, мы можем сравнить наборы углов на двух линиях с помощью глядя на их позиции.

    Углы, лежащие на одной стороне трансверсали и находятся в совпадающих позициях, называются соответствующих углов ( корр. \(\угол\) s ). В на рисунке это соответствующие углы:

    • \(a\) и \(е\)
    • \(б\) и \(ф\)
    • \( д\) и \(ч\)
    • \(с\) и \(г\).
    1. На рисунке \(a\) и \(e\) оба левые от трансверсали и над линией.

      Запишите расположение следующих соответствующих углов. Первый сделан для тебя.

      \(b\) и \(f\): справа от поперечных и вышележащих линий


      \(г\) и \(ч\):


      \(с\) и \(г\):


    Переменные углы ( альт. \(\угол\) s ) ложь на противоположных сторонах трансверсали, но не являются смежными или вертикально напротив. Когда противоположные углы лежат между две линии, они называются альтернативные внутренние углы . В на рисунке это альтернативные внутренние углы:

    • \(d\) и \(ф\)
    • \(с\) и \(е\)

    Когда противоположные углы лежат снаружи из двух линий, они называются альтернативными внешними углы . На рисунке это альтернативные экстерьеры углы:

      w3.org/1999/xhtml»>
    • \(а\) и \(г\)
    • \(б\) и \(ч\)
    1. Запишите расположение следующих альтернативных углов:

      \(г\) и \(е\):


      \(с\) и \(е\):


      \(а\) и \(г\):


      \(b\) и \(h\):


    Внутренние углы ( со-цел. \(\угол\) s ) лежат по одну сторону от поперечной и между двумя линии. На рисунке это совнутренние углы:

    • \(с\) и \(ф\)
    • \(г\) и \(е\)
    1. Запишите расположение следующего соинтерьера углы:

      \(г\) и \(д\):


      \(с\) и \(е\):


    w3.org/1999/xhtml»> Определение типов углов

    Две прямые пересекаются поперечной, как показано ниже.

    Запишите следующие пары углы:

    1. две пары соответствующих углы:
    2. две пары чередующихся внутренние углы:
    3. две пары чередующихся внешние углы:
    4. две пары совмещенных интерьеров углы:
    5. две пары вертикально противоположные углы:

    Параллельные прямые, пересекаемые секущей

    Изучение размеров углов

    На рисунке слева внизу EF представляет собой трансверсальной к AB и CD. На рисунке справа внизу PQ представляет собой трансверсальные параллельным прямым JK и LM.

    1. Используйте транспортир, чтобы Измерьте размеры всех углов каждой фигуры. Написать замеры на фигурах.
    2. Используйте свои измерения, чтобы заполните следующую таблицу.

      Корр.\(\угол\)s

      \( \шляпа{1} = \текст{_______};~\шляпа{5} = \текст{_______}\)

      \( \шляпа{4} = \текст{_______};~\шляпа{8} = \текст{_______}\)

      \( \шляпа{2} = \текст{_______};~\шляпа{4} = \текст{_______}\)

      \( \шляпа{3} = \текст{_______};~\шляпа{7} = \текст{_______}\)

      \( \шляпа{9} = \текст{_______};~\шляпа{13} = \текст{_______}\)

      \( \шляпа{12} = \текст{_______};~\шляпа{16} = \текст{_______}\)

      \( \шляпа{10} = \текст{_______};~\шляпа{14} = \текст{_______}\)

      \( \шляпа{11} = \текст{_______};~\шляпа{15} = \текст{_______}\)

      Alt. int.\(\угол\)s

      \( \шляпа{4} = \текст{_______};~\шляпа{6} = \текст{_______}\)

      \( \шляпа{3} = \текст{_______};~\шляпа{5} = \текст{_______}\)

      \( \шляпа{12} = \текст{_______};~\шляпа{14} = \текст{_______}\)

      \( \шляпа{11} = \текст{_______};~\шляпа{13} = \текст{_______}\)

      Альт.расшир.\(\угол\)s

      \( \шляпа{1} = \текст{_______};~\шляпа{7} = \текст{_______}\)

      \( \шляпа{2} = \текст{_______};~\шляпа{8} = \текст{_______}\)

      \( \шляпа{9} = \текст{_______};~\шляпа{15} = \текст{_______}\)

      \( \шляпа{10} = \текст{_______};~\шляпа{16} = \текст{_______}\)

      Co-int. \(\угол\)s

      \(\шляпа{4} + \шляпа{5} = \текст{_______}\)

      \(\шляпа{3} + \шляпа{6} = \текст{_______}\)

      \( \шляпа{12} + \шляпа{13} = \текст{_______}\)

      \( \шляпа{11} + \шляпа{14} = \текст{_______}\)

    3. Посмотрите на свой заполненный таблица к вопросу 2. Что вы заметили об углах, образованных когда секущая пересекает параллельные прямые?

    Когда линии параллельно:

    • соответствующие углы равны
    • альтернативных внутренних углов равны
    • альтернативных внешних углов равны
    • внутренних углов в сумме дают 180°

    w3.org/1999/xhtml»> Определение углов на параллельных прямых

    1. Заполните соответствующие углы к заданным.

    2. Заполнить альтернативный экстерьер углы.

      1. Заполнить альтернативный интерьер углы.
      2. Обведите две пары совмещенных интерьеров углы на каждой фигуре.

      1. Не измеряя, заполните все углы на следующих рисунках равны \(x\) и \(у\).
      2. Объясните причины для каждого \(x\) и \(y\), которые вы заполнили своему партнеру.
    3. Задайте значение \(x\) и \(у\) ниже. 9{\circ} &&[\angle\text{s на прямой}] \end{align}\)

    4. Разработать размеры \(р,~q\) и \(г\).


    5. Найдите размеры \(a,~b,~c\) и \(d\).


    6. Найти размеры всех углов этой фигуры.


    7. Найти размеры из всех углов. (Вы видите две трансверсали и два набора параллельных линий?)


    Удлинитель

    Два уголка в следующая диаграмма обозначена как \(x\) и \(y\). Заполните все углы, равные \(x\) и \(y\).

    Сумма углов четырехугольника

    Диаграмма ниже представляет собой сечение предыдущую схему. 9{\circ}\)


    Можете ли вы придумать другой способ Используйте приведенную выше диаграмму, чтобы определить сумму углов в четырехугольник?

    w3.org/1999/xhtml»> Решение дополнительных геометрических задач

    Соотношение углов на параллельных прямых

    1. Вычислить размеры от \(\hat{1}\) до \(\hat{7}\).


    2. Рассчитать размеры \(х,~у\) и \(z\).


    3. Рассчитать размеры \(a, ~b, ~c\) и \(d\).


    4. Рассчитать размер \(Икс\).


    5. Рассчитать размер \(Икс\).


    6. Вычислите размер \(x\).


    7. Рассчитать размеры \(а\) и \(\шляпа{КЭП}\).


    w3.org/1999/xhtml»> Включая свойства треугольников и четырехугольников

    1. Вычислить размеры от \(\hat{1}\) до \(\hat{6}\).


    2. РГТУ — трапеция. Вычислить размеры \(\шляпа{T}\) и \(\шляпа{R}\).

    3. JKLM представляет собой ромб. Вычислить размеры \(\шляпа{JML}, \шляпа{M_2}\) и \(\шляпа{K_1}\).

    4. ABCD является параллелограмм. Рассчитайте размеры \(\hat{ADB}, \hat{ABD}, \hat{C}\) и \(\hat{DBC}\)

    1. Посмотрите на рисунок ниже. Имя элементы, перечисленные рядом.

      1. пара вертикально противоположные углы
      2. пара соответствующих углы
      3. пара альтернативных внутренние углы
      4. пара совмещенных интерьеров углы
    2. На схеме AB \(\параллельно\) CD.

    Как углы перевести в радианы: Онлайн калькулятор: Перевод градусов в радианы

    Mathway | Популярные задачи

    1Найти точное значениеsin(30)
    2Найти точное значениеsin(45)
    3Найти точное значениеsin(30 град. )
    4Найти точное значениеsin(60 град. )
    5Найти точное значениеtan(30 град. )
    6Найти точное значениеarcsin(-1)
    7Найти точное значениеsin(pi/6)
    8Найти точное значениеcos(pi/4)
    9Найти точное значениеsin(45 град. )
    10Найти точное значениеsin(pi/3)
    11Найти точное значениеarctan(-1)
    12Найти точное значениеcos(45 град. )
    13Найти точное значениеcos(30 град. )
    14Найти точное значениеtan(60)
    15Найти точное значениеcsc(45 град. )
    16Найти точное значениеtan(60 град. )
    17Найти точное значениеsec(30 град. )
    18Найти точное значениеcos(60 град. )
    19Найти точное значениеcos(150)
    20Найти точное значениеsin(60)
    21Найти точное значениеcos(pi/2)
    22Найти точное значениеtan(45 град. )
    23Найти точное значениеarctan(- квадратный корень из 3)
    24Найти точное значениеcsc(60 град. )
    25Найти точное значениеsec(45 град. )
    26Найти точное значениеcsc(30 град. )
    27Найти точное значениеsin(0)
    28Найти точное значениеsin(120)
    29Найти точное значениеcos(90)
    30Преобразовать из радианов в градусыpi/3
    31Найти точное значениеtan(30)
    32Преобразовать из градусов в радианы45
    33Найти точное значениеcos(45)
    34Упроститьsin(theta)^2+cos(theta)^2
    35Преобразовать из радианов в градусыpi/6
    36Найти точное значениеcot(30 град. )
    37Найти точное значениеarccos(-1)
    38Найти точное значениеarctan(0)
    39Найти точное значениеcot(60 град. )
    40Преобразовать из градусов в радианы30
    41Преобразовать из радианов в градусы(2pi)/3
    42Найти точное значениеsin((5pi)/3)
    43Найти точное значениеsin((3pi)/4)
    44Найти точное значениеtan(pi/2)
    45Найти точное значениеsin(300)
    46Найти точное значениеcos(30)
    47Найти точное значениеcos(60)
    48Найти точное значениеcos(0)
    49Найти точное значениеcos(135)
    50Найти точное значениеcos((5pi)/3)
    51Найти точное значениеcos(210)
    52Найти точное значениеsec(60 град. )
    53Найти точное значениеsin(300 град. )
    54Преобразовать из градусов в радианы135
    55Преобразовать из градусов в радианы150
    56Преобразовать из радианов в градусы(5pi)/6
    57Преобразовать из радианов в градусы(5pi)/3
    58Преобразовать из градусов в радианы89 град.
    59Преобразовать из градусов в радианы60
    60Найти точное значениеsin(135 град. )
    61Найти точное значениеsin(150)
    62Найти точное значениеsin(240 град. )
    63Найти точное значениеcot(45 град. )
    64Преобразовать из радианов в градусы(5pi)/4
    65Найти точное значениеsin(225)
    66Найти точное значениеsin(240)
    67Найти точное значениеcos(150 град. )
    68Найти точное значениеtan(45)
    69Вычислитьsin(30 град. )
    70Найти точное значениеsec(0)
    71Найти точное значениеcos((5pi)/6)
    72Найти точное значениеcsc(30)
    73Найти точное значениеarcsin(( квадратный корень из 2)/2)
    74Найти точное значениеtan((5pi)/3)
    75Найти точное значениеtan(0)
    76Вычислитьsin(60 град. )
    77Найти точное значениеarctan(-( квадратный корень из 3)/3)
    78Преобразовать из радианов в градусы(3pi)/4
    79Найти точное значениеsin((7pi)/4)
    80Найти точное значениеarcsin(-1/2)
    81Найти точное значениеsin((4pi)/3)
    82Найти точное значениеcsc(45)
    83Упроститьarctan( квадратный корень из 3)
    84Найти точное значениеsin(135)
    85Найти точное значениеsin(105)
    86Найти точное значениеsin(150 град. )
    87Найти точное значениеsin((2pi)/3)
    88Найти точное значениеtan((2pi)/3)
    89Преобразовать из радианов в градусыpi/4
    90Найти точное значениеsin(pi/2)
    91Найти точное значениеsec(45)
    92Найти точное значениеcos((5pi)/4)
    93Найти точное значениеcos((7pi)/6)
    94Найти точное значениеarcsin(0)
    95Найти точное значениеsin(120 град. )
    96Найти точное значениеtan((7pi)/6)
    97Найти точное значениеcos(270)
    98Найти точное значениеsin((7pi)/6)
    99Найти точное значениеarcsin(-( квадратный корень из 2)/2)
    100Преобразовать из градусов в радианы88 град.

    что такое градус и радиан, как перевести градусы в радианы и обратно

    Содержание:

    • Что такое градус
    • Что такое радиан
    • Как перевести градусы в радианы
    • Как перевести радианы в градусы
    • Примеры

    Содержание

    • Что такое градус
    • Что такое радиан
    • Как перевести градусы в радианы
    • Как перевести радианы в градусы
    • Примеры

    Что такое градус

    Во многих задачах по геометрии, физике и другим дисциплинам приходится решать примеры, где по условию даны, либо неизвестны углы. Данные элементы в распространенных случаях измеряют в градусах или радианах. Упростить решение порой достаточно просто, если уметь переводить эти величины, то есть выражать градусы в радианах или наоборот. Существует ряд полезных формул и закономерностей, которые помогут справиться с подобными задачами.

    Градусами (знак °), наряду с минутами и секундами, называют стандартизированные единицы, в которых определяют меру углов на плоскости. С помощью градусов также вводят данные величин в рамках картографической деятельности, вычисляют координаты, характерные для каких-либо точек на поверхности Земли, рассчитывают азимут.

    Если представить себе некую окружность, то есть полноценный оборот, то ее мера будет соответствовать 360°. Развернутый угол, характеризующийся, как половина полного оборота, равен 180°. Из курса геометрии известно, что каждый прямой угол составляет 90°.

    Осторожно! Если преподаватель обнаружит плагиат в работе, не избежать крупных проблем (вплоть до отчисления). {\circ }={\frac {400}{360}}градов=1,(1) градов=1,11111111111… градов \)

    Что такое радиан

    ​Радиан является углом, который равен дуге с длинной, соответствующей ее радиусу.

    Радианной мерой называют меру углов, которая предполагает использование единиц измерения углов в виде 1 радиана.

    Таким образом, радианная мера какого-либо угла представляет собой соотношение данного угла к радиану. Согласно формулировке термина, полный угол по величине соответствует  \(2\pi \)  радиан.

    Вычисление радианной меры заключается в определении того, как относится длина дуги окружности, расположенная между сторонами угла, к радиусу рассматриваемой окружности. При этом центральная точка окружности должна совпадать с вершиной этого угла.

    В процессе решения примеров по геометрии, чтобы рассчитать, чему равна радианная мера угла, принято изображать единичную окружность, центральная точка которой совпадает с вершиной угла. В результате радианная мера угла составит величину длины дуги единичной окружности, заключенной между сторонами угла.

    Заметим наличие пропорциональной зависимости между такими величинами, как длина дуги окружности и ее угловая мера с радиусом. Существует соотношение, характерное для длины дуги окружности, радиус которой равен R, а угловая величина соответствует \(\alpha\) и выражена в радианах:

    \(\alpha \cdot R \)

    Исходя из того, что угловая мера в радианах вычисляется, как отношение длины дуги окружности и длины радиуса рассматриваемой окружности, допустимо принять угол в радианном исчислении за величину безразмерного типа.

    Как перевести градусы в радианы

    Представим, что имеется некоторая окружность с радиусом r. Пусть на диаметр рассматриваемой окружности опирается угол, являющийся центральным. Если потребуется вычислить, чему равен этот угол в радианах, то целесообразно найти частное от деления длины дуги на длину радиуса окружности. Угол, речь о котором идет в примере, можно сопоставить с длиной дуги, которая определяется, как ½ от длины окружности:

    \(\pi \cdot r \)

    С помощью деления длины дуги на радиус получится выяснить, чему равен угол в радианах:

    \(\frac{\pi \cdot r}{r} = \pi рад\). {\circ }\cdot 60’\cdot 60»}{2\pi }}\approx 206264{,}8»\).

    Примеры

    Задача 1

    ​Предположим, что имеется некий угол \alpha, радианная мера которого составляет 3,2 рад. Требуется перевести это значение в градусы.

    Решение

    Воспользуемся записанными ранее закономерностями и выполним перевод радиан в градусы:

    \(3,2 рад = \frac{3,2 \cdot 180}{\pi } ° \approx \frac{3,2 \cdot 180}{3,14 } ° \approx \frac{576}{3,14 }\approx 183,4°\)

    Ответ: 3,2 рад \(\approx 183,4°\).

    Задача 2

    Градусная мера угла составляет 47°. Необходимо, используя правила перевода, преобразовать градусную меру в радианную и дать ответ в радианах.

    Решение

    С помощью уже знакомого правила перевода градусов в радианы выполним соответствующие вычисления:

    \(47° \approx\frac{47 \cdot 3,14}{180} \approx 0,82 рад\).

    Ответ: \(47° \approx 0,82\) рад.

    Задача 3

    С помощью соотношения, демонстрирующего зависимости единиц измерения углов, представить в радианах угол, градусная мера которого составляет \(5^\circ43’46»\). \circ43’46» = 0{,}1~\mathrm {rad}\).

    Насколько полезной была для вас статья?

    У этой статьи пока нет оценок.

    градусов в радианы: преобразовать и рассчитать

    Мы используем разные единицы измерения для определения длины (дюймы, м или км), массы (кг, мг или г) или температуры (градусы Цельсия или Фаренгейта), а математика позволяет нам конвертировать между этими единицами измерения. Точно так же угол окружности можно измерить двумя способами: с градусами и с радианами , и с помощью формул мы можем конвертировать между ними.

    Что такое радиан?

    Напомним, что градус угла ( ° ) измеряет вращение между каждым плечом. Полный оборот или оборот составляет 360 ° , а полукруг — 180 ° . Чтобы понять, что радиана измеряют или радиана ( рад ), мы должны сначала обсудить центральный угол — угол, вершина которого находится в центре круга. Радиан центрального угла измеряется соответствующей ему дугой, длина которой равна радиусу. Другими словами, радиан — это угловая мера результирующего дуга , когда радиус изогнут вокруг окружности. См. иллюстрацию ниже.

    Центральный угол 360 ° имеет радиан 2π, что выражается в уравнении:
    2π радиан  = 360°
    буду использовать для преобразования между двумя единицами измерения.

    Как преобразовать градусы в радианы :

    Используйте приведенные ниже формулы для преобразования градусов в радианы.

    Чтобы преобразовать градуса в радианы ,
    умножьте число градусов на π / 180° .

    Чтобы преобразовать радиана в градуса,
    умножьте радиана на 180° / π .

    Примеры:

    1. В каких радианах измеряется 150°?
      Решение:
      Умножьте 150° на π / 180°.
      150° x (π / 180°) = (150π / 180°) = (5π / 6) радиан или ≈ 0,83π радиан .
      Примечание: Градусы в 150° и 180° были отменены во время умножения.
    2. В каких радианах измеряется угол 36°?
      Решение:
      Умножьте 36° на π / 180°.
      36° x (π / 180°) = (36π / 180°) = (π / 5) радиан или ≈ 0,2π радиан .
    3. Какова градусная мера дуги, мера которой равна 3π/8 радиан?
      Решение:
      Умножьте 3π / 8 на 180° / π.
      (3 π / 8) x (180° / π ) = 540° / 8 = 67,5°
    4. Какова градусная мера дуги, мера которой равна 1,8π радиан?
      Решение:
      Умножьте 1,8π на 180° / π.
      1,8 π x (180° / π ) = 324°
    5. Какова градусная мера угла, мера которого равна 12 радианам?
      Решение:
      Умножить 12 на 180°/π.
      12 x (180°/π) = 2160°/π 687,9°

    Спасибо за прочтение. Мы надеемся, что это эффективно! Не стесняйтесь возвращаться на эту страницу, если у вас возникнут вопросы о преобразовании  градуса в радиана.

    Ознакомьтесь с некоторыми другими сообщениями в нашем блоге или инвестируйте в свое будущее с помощью одного из наших курсов самообучения!
    Щелкните здесь, чтобы ознакомиться с Руководством по подготовке к экзамену AP по программе Calculus AB 2021 ! Конвертер

    градусов в радианы

    Создано Mariamy Chrdileli

    Отзыв от Julia Żuławinska

    Последнее обновление: 02 февраля 2023 г.

    Содержание:
    • Как преобразовать градусы в радианы?
    • Как работает конвертер градусов в радианы?
    • Другие сопутствующие инструменты
    • Часто задаваемые вопросы

    Добро пожаловать в Omni конвертер градусов в радианы, простой инструмент, который поможет вам конвертировать градусы в радианы.

    У вас проблемы с переводом градусов в радианы? Не волнуйся; вы определенно в правильном месте! Приходите, чтобы узнать, что такое радиан, , как преобразовать градусы в радианы, и как использовать наш удобный конвертер!👩‍🏫

    Как преобразовать градусы в радианы?

    Начнем с самого начала — что такое радиан? Радиан (обозначается символом рад) является стандартной единицей измерения углов. 1 радиан составляет примерно 57,2958 градуса.

    Чтобы преобразовать градуса в радианы, , вы можете использовать следующую формулу:

    радианы = π/180° × градусы

    Например, если вы пытаетесь определить угол 90° в радианах, вы бы выполнили следующие вычисления:

    радиан = π/180° × 90° = π/2 рад ≈ 1,5708 рад

    Звучит громоздко? Не беспокойтесь, конвертер омни-градусов в радианы сделает всю работу за вас!

    Как работает конвертер градусов в радианы?

    Теперь, когда вы знаете, как преобразовать градусы в радианы, давайте обсудим, как работает калькулятор преобразования градусов в радианы ; это довольно просто!

    Все, что вам нужно сделать, это ввести интересующий вас градус, и калькулятор степени в радианах будет конвертировать введенные данные в радианы. По умолчанию вы можете ввести единицу измерения углов в градусах, но вы также можете изменить единицу измерения по умолчанию и выбрать ту, которая лучше всего соответствует вашим потребностям!

    Считаете ли вы, что конвертер градусов в радианы полезен? Попробуйте другие сопутствующие инструменты:

    • Угловой преобразователь;
    • Конвертер
    • радиан в градусы;
    • Перевод градусов минут секунд в десятичные градусы;
    • Преобразователь десятичных градусов в градусы минуты секунды;
    • Конвертер градусов в минуты; и
    • Конвертер градусов в секунды.

    © 2015 - 2019 Муниципальное казённое общеобразовательное учреждение «Таловская средняя школа»

    Карта сайта